Marketing Management test 3 study

अब Quizwiz के साथ अपने होमवर्क और परीक्षाओं को एस करें!

90) Explain the concept of greenwashing along with an example.

Student answers will vary. Greenwashing gives products the appearance of being environmentally friendly without living up to that promise. An automobile manufacturer who promotes its cars as being environmentally friendly when the company is in fact a major polluter, would be an example of greenwashing.

82) Which of the following types of power is objectively observable? A) coercive power B) legitimate power C) group power D) expert power E) referent power

A

Jason is a technology enthusiast who is happy to conduct alpha and beta testing and report on early weaknesses of consumer electronics products. He is most likely to fall in the ________ adopter category.

Innovator

144) What function does a company's public relations department perform when it promotes understanding of the organization through internal and external communications? A) lobbying B) corporate communications C) press relations D) product publicity E) counseling

B

138) Media coverage and telecasts only expose the brand and do not necessarily embellish its meaning in any direct way.

TRUE

86) Although consumers may have fairly good knowledge of the range of prices involved, very few can accurately recall specific prices of products.

TRUE Page Ref: 387 Objective: 1 Difficulty: Easy

114) Companies sometimes initiate price cuts in an attempt to dominate the market through lower costs.

TRUE Page Ref: 407 Objective: 4 Difficulty: Easy

75) Cause-related marketing efforts are unlikely to backfire as customers generally view the company's motives as genuine.

FALSE

When Sony introduced the first high-definition television to the Japanese market in 1990, it was prices at $43,000, which is an example of _______ pricing.

Market-Skimming

The least costly way of market testing consumer goods it ________.

Sales-Wave Research

The first step in setting pricing policy is ________.

Selecting the pricing objective

98) Imagine that you are in charge of creating a distinctive store atmosphere at a clothing retailer. What can you do to make your store stand out in the customer experience?

Student answers will vary. Answers can include the use of fragrances, music, store design and layout, and activities.

118) The historical approach correlates past sales to past advertising expenditures using advanced statistical techniques.

TRUE

31) A marketing channel overcomes the time, place, and possession gaps that separate goods and services from those who need or want them.

TRUE

119) Motivational or "borrowed interest" devices — such as the presence of cute babies, frisky puppies, popular music, or provocative sex appeals — are often employed to attract attention and raise involvement with an ad.

TRUE

80) Reach of an advertising message is most important when launching new products, flanker brands, extensions of well-known brands, or infrequently purchased brands.

TRUE

84) Purchase decisions are based on how consumers perceive prices and what they consider the current actual price to be-not the marketer's stated price.

TRUE Page Ref: 386 Objective: 1 Difficulty: Easy

When are buyers more price sensitive?

When buyers can store the product

Which percentage of all new products is truly innovative and new to the world?

Fewer than 10%

72) Sustainability ratings exist, but there is no consistent agreement about what metrics are appropriate.

TRUE

81) The higher the forgetting rate associated with a brand, product category, or message, the lower the warranted level of repetition.

FALSE

83) Endearment gives products the appearance of being environmentally friendly without living up to that promise.

FALSE

83) Outdoor advertising is more effective at creating new brand associations than enhancing brand awareness or brand image.

FALSE

________ consists of putting goods in boxes or trailers that are easy to transfer between two transportation modes. A) Containerization B) Haulage C) Inventory carrying D) Order processing E) Warehousing

A) Containerization

59) Electronic shelf labeling allows retailers to ________. A) check inventory levels instantaneously B) order electronically from suppliers C) run continual promotional messages D) advertise sales and special offers E) change price levels instantaneously

E

13) Sales agents and brokers are called facilitators in a marketing channel.

FALSE

63) Department stores only face competition from other department stores as other retailing forms are not a threat to their sales and profitability.

FALSE

64) Intensive distribution relies on only some of the intermediaries willing to carry a particular product.

FALSE

Which adopter group is the smallest?

Innovators

93) Cocreation can help a company create favorable word of mouth.

TRUE Page Ref: 577 Objective: 3 Difficulty: Easy

Among the best practitioners of ________, or charging a fairly low price for a high-quality offering are IKEA, Target, and Southwest Airlines.

Value Pricing

77) Many companies focus on multiple cause-related marketing programs to simplify execution and maximize impact.

FALSE

77) Print advertising, newspapers and magazines, is generally acknowledged as the most powerful advertising medium.

FALSE

87) An advertiser makes "local buys" when it buys TV time in just a few markets or in regional editions of magazines.

FALSE

70) If the Coolest Cooler introduced its cooler to the general market and developed communications around the idea that it was going to develop awareness, interest, evaluation, trial, and then adoption, it would be using the innovation-adoption model of consumer responses.

TRUE

89) Some high-volume products may have such high handling costs that they are less profitable and deserve less shelf space than low-volume products.

TRUE

91) Low prices on some items serve as traffic builders or loss leaders for retailers.

TRUE

92) EDLP can lead to lower advertising costs and higher retail profits.

TRUE

93) In the case of prestige goods, the demand curve sometimes slopes upward.

TRUE Page Ref: 390 Objective: 2 Difficulty: Easy

94) Companies prefer customers who are less price sensitive.

TRUE Page Ref: 391 Objective: 2 Difficulty: Easy

107) Psychological discounting involves setting an artificially high price and then offering the product at substantial savings.

TRUE Page Ref: 405 Objective: 3 Difficulty: Easy

21) Kraft utilized customers to name its new flavor of its iconic Vegemite product, and their selection, iSnack 2.0, increased sales dramatically.

FALSE

66) Which of the following types of advertising timing patterns calls for spending all advertising dollars in a single period? A) frequency capping B) flighting C) concentration D) pulsing E) continuity

C

68) Advertisements for which of the following product categories would merit a continuous advertising timing pattern the most? A) air conditioners B) life insurance C) breakfast cereal D) automobiles E) holiday package tours

C

69) ________ is an advertising timing pattern that calls for advertising during a certain period, followed by a period with no advertising, followed by a second specific period of advertising activity. A) Pulsing B) Continuity C) Flighting D) Concentration E) Frequency capping

C

141) ________ consists of putting goods in boxes or trailers that are easy to transfer between two transportation modes. A) Containerization B) Haulage C) Inventory carrying D) Order processing E) Warehousing

A

141) ________ is a marketing communications tool that includes a variety of programs to promote or protect a company's image or individual products. A) Public relations B) Advertising C) Sales promotion D) Personal selling E) Direct marketing

A

40) Which of the following equations accurately describes the total number of exposures (E) of an advertising message through a given medium? A) E = reach × frequency B) E = (reach × frequency) / impact C) E = reach × frequency × impact D) E = (reach + frequency) / impact E) E = frequency / reach

A

73) ________ seeks to determine whether an ad is communicating effectively. A) Copy testing B) Flighting C) Pulsing D) Frequency capping E) Square inch analysis

A

79) A manufacturer offers its intermediaries an extra benefit for performing a promotional activity. This is an example of the use of ________ power. A) reward B) coercive C) functional D) expert E) referent

A

8) A firm uses its sales force to sell to large accounts and outbound telemarketing to sell to medium-sized accounts. The firm is using ________ marketing. A) hybrid B) pull C) personalized D) vertical E) internal

A

99) Which of the following is a sales promotion tool that typically does NOT build brand image? A) consumer refund offers B) free samples C) premiums related to the product D) frequency awards E) coupons that include a selling message

A

53) Which of the following is a disadvantage of using magazines as an advertising medium? A) low geographic and demographic selectivity B) long ad purchase lead time C) low-quality reproduction D) small "pass-along" readership E) lack of credibility

B

53) ________ refers to the ability to meet humanity's needs without harming future generations. A) Greenwashing B) Sustainability C) Ecological footprinting D) Scalability E) Legal practice

B

Aldi, Lidl, Dollar General, and Family Dollar are examples of ________, as they carry a more restricted merchandise mix than discount stores at even lower prices. A) off-price retailers B) extreme value stores C) superstores D) convenience stores E) specialty stores

B) extreme value stores

Because shelf space is scarce, many supermarkets now charge a ________ for accepting a new brand, to cover the cost of listing and stocking it. A) retainer B) slotting fee C) residual fee D) contingent fee E) royalty

B) slotting fee

In ________, a salesperson goes to the home of a host who has invited friends, demonstrates the products, and takes orders. A) catalog marketing B) franchising C) direct-response selling D) network marketing E) direct marketing

D) network marketing

155) MPR is effective in blanketing local communities and reaching specific groups and hence has to be planned separately from the less cost-effective advertising.

FALSE

126) Personal influence carries especially great weight when products are inexpensive, risk-free, or purchased frequently.

FALSE

132) The annual plan control process begins with measuring performance.

FALSE

136) Self-audits tend to provide more objective information than audits conducted by external consultants.

FALSE

15) Reinforcement advertising aims to stimulate repeat purchase of products and services.

FALSE

151) The main objective of marketing public relations is to secure editorial space in print and broadcast media to promote or "hype" a product, service, idea, place, person, or organization.

FALSE

154) MPR can build credibility by placing stories in the media to bring attention to a product, service, person, organization, or idea.

FALSE

65) Intensive distribution is becoming a mainstay for specialists looking for an edge in markets increasingly driven by price.

FALSE

69) Although salespeople are legally prohibited from saying things about their products that are not true, they may legally suggest things about competitors' products that are not true.

FALSE

99) Social marketing is a new trend in marketing.

FALSE Page Ref: 640 Objective: 3 AACSB: Ethical understanding and reasoning abilities Difficulty: Easy

Jordan pays overhead each month, including her company's bills for rent, heat, interest, and salaries, which are examples of __________.

Fixed Costs

74) Which of the following is true for direct product profitability? A) It is highly correlated with the gross margin on a product. B) It is negligible compared to the gross margin on a product. C) It bears little relation to the gross margin on a product. D) It is significantly lower than the gross margin on a product. E) It is exactly the same as the gross margin on a product.

C

73) Because of insincere firms jumping on the "green" bandwagon, consumers bring a healthy skepticism to environmental claims, but they are also unwilling to sacrifice product performance and quality.

TRUE

115) Two-sided messages are more effective with more educated audiences and those who are initially opposed.

TRUE

49) In the growth stage of a product's life cycle, demand has its own momentum through word of mouth and interactive marketing.

TRUE

92) A firm is said to be following a market-skimming pricing strategy, if it introduces a product into the market at a high price and slowly drops the price over time.

TRUE Page Ref: 390 Objective: 2 Difficulty: Easy

61) Which of the following advertising practices involves advertisers paying filmmakers to have their products make cameo appearances in movies and television shows? A) brand extension B) flyposting C) co-branding D) product placement E) ambush marketing

D

13) Which of the following channel functions constitute only a backward flow? A) movement of physical goods B) placing orders with manufacturers C) persuasive communication D) storage of physical goods E) overseeing actual transfer of ownership

b

17) A factory outlet is an example of a(n) ________ retailer. A) off-price B) specialty C) discount D) department E) catalog

A

2) ________ advertising aims to create brand awareness and knowledge of new products or new features of existing products. A) Informative B) Corporate C) Reinforcement D) Persuasive E) Reminder

A

24) When customers fall into different user groups with distinct buying preferences and practices, a ________ organization is desirable. A) market-management B) product-management C) brand-management D) geographic E) functional

A

31) Which of the following is generally acknowledged as the most powerful advertising medium and reaches a broad spectrum of consumers at low cost per exposure? A) television B) radio C) newspapers D) magazines E) billboards

A

49) Armon Apparels designs, manufactures, and distributes athletic apparel and accessories for men and women. The company has only nine distributors across the United States. These distributors control a nationwide network of 600 retailers. The company does not sell its products through other channels. This is an example of ________ distribution. A) selective B) intensive C) exclusive D) internal E) passive

A

47) Top and middle management are primarily responsible for ________. A) annual-plan control B) efficiency control C) profitability control D) technological control E) innovation control

A Page Ref: 643 Objective: 5 Difficulty: Easy

Bill and Josh are considering opening a retail store. They have identified their target market and location and are finalizing the details of the merchandise they will carry. Since the neighborhood is rundown and the customers in the area are very price-conscious, Bill and Josh want to offer goods from well-known brands, but at lower rates than the full retail prices of the products. They choose to stock excess production from manufacturers or goods that have remained unsold at other retailers. This is a description of a(n) ________ retailer . A) off-price B) specialty C) discount D) department E) catalog

A) off-price

As inventory draws down, management must know at what stock level to request additional stock. This stock level is called the ________. A) reorder point B) least fixed point C) point of divergence D) inflection point E) critical point

A) reorder point

27) An independent retailer using a central buying organization and joint promotion efforts with other retailers is part of a ________. A) corporate chain store B) voluntary chain C) retailer cooperative D) merchandising conglomerate E) franchise organization

C

3) A manufacturer uses the company's sales force and trade promotions to carry, promote, and sell products to end users. Which of the following strategies is this manufacturer using? A) personalization strategy B) tailoring strategy C) push strategy D) pull strategy E) consumer promotion strategy

C

60) All response hierarchy models of the communication process assume the buyer passes through cognitive, affective, and behavioral stages, in that order. Which of the following product categories lends itself most appropriately to such a "learn-feel-do" sequence? A) clothes B) dishwashers C) real estate D) personal computer E) air tickets

C

Avon, Tupperware, and Southwestern Company of Nashville are among companies in the​ multibillion-dollar ________​ industry, which involves selling​ door-to-door or at home sales parties. A. catalog showroom B. direct marketing C. direct selling D. automatic vending E. buying services

C

Which of the following is associated with​ post-purchase services? A. An image of fairness B. An image of reliability C. Shipping and delivery options D. Advertising costs E. Higher retail profits

C

​________ includes planning the infrastructure to meet​ demand, and then implementing and controlling the physical flows of materials and final goods from points of origin to points of​ use, to meet customer requirements at a profit. A. Electronic data interchange​ (EDI) B. Lean manufacturing C. Market logistics D. Electronic funds transfer​ (EFT) E. ​Supply-chain management

C

137) Armordo is a famous vacuum cleaner brand in Africa. Clara has heard of Armordo and she knows that the product is a success. She has started considering whether or not to buy the vacuum cleaner. What stage of the consumer-adoption process is Clara in? A) awareness B) adoption C) evaluation D) trial E) interest

C Diff: 3 LO: 15.8: What factors affect the rate of diffusion and consumer adoption of newly launched products and services? AACSB: Analytical thinking; Application of knowledge

67) When museums charge a lower admission fee to students and senior citizens, then this form of price discrimination is known as ________. A) location pricing B) channel pricing C) customer-segment pricing D) special-customer pricing E) loss-leader pricing

C Page Ref: 406 Objective: 3 Difficulty: Easy

73) ________ refers to selling below cost with the intention of destroying competition. A) Bid rigging B) Loss-leader pricing C) Predatory pricing D) Price discrimination E) Price penetration

C Page Ref: 407 Objective: 3 Difficulty: Easy

79) When a company maintains its price but removes or prices separately one or more elements that were part of the former offer, such as free delivery or installation, it is known as ________. A) escalating B) differentiation C) unbundling D) reverse discounting E) delayed quotation pricing

C Page Ref: 408 Objective: 5 Difficulty: Easy

46) ________ is a "hybrid" data collection technique that combines self-explicated importance ratings with pair-wise trade-off tasks. A) Sales wave analysis B) Simulative data testing C) Adaptive conjoint analysis D) Business analysis E) Exponential data collection

C Page Ref: 584 Objective: 4 Difficulty: Easy

101) The purpose of profitability control is to ________. A) examine whether the planned results are being achieved B) examine where the company is making and losing money C) evaluate and improve the spending efficiency and impact of marketing expenditures D) examine whether the company is pursuing its best opportunities with respect to markets, products, and channels E) understand the efficiency of the sales force, advertising, sales promotion, and distribution

B

95) Which of the following methodologies takes the list of desired customer attributes (CAs) generated by market research and turns them into a list of engineering attributes (EAs) that engineers can use? A) quality control processes B) quality function deployment C) rapid prototyping D) marketing control E) control system formation

B Diff: 2 LO: 15.6: What is the best way to manage concept and strategy development? AACSB: Reflective thinking

83) Modern firms use the ________ tool to design products on a computer and then produce rough models to show potential consumers for their reactions. A) morphological analysis B) rapid prototyping C) concept testing D) perceptual mapping E) conjoint analysis

B Diff: 2 LO: 15.6: What is the best way to manage concept and strategy development? AACSB: Reflective thinking; Application of knowledge

144) Five characteristics influence the rate of adoption of an innovation. One of these is ________. A) marketing expertise B) relative advantage C) packaging attractiveness D) government regulations E) place of value exchange

B Diff: 2 LO: 15.8: What factors affect the rate of diffusion and consumer adoption of newly launched products and services? AACSB: Reflective thinking

29) A ________ is a wholesaler-sponsored group of independent retailers engaged in bulk buying and common merchandising. A) retailer cooperative B) voluntary chain C) consumer cooperative D) merchandising conglomerate E) franchise organization

B Diff: 2 LO: 18.1: What major types of marketing intermediaries occupy this sector? AACSB: Reflective thinking

10) Which of the following is the first step in setting a pricing policy? A) selecting a pricing method B) selecting the pricing objective C) determining demand D) estimating cost E) analyzing competitors' costs, prices, and offers

B Page Ref: 389 Objective: 2 Difficulty: Easy

16) A market-penetration pricing strategy is most suitable when _______. A) a low price slows down market growth B) production and distribution costs fall with accumulated production experience C) a high price dissuades potential competitors from entering the market D) the market is characterized by inelastic demand E) a low price encourages actual competition

B Page Ref: 389 Objective: 2 Difficulty: Moderate

10) Which of the following conditions necessitates that the objective of advertising should be to stimulate more usage of a product? A) The advertised product belongs to a nascent product category. B) The company is not the market leader. C) The advertised brand is superior to the market leader. D) The product class is mature. E) Brand usage for the product is very high.

D

In the ________ type of retailing, customers usually find their own goods, although they can ask salespeople for assistance. A) self-service B) self-selection C) full-service D) limited service E) limited-selection

B) self-selection

80) Which method for establishing the total marketing communications budget sets communication budgets to achieve the same amount of share-of-voice as competitors? A) comparative-parity method B) objective-and-task method C) affordable method D) competitive-parity method E) percentage-of-sales method

D

34) During which of the following stages in new product development decision making do managers analyze if they can find a good thought consumers say they would try? A) idea generation B) idea screening C) business analysis D) concept development and testing E) product development and testing

D Diff: 2 LO: 15.3: What organizational structures and processes do managers use to oversee new-product development? AACSB: Reflective thinking

57) Which of the following is an advantage of sales-wave research? A) It gives surprisingly accurate results on advertising effectiveness. B) It allows the brand to gain favorable shelf position. C) It guarantees immediate brand recognition and high sales volume. D) It can be implemented quickly with a fair amount of security. E) It easily creates a buzz in the minds of customers.

D Page Ref: 588 Objective: 4 Difficulty: Easy

Staples is a giant retailer that concentrates on selling office supplies. Staples is an example of a(n) ________. A) ambush marketer B) supercenter C) megamarketer D) category killer E) guerilla marketer

D) category killer

American businessman King Gillette pioneered the sales model in which razor handles were given away for free or sold at a loss, but sales of disposable razor blades were very profitable. This is known as the ________ model. A) two-tiered pricing B) predatory pricing C) cross selling D) loss leading E) product churning

D) loss leading

126) Suppose the manufacturer pays a commission on every car sold. Then, the salesperson's commission is classified as a(n) ________. A) cost of labor B) traceable common cost C) nontraceable common cost D) advertising cost E) direct cost

E

120) Celebrities are likely to be effective when they are credible or personify a key product attribute.

TRUE

13) An advertising objective is a specific communications task and achievement level to be accomplished with a specific audience in a specific period of time.

TRUE

131) Annual-plan control involves the use of financial analysis to evaluate the performance of marketing plans.

TRUE

131) Marketing communications budgets tend to be higher when there is much change in the marketing program over time and more complex customer decision making.

TRUE

39) A customer-management organization deals with individual customers rather than the mass market or market segments.

TRUE

84) Market managers are staff people, with duties like those of a product manager.

TRUE Page Ref: 629 Objective: 2 Difficulty: Easy

93) Sustainability ratings exist, but there is no consistent agreement about what metrics are appropriate.

TRUE Page Ref: 635 Objective: 3 AACSB: Ethical understanding and reasoning abilities Difficulty: Easy

102) The actual success of a social marketing program should be evaluated in terms of the program objectives.

TRUE Page Ref: 642 Objective: 3 AACSB: Ethical understanding and reasoning abilities Difficulty: Easy

103) Desktop marketing gives marketers information and decision structures on computer dashboards.

TRUE Page Ref: 643 Objective: 5 Difficulty: Easy

1) ________ are sets of interdependent organizations participating in the process of making a product or service available for use or consumption. A) Marketing channels B) Interstitials C) Communication channels D) Sales territories E) Marketing terrains

a

70) Winstar is a large scale manufacturer which has more than a hundred partners across the globe. When making decisions concerning distribution and channel optimization, the company invites members from its channel partners to be part of its advisory committee. This helps the company maintain harmony with its partners. Which of the following conflict resolution techniques is Winstar using? A) diplomatic counselling B) mediation C) arbitration D) co-option E) joint membership

d

38) A(n) ________ is an elaborated version of a product idea expressed in consumer terms. A) test brand B) alpha product C) beta product D) business schedule E) product concept

E Page Ref: 581 Objective: 4 Difficulty: Easy

30) Marketing has sole ownership of customer interaction.

FALSE

80) Social marketing programs take little time to develop and are generally easy to implement.

FALSE

91) Coercive and referent power are subjective and depend on the ability and willingness of parties to recognize them.

FALSE

102) The U.S. government often uses Dutch auctions to procure supplies.

FALSE Page Ref: 402 Objective: 2 Difficulty: Easy

109) In first-degree price discrimination, the seller charges less to buyers of larger volumes.

FALSE Page Ref: 406 Objective: 3 Difficulty: Easy

76) Marketing has sole ownership of customer interaction.

FALSE Page Ref: 625 Objective: 2 Difficulty: Easy

116) In a price-war trap, higher-priced competitors match the firm's lower prices but have longer staying power because of deeper cash reserves.

FALSE Page Ref: 408 Objective: 4 Difficulty: Easy

78) Organic growth refers to increasing the profitability of the organization by increasing employee productivity.

FALSE Page Ref: 569 Objective: 1 Difficulty: Easy

83) Shorter product life cycles are common in industries characterized by low level of competition.

FALSE Page Ref: 572 Objective: 1 Difficulty: Easy

84) Companies should use normal investment criteria to budget for new-product developments.

FALSE Page Ref: 573 Objective: 2 Difficulty: Easy

95) Attribute listing lists several ideas and considers each in relationship to the others.

FALSE Page Ref: 579 Objective: 3 Difficulty: Easy

97) Morphological analysis starts by forming a new dimension and then thinks about the possible problems of the dimension.

FALSE Page Ref: 579 Objective: 3 Difficulty: Easy

98) A DROP-error occurs when the company accepts a bad idea.

FALSE Page Ref: 580 Objective: 3 Difficulty: Easy

100) Concept testing means validating the product concept by discussing within the design group.

FALSE Page Ref: 582 Objective: 4 Difficulty: Easy

102) Perceived value refers to the gap between actual needs and satisfied needs of a customer.

FALSE Page Ref: 582 Objective: 4 Difficulty: Easy

109) Marketers can set communications objectives at any level of the hierarchy-of-effects model.

TRUE

150) As inventory draws down, management must know at what stock level to place a new order. This stock level is called the order point.

TRUE

12) Marketing channels are the set of pathways a product or service follows after production, culminating in purchase and consumption by the final end user.

TRUE

123) Merchant wholesalers are independently owned businesses that take title to the merchandise they handle.

TRUE

123) The principle of congruity implies that communicators can use their good image to reduce some negative feelings toward a brand but in the process might lose some esteem with the audience.

TRUE

124) The beginning communications objective associated with a new-to-the world product — like an electric car — is always establishing category need.

TRUE

15) Companies should first think of the target market and then design the supply chain backward from that point. This strategy is called demand chain planning.

TRUE

152) Companies who want to carry near-zero inventory should build for order, not for stock.

TRUE

32) Companies producing a variety of products and brands often establish a product- (or brand-) management organization.

TRUE

33) All functions in a marketing channel use up scarce resources and can be shifted among channel members.

TRUE

35) There are three types of product-team structures: vertical, triangular, and horizontal.

TRUE

43) A franchise organization is a corporate retail organization.

TRUE

68) Distributors' territorial rights define the terms under which the producer will enfranchise other distributors.

TRUE

69) Firms are increasingly recognizing the importance of influencing consumers at the point of purchase.

TRUE

79) Researchers studying print advertisements report that the picture, headline, and copy matter in that order.

TRUE

of the following terms best represents this practice? A) channel integration B) mass customization C) online personalization D) push strategy E) internal marketing

A

134) As inventory draws down, management must know at what stock level to request additional stock. This stock level is called the ________. A) reorder point B) least fixed point C) point of divergence D) inflection point E) critical point

A Diff: 1 LO: 18.6: What are some important issues in logistics? AACSB: Reflective thinking

37) Full-service retailers generally have the lowest costs of all retail types.

FALSE

112) Communications effectiveness depends solely on the content of a message, irrespective of how it is expressed.

FALSE

114) Advanced supply-distributor arrangements for administered vertical marketing systems do not rely on distribution programming.

FALSE

114) One-sided presentations that praise a product are found to be more effective than two-sided arguments that also mention shortcomings.

FALSE

114) Sales promotion consists of a collection of incentive tools designed to mainly stimulate long-term brand associations of products or services with consumers or the trade.

FALSE

130) The percentage-of-sales method leads to a budget set by market opportunities rather than by the availability of funds.

FALSE

75) Internal marketing requires that everyone in the organization buy into the concepts and goals of marketing and engage in choosing, providing, and communicating customer value.

TRUE Page Ref: 625 Objective: 2 Difficulty: Easy

82) Because the retail trade tends to think of profitability in terms of product categories, some companies are switching to a category management organizational model.

TRUE Page Ref: 628-629 Objective: 2 Difficulty: Easy

85) A customer-management organization deals with individual customers rather than the mass market or market segments.

TRUE Page Ref: 629 Objective: 2 Difficulty: Easy

88) Firms that are viewed as being socially responsible have the added benefit of being able to attract employees.

TRUE Page Ref: 631 Objective: 3 AACSB: Ethical understanding and reasoning abilities Difficulty: Easy

92) Often, the more committed a company is to sustainability and environmental protection, the more dilemmas that can arise.

TRUE Page Ref: 634 Objective: 3 AACSB: Ethical understanding and reasoning abilities Difficulty: Easy

94) Because of insincere firms jumping on the "green" bandwagon, consumers bring a healthy skepticism to environmental claims, but they are also unwilling to sacrifice product performance and quality.

TRUE Page Ref: 635 Objective: 3 AACSB: Ethical understanding and reasoning abilities Difficulty: Easy

105) Annual-plan control involves the use of financial analysis to evaluate the performance of marketing plans.

TRUE Page Ref: 643 Objective: 5 Difficulty: Easy

108) The ratio of promoters to detractors is a customer metric that is used to evaluate the performance of marketing plans.

TRUE Page Ref: 644 Objective: 5 Difficulty: Easy

20) Which of the following activities is a reverse-flow channel of marketing? A) raw materials movement B) product recycling C) materials ordering D) finished goods storage E) customer order placement

b

31) An intensive distribution strategy serves well for ________. A) premium cars B) commercial trucks C) private label products D) industrial equipment E) newspapers

e

he ________ marketing sales system works by recruiting independent businesspeople who act as distributors. A) catalog he ________ marketing sales system works by recruiting independent businesspeople who act as distributors. A) catalog B) multilevel C) direct-response D) corporate E) direct C) direct-response D) corporate E) direct

he ________ marketing sales system works by recruiting independent businesspeople who act as distributors. A) catalog B) multilevel C) direct-response D) corporate E) direct

102) Which of the following is an advantage of using the percentage-of-sales method to determine the marketing communications budget? A) The percentage-of-sales method encourages stability when competing firms spend approximately the same portion of their sales on communications. B) The percentage-of-sales method views sales as the determiner of communications rather than as the result. C) The percentage-of-sales method leads to a budget set by market opportunities rather than the availability of funds. D) The percentage-of-sales method encourages experimentation with countercyclical communication or aggressive spending. E) The percentage-of-sales method encourages building the communication budget by determining what each product and territory deserves.

A

10) Which of the following is the most accurate description of a value network? A) a system of partnerships and alliances that a firm creates to source, augment, and deliver its offerings B) a system of organizations and resources involved in moving a product from supplier to customer C) an arrangement whereby an organization transforms inputs into finished goods D) a network that allows an organization take the finished products to the end-users E) a communication network that allows an organization to transfer information to end-customers

A

106) A group of small sellers takes the initiative and organizes a new business entity to carry on wholesaling and possibly some production. This entity is called a(n) ________. A) retailer cooperative B) franchise organization C) area-based cartel D) sponsored voluntary chain E) alternate selling channel

A

107) Which of the following consumer promotion tools refers to explicit or implicit promises by sellers that the product will perform as specified or that the seller will fix it or refund the customer's money during a specified period? A) product warranties B) coupons C) free trials D) rebates E) patronage awards

A

107) Your firm has experienced a decline in sales over the last three quarters. You have traced the problems to distribution inefficiencies. Which of the following should you track to ensure that the firm's distribution efficiency is maximized? A) average sales per point of sale B) sales from new products C) trial rate D) repurchase rate E) new customer gains

A

35) ________ refers to simple exaggerations in advertisements that are not meant to be believed and that are permitted by law. A) Puffery B) Boosterism C) Astroturfing D) Doublethink E) Subliminal advertising

A

41) Toyota has an advantage over Lexus due to the fact that there are more Toyota dealers, which helps customers save on transportation and search costs in buying and repairing an automobile. Which of the following service outputs relates to this competitive advantage? A) spatial convenience B) service backup C) lot size D) waiting time E) delivery time

A

44) Atburex is a furniture manufacturing company in the United States. The company provides a 60-day credit period and also offers on-site delivery and installation. These special benefits refer to which of the following service outputs? A) service backup B) large product variety C) spatial convenience D) large lot size E) short waiting time

A

47) Which of the following channel alternatives is most suited to handle complex products and transactions? A) sales forces B) the Internet C) dealers D) telemarketers E) direct mails

A

77) Most retailers will put low prices on some items in order to increase traffic to the store. These low-priced products are known as ________. A) loss leaders B) price ceilings C) price skimmers D) price floors E) cold calls

A

78) ________ power can be effective, but its exercise produces resentment and can lead the intermediaries to organize countervailing power. A) Coercive B) Reward C) Legitimate D) Expert E) Referent

A

110) To succeed in the future, marketing must ________. A) become more holistic B) build brands through promotion rather than performance C) be more departmental D) focus on free-spending E) rely more on mass marketing

A

114) ________ analysis looks at specific products, territories, and so forth that failed to produce expected sales. A) Microsales B) Sales variance C) Expense-to-sales D) Full costing E) Financial

A

128) Exxon Consulting works as an agent on behalf of business consumers to collect information on various industrial products. When faced with buying decisions, businesses can approach Exxon to obtain detailed information on the various options available to them. The company earns revenue by selling such information to various customers. Exxon Consulting can be called a(n) ________. A) infomediary B) market maker C) customer community D) third party arbitrator E) informant

A

13) Which of the following is an example of an advertising platform? A) posters and leaflets B) company magazines C) fairs and trade shows D) sales presentations E) continuity programs

A

18) Which of the following is an example of a personal selling communication platform? A) sales presentations B) company blogs C) telemarketing D) TV shopping E) press kits

A

2) Which of the following elements of the marketing communications mix refers to any paid form of nonpersonal presentation and promotion of ideas, goods, or services by an identified sponsor via print, broadcast, network, electronic, and display media? A) advertising B) personal selling C) sales promotion D) direct marketing E) public relations

A

21) There are three types of product-team structures. These are ________. A) vertical, triangular, and horizontal product teams B) vertical, horizontal, and circular product teams C) horizontal, vertical, and rectangular product teams D) horizontal, vertical, and flattened product teams E) vertical, rectangular, and circular product teams

A

25) Which of the following is an example of a zero-level channel? A) A company takes online orders from customers and ships the products to them. B) An organization uses a combination of direct salespeople and sales agencies to increase sales. C) A company sells its products through wholesalers and retailers. D) A company sells its products through chains of supermarkets and other large sellers. E) A large company forms alliances with smaller companies to increase sales coverage.

A

28) A jobber in a three-level marketing channel is a(n) ________. A) small-scale wholesaler B) external broker C) advertising agent D) independent evaluator E) communication channel

A

43) Which of the following terms refers to add-on services such as credit, delivery, installation, and repairs, and is provided by a marketing channel? A) service backup B) product accessories C) external products D) product variety E) service extensions

A

48) Which of the following is a major disadvantage of using the Internet as a marketing channel? A) It is less effective for complex products. B) It lacks convenience and practicality. C) It cannot be used to reach a wide audience. D) It is considered expensive. E) It causes the company to lose direct contact with customers.

A

48) Which of the following is an advantage of using television as an advertising medium? A) high attention and reach B) low absolute cost C) absence of clutter D) long duration of ad exposure E) high audience selectivity

A

49) Which of the following is a disadvantage of using television as an advertising medium? A) high absolute cost B) low audience attention C) lack of reach among audience D) high audience selectivity E) absence of clutter

A

51) One opportunity for designing a cause program so the sponsoring company is not overlooked among other corporate sponsors is to support a(n) ________, or a disease that afflicts fewer than 200,000 people. A) orphan cause B) overlooked calamity C) behavioral campaign D) social marketing plan E) value campaign

A

52) Which of the following is an advantage of using magazines as an advertising medium? A) high-quality reproduction B) short ad purchase lead time C) high efficiency in circulation D) no ad competition in same medium E) low cost of advertising

A

52) Which of the following products is most likely to be sold using an exclusive distribution strategy? A) designer luggage B) cigarettes C) alcoholic beverages D) car fuel E) medicine

A

53) Which of the following covers payment terms and producer guarantees? A) conditions of sale B) pricing policies C) exclusive dealings D) mutual services E) territorial rights

A

55) Cause marketing has a particularly interested audience in civic-minded ________. A) Millennial consumers B) baby boomers C) Gen Xers D) Gen Yers E) shadow boomers

A

57) Which of the following is the correct order of stages that a buyer is assumed to pass through, by the four classic response hierarchy models? A) cognitive stage — affective stage — behavioral stage B) affective stage — cognitive stage — behavioral stage C) behavioral stage — affective stage — cognitive stage D) cognitive stage — behavioral stage — affective stage E) affective stage — behavioral stage — cognitive stage

A

60) Moonburst is a newly launched brand of energy drinks, one among many other recently introduced competing brands. The advertising agency handling Moonburst's account decides that to better promote Moonburst, it has to zero in on an advertising medium that would offer immunity from the clutter of other brands, flexibility to alter its advertising message, and the ability to fit in with the modest advertising budget. Also, the medium has to provide a high repeat exposure of the advertising message to the target audience. The advertising agency would be happy to trade-off audience selectivity and creative possibilities if the medium satisfies the above criteria. Which of the following would be the best option for Moonburst? A) outdoor media B) radio C) magazines D) television E) Yellow Pages

A

32) ________ consist of the sum of the fixed and variable costs for any given level of production. A) Total costs B) Average costs C) Opportunity costs D) Learning costs E) Target costs

A Page Ref: 393 Objective: 2 Difficulty: Easy

45) A retailer who holds on to a(n) ________ policy charges a constant low price with little or no price promotions and special sales. A) everyday low pricing B) high-low pricing C) low cost D) going-rate pricing E) auction-type pricing

A Page Ref: 400 Objective: 2 Difficulty: Easy

68) Madame Tussaud's wax museum is a popular tourist attraction in London. The museum charges higher entry rates for tourists compared to locals. This form of price discrimination is known as ________. A) customer-segment pricing B) image pricing C) location pricing D) special customer pricing E) special event pricing

A Page Ref: 406 Objective: 3 AACSB: Analytic skills Difficulty: Moderate

60) During ________, the company's technical people observe how customers use the product, a practice that often exposes unanticipated problems of safety and servicing and alerts the company to customer training and servicing requirements. A) beta testing B) research testing C) sales-wave testing D) alpha testing E) simulated testing

A Page Ref: 589 Objective: 4 Difficulty: Easy

45) ________ is the process that turns marketing plans into action assignments and ensures that such assignments are executed in a manner that accomplishes the plan's stated objectives. A) Marketing implementation B) Marketing research C) Marketing analysis D) Brand management E) Product management

A Page Ref: 642 Objective: 5 Difficulty: Easy

46) ________ is the process by which firms assess the effects of their marketing activities and programs and make necessary changes and adjustments. A) Marketing control B) Marketing implementation C) Test marketing D) Market watch E) Market analysis

A Page Ref: 643 Objective: 5 Difficulty: Easy

Which of the following is a prepurchase service offered by retailers? A) accepting orders over the telephone B) shipping the product C) delivery to the customer's doorstep D) general information E) interior decoration of the retail outlet

A) accepting orders over the telephone

31) Which of the following statements correctly reflects a characteristic of public relations as a marketing communications tool? A) Public relations can reach prospects who prefer to avoid mass media and targeted promotions. B) They incorporate some concession, inducement, or contribution that gives value to the consumer. C) Given their live, real-time quality, public relations tools are more actively engaging for consumers. D) Public relations communications can be prepared to appeal to the addressed individual. E) Public relations tools create an immediate and interactive episode between two or more persons.

A

88) Which of the following ads depict an informational appeal? A) Thompson Water Seal can withstand intense rain, snow, and heat. B) The California Milk Processor Board ran the successful "Got Milk?" ad to boost declining sales. C) VW advertised to active, youthful people with its famed "Drivers Wanted" campaign. D) Pringles advertised "Once You Pop, the Fun Don't Stop" for years. E) KFC has marketed its fast-food products under the slogan, "Finger Lickin' Good."

A

1) Appointing teams to manage customer-value-building processes and break down walls between departments is part of which of the following shifts in business practices? A) reengineering B) outsourcing C) benchmarking D) supplier partnering E) customer partnering

A

1) ________ are sets of interdependent organizations participating in the process of making a product or service available for use or consumption. A) Marketing channels B) Interstitials C) Communication channels D) Sales territories E) Marketing terrains

A

143) A manufacturer wants to achieve rapid market penetration through a low-price policy. However, the manufacturer's dealers prefer to work with high margins and pursue short-run profitability. The major reason for this conflict is ________. A) goal incompatibility B) unclear roles C) ambiguous rights D) differences in perception E) dependence on the manufacturer

A

145) General Motors' executives work for a short time in some dealerships, and some dealership owners work in GM's dealer policy department. This strategy helps the company avoid conflicts with its dealers. This is an example of the ________ strategy. A) employee exchange B) dual compensation C) joint membership D) co-optation E) diplomacy

A

29) Door-to-door sales, home parties, mail order, telemarketing, TV selling, Internet selling, and manufacturer-owned stores are examples of ________. A) zero-level channels B) jobbers C) wholesalers D) manufacturer's representatives E) consumers

A

3) Tracy works for a cellular phone company that offers trade-in allowances toward the upgrade of new phones, which is an example of ________. A) a sales promotion B) interactive marketing C) word-of-mouth marketing D) personal selling E) events and experiences

A

5) Which of the following business practices involves reducing the number of organizational levels to get closer to the customer? A) flattening B) globalizing C) decentralization D) merging E) justifying

A

58) Which of the following problems is most likely when a company chooses to use a sales agency instead of company salespeople? A) The company will find it difficult to control the sales process. B) They do not take title to goods or negotiate purchases or sales. C) The value-added per sale is the lowest for sales agencies. D) Agencies will pay less attention to customers who buy the most or in large volumes. E) Sales agencies are often difficult to access due to strict contractual obligations.

A

64) Which of the following represents the objective of a social marketing campaign aimed at changing people's behavior? A) Motivate people with obesity to eat healthy and exercise more often. B) Change public attitudes and stereotypes associated with people who are obese. C) Explain the different causes of obesity and how it can be prevented. D) Encourage people to participate in a walkathon aimed at promoting awareness about obesity. E) Attract people with obesity to sign up for a one-time free medical check up.

A

10) Which of the following types of retailing generally entails the highest costs? A) full-service B) self-service C) limited-selection D) limited service E) self-selection

A Diff: 1 LO: 18.1: What major types of marketing intermediaries occupy this sector? AACSB: Analytical thinking

17) A factory outlet is an example of a(n) ________ retailer. A) off-price B) specialty C) discount D) department E) catalog

A Diff: 1 LO: 18.1: What major types of marketing intermediaries occupy this sector? AACSB: Analytical thinking

61) Which of the following is an example of a category killer? A) PETCO B) Walmart C) Kohl's D) The Limited E) Tesco

A Diff: 2 LO: 18.2: What major changes are occurring in the modern retail marketing environment with respect to competitive market structure and technology? AACSB: Application of knowledge

________ refers to buying large carload lots and dividing them into smaller units before shipping them out to consumers. A) Bulk breaking B) Containerization C) Wholesaling D) Warehousing E) Broking

A) Bulk breaking

One of the possible objectives of marketing communications is helping consumers evaluate a brand's perceived ability to meet a currently relevant need. Which of the following is a negatively oriented relevant brand need? A) normal depletion B) sensory gratification C) social approval D) intellectual stimulation E) value enhancement

A) Normal depletion

Which of the following marketing communications tools is most effective at the later stages of the buying process? A) personal selling B) public relations C) advertising D) sales promotions E) direct marketing

A) Personal Selling

Which of the following is an example of an advertising platform? A) posters and leaflets B) company magazines C) fairs and trade shows D) sales presentations E) continuity programs

A) Posters and Leaflets

2) ________ price refers to what the consumers feel the product should cost. A) Fair B) Typical C) Usual discounted D) List E) Maximum retail

A Page Ref: 387 Objective: 1 Difficulty: Easy

22) When customers fall into different user groups with distinct buying preferences and practices, a ________ is desirable. A) market-management organization B) product-management organization C) brand-management organization D) geographic organization E) functional organization

A Page Ref: 629 Objective: 2 Difficulty: Easy

104) Gabrielle is the chief marketing officer of Boyd Pharmaceuticals. She is meeting with Trent, the chief financial officer to decide on the company's marketing communications budget. They pore over the sales reports and other financial records and determine the amount of resources they can spare for marketing communications, after resources have been allocated to other functions, such as R&D, logistics, etc. What method did Gabrielle and Trent use to arrive at the marketing communications budget? A) objective-and-task method B) affordable method C) competitive-parity method D) activity-based method E) percentage-of-sales method

B

118) Agricultural assemblers, petroleum bulk plants and terminals, and auction companies are examples of ________. A) full-service wholesalers B) specialized wholesalers C) limited-service wholesalers D) merchant wholesalers E) brokers

B

135) A stock order point of 10 means ordering the product ________. A) every 10 days B) when stock falls to 10 units C) every 10 units D) when stock falls to 9 units E) in batches of 10 items

B

146) The most effective communications mix tool at later stages of the buying process is ________ because it is particularly effective at building buyer preference, conviction, and action. A) sales promotions B) personal selling C) advertising D) direct and interactive marketing E) events

B

147) Which of the following is a medium of visual identity for a company? A) annual reports B) dress codes C) press releases D) seminars E) trade shows

B

15) E&OE produces and markets its own brand of skincare products using herbal remedies and natural ingredients through standalone stores as well as an online portal. E&OE is a(n) ________ retailer. A) off-price B) specialty C) discount D) department E) extreme value

B

151) If a producer wants to achieve rapid market penetration through a low-price policy, while a dealer wants to work with high margins to pursue short-run profitability, the source of the channel conflict is ________. A) unclear roles and rights B) goal incompatibility C) differences in perception D) intermediaries' dependence on the manufacturer E) strategic justification

B

16) A company selling in a national market often organizes its sales force along ________. A) functional groups B) geographic lines C) product teams D) brand groups E) product categories

B

19) Companies that sell products door-to-door or at home sales parties are engaging in ________. A) franchising B) network marketing C) direct-response marketing D) corporate selling E) direct marketing

B

19) The starting point in planning marketing communications is a ________ that profiles all interactions customers in the target market may have with the company and all its products and services. A) marketing budget B) communications audit C) market research program D) publicity campaign E) product launch

B

34) Which of the following wholesaler functions reduces inventory costs and risks to suppliers and customers? A) selling and promoting B) warehousing C) transportation D) market information E) assortment building

B

39) Which of the following tools or combinations of tools is most influential at the comprehension stage of buyer readiness? A) sales promotion and advertising B) advertising and personal selling C) publicity and personal selling D) reminder advertising and publicity E) sales promotion and personal selling

B

54) Cause-related marketing is part of ________. A) sustainability B) corporate societal marketing C) social media D) global marketing E) community marketing

B

55) Electrobar, a European manufacturer of industrial kitchenware, sells to industrial canteens, restaurants, hotels, and so forth. The company provides a one-year warranty on all products and also allows customers to pay in installments — they pay 50 percent on delivery and the rest as equal installments. This refers to which element in the "trade-relations mix"? A) price policy B) conditions of sale C) distributors' territorial rights D) exclusive dealings E) mutual services and responsibilities

B

57) Which of the following is a disadvantage of using the Internet as an advertising medium? A) limited audience selectivity B) increasing clutter C) lack of interactive possibilities D) relatively high costs involved E) fleeting ad exposure time

B

6) When is a pull strategy appropriate? A) when there is low brand loyalty B) when consumers are able to perceive differences between brands C) when brand choice is made in the store D) when it is a low involvement purchase E) when the product is an impulse item

B

1) When consumers examine products, they often compare an observed price to an internal price they remember. This is known as a(n) ________ price. A) markup B) reference C) market-skimming D) accumulated E) target

B Page Ref: 387 Objective: 1 Difficulty: Easy

15) A company that is looking to maximize its market share would do well to follow ________ pricing. A) markup B) market-penetration C) market-skimming D) survival E) target-return

B Page Ref: 389 Objective: 2 Difficulty: Easy

28) Which of the following is true regarding price elasticity? A) The higher the elasticity, the lesser is the volume growth resulting from a 1 percent price reduction. B) Within the price indifference band, price changes have little or no effect on demand. C) If demand is elastic, sellers will consider increasing the price. D) Price elasticity does not depend on magnitude and direction of the contemplated price change. E) When demand is inelastic, sellers should lower prices in order to increase total revenue.

B Page Ref: 392 Objective: 2 Difficulty: Moderate

52) ________ let would-be suppliers submit only one bid; they cannot know the other bids. A) Descending bid auctions B) Sealed-bid auctions C) English auctions D) Dutch auctions E) Reverse auctions

B Page Ref: 402 Objective: 2 Difficulty: Easy

78) When a company requires the customers to pay today's price and all or part of any inflation increase that takes place before delivery, it is known as ________. A) special-customer pricing B) an escalator clause C) delayed quotation pricing D) unbundling E) time pricing

B Page Ref: 408 Objective: 5 Difficulty: Easy

69) ________ advocates argue that all costs must ultimately be imputed in order to determine true profitability. A) Direct-cost approach B) Full-cost approach C) Traceable-cost approach D) Activity based costing approach E) Fixed cost approach

B Page Ref: 659 Objective: APP Difficulty: Easy

The starting point in planning marketing communications is a ________ that profiles all interactions customers in the target market may have with the company and all its products and services. A) marketing budget B) communications audit C) market research program D) publicity campaign E) product launch

B) Communications Audit

One of the possible objectives of marketing communications is helping consumers evaluate a brand's perceived ability to meet a currently relevant need. Which of the following is a positively oriented relevant brand need? A) problem removal B) social approval C) normal depletion D) problem avoidance E) incomplete satisfaction

B) Social Approval

________ includes all the activities in selling goods or services to those who buy for resale or business use. A) Retailing B) Wholesaling C) Procurement D) Promoting E) Warehousing

B) Wholesaling

Which of the following is considered a wholesaler? A) retailers B) brokers C) producers D) manufacturers E) farmers

B) brokers

Companies that sell products door-to-door or at home sales parties are engaging in ________. A) franchising B) network marketing C) direct-response marketing D) corporate selling E) direct marketing

B) network marketing

Which of the following wholesaler functions reduces inventory costs and risks to suppliers and customers? A) selling and promoting B) warehousing C) transportation D) market information E) assortment building

B) warehousing

147) Pepsi Co. sold its cola syrup to Russia and agreed to buy Russian vodka at a certain rate for sale in the United States for the next 5 years. What kind of a countertrade did both the parties indulge in?

Both the parties indulged in an offset. In such countertrade, the seller receives full payment in cash but agrees to spend a substantial amount of the money in that country within a stated time period. Page Ref: 404 Objective: 3 Difficulty: Moderate

Which of the following is true for franchisees? A) The franchisee is paid by the franchisor to be part of the franchise system. B) The franchisee licenses a trademark to the franchisor. C) The franchisee must change its operations to suit those of the franchisor. D) The franchisee collects royalty payments from the franchisor. E) The franchisee owns the trade or service mark.

C) The franchisee must change its operations to suit those of the franchisor.

36) ________ is finding the most cost-effective media to deliver the desired number and type of exposures to the target audience. A) Media scheduling B) Content analysis C) Media selection D) Communication design E) Copy testing

C

42) Under which of the following conditions is the reach of media the most important factor in media selection? A) when introducing frequently purchased brands B) when going into a defined target market C) when launching extensions of well-known brands D) when there are strong competitors to a brand E) when there is high consumer resistance to the product

C

47) Which of the following is a disadvantage of using newspapers as an advertising medium? A) lack of flexibility B) high costs of advertisement space C) poor reproduction quality D) low believability E) absence of local market coverage

C

58) LCH is a leading electronics company that produces and markets its own brand of desktop and laptop computers for both individual consumers and businesses. Which of the following sequences of consumer responses is relevant as a marketing communications model for LCH's products? A) learn-do-feel B) feel-learn-do C) do-feel-learn D) feel-do-learn E) do-learn-feel

C

61) Which of the following represents the objective of a cognitive social marketing campaign? A) Motivate people with obesity to eat healthy and exercise more often. B) Change public attitudes and stereotypes associated with people who are obese. C) Explain the different causes of obesity and how it can be prevented. D) Encourage people to participate in a walkathon aimed at promoting awareness about obesity. E) Help people with obesity to implement lifestyle changes.

C

63) According to the hierarchy-of-effects model, which of the following corresponds to the behavioral stage that a buyer passes through? A) awareness B) knowledge C) purchase D) preference E) conviction

C

9) As a marketing practice, monitoring involves ________. A) designing the organization and setting up processes to respond quickly to changes in the environment B) becoming more accountable by measuring, analyzing, and documenting the effects of marketing actions C) tracking what is said online and elsewhere and studying customers, competitors, and others to improve business practices D) determining the most profitable businesses and customers and expending greater organizational resources to capitalize on them E) factoring the interests of customers, employees, shareholders, and other stakeholders into the activities of the enterprise

C

9) Which of the following elements of the marketing communication mix involves use of mail, telephone, fax, e-mail, or Internet to communicate with or solicit response or dialogue from specific customers and prospects? A) advertising B) personal selling C) direct marketing D) public relations E) sales promotion

C

92) Which of the following elements of the marketing communications mix consists of a collection of incentive tools, mostly short term, designed to stimulate quicker or greater purchase of particular products or services by consumers, or the trade? A) advertising B) public relations C) sales promotion D) events and experiences E) personal selling

C

Containerization consists of putting the goods in boxes or trailers that are easy to transfer between two transportation modes.​ ________ describes the use of rail and trucks. A. Fishyback B. Airship C. Piggyback D. Airtruck E. Trainship

C

Of the​ following, which affects product​ pricing, on-time delivery​ performance, and the condition of the goods when they​ arrive? A. Inventory B. Order processing C. Transportation D. Lean manufacturing E. Warehousing

C

80) Which of the following is a postpurchase service offered by retailers? A) accepting orders over the telephone B) advertising and window displays C) delivery to the customer's doorstep D) general information E) interior decoration of the retail outlet

C Diff: 2 LO: 18.3: What marketing decisions do marketing intermediaries make? AACSB: Reflective thinking

105) In addition to its store brands and nationally well-known brands of detergents, Reynold's also carries much cheaper varieties of detergents that are not advertised and have little-known names. They are often manufactured from lower-quality ingredients and save on packaging and advertising costs. These are known as ________. A) common carriers B) shills C) generics D) private labels E) marques

C Diff: 2 LO: 18.4: What does the future hold for private label brands? AACSB: Analytical thinking

42) Which of the following should be done to increase the likelihood that social marketing programs will be successful? A) Choose target markets that are the least ready to respond. B) Promote multiple, doable behaviors in clear, simple terms. C) Make it easy to adopt the behavior. D) Adopt a soft sell approach, as opposed to attention grabbing messages. E) Explain the benefits in an exaggerated manner.

C Page Ref: 641 Objective: 3 AACSB: Ethical understanding and reasoning abilities Difficulty: Easy

66) Which of the following is true about market share? A) Outside forces affect all companies in the same way. B) A company's performance should be judged against the average performance of all companies. C) A decline in market share does not necessarily mean the company is performing worse than other companies. D) A decline in market share cannot be deliberately engineered. E) All shifts in market share have marketing significance.

C Page Ref: 654 Objective: APP Difficulty: Moderate

128) The manufacturer launches a brand building advertising campaign. The campaign does not promote any one specific car but is aimed at promoting the company as a whole. Which type of cost does this fall under? A) direct costs B) material costs C) nontraceable costs D) traceable costs E) labor costs

C

43) Under which of the following conditions is the frequency the most important factor in media selection? A) when introducing flanker brands B) when launching infrequently purchased brands C) when going into undefined target markets D) when there is high consumer resistance to the product E) when there is modest competition to the brand in the market

D

43) Which of the following marketing communications tools is most influential at the reordering stage of buyer readiness? A) events and experiences B) publicity C) direct marketing D) sales promotion E) interactive marketing

D

46) Which of the following allows a firm to maintain control over service level and obtain more dedicated and knowledgeable selling? A) selective distribution B) intensive distribution C) push strategy D) exclusive distribution E) pull strategy

D

5) Spike Inc. is a sportswear manufacturer that recently launched its new line of customizable running shoes. The shoes come with a digital component that allows them to adapt to the runner's biomechanics. To promote this new product, Spike launches an advertising campaign and entices a famous athlete to endorse the product. This is an example of a ________. A) trade promotion B) reverse flow C) push strategy D) pull strategy E) backward flow

D

142) When Goodyear began selling its popular tire brands through Sears, Walmart, and Discount Tire, independent dealers that sold the same tires at higher prices were angry. This is an example of a(n) ________ conflict. A) horizontal B) vertical C) intermediate D) multichannel E) parallel

D

146) ________ refers to the task of securing editorial space — as opposed to paid space — in print and broadcast media to promote or "hype" a product, service, idea, place, person, or organization. A) Advertising B) Media planning C) Communication design D) Publicity E) Copy testing

D

24) Producers often shift some functions to intermediaries. Which of the following is the most significant benefit of doing this? A) It increases customer loyalty. B) It provides the producer with greater control over operations. C) It reduces the amount of direct customer interaction. D) It lowers the producer's costs and prices. E) It ensures greater information security.

D

3) Which of the following is an example of retailing? A) Dylan's sends catalogs to retail, industrial, and institutional customers. B) SEZ U Inc. sells a limited line of fast-moving goods to small retailers for cash. C) BEL Inc. sells FMCG goods to merchant wholesalers and distributors. D) Praxis International sells products to consumers directly through the Internet. E) Hub Styles procures its raw materials directly from farmers in the region.

D

3) ________ involves studying "best practice companies" to improve performance. A) Empowering B) Globalizing C) Flattening D) Benchmarking E) Focusing

D

60) A social marketing program which aims to alter ideas about abortion is an example of a(n) ________ campaign. A) cognitive B) active C) behavioral D) value E) normative

D

61) When planning communications for a detergent brand, which of the following sequences of buyer responses should the marketer choose on which to base the communications model? A) feel-do-learn B) do-feel-learn C) feel-learn-do D) learn-do-feel E) learn-feel-do

D

78) American businessman King Gillette pioneered the sales model in which razor handles were given away for free or sold at a loss, but sales of disposable razor blades were very profitable. This is known as the ________ model. A) two-tiered pricing B) predatory pricing C) cross selling D) loss leading E) product churning

D

81) Which of the following is an ancillary service offered by retailers? A) accepting orders over the telephone B) advertising and window displays C) delivery to the customer's doorstep D) general information E) alterations and tailoring

D

87) Which of the following ads depict a transformational appeal? A) Thompson Water Seal can withstand intense rain, snow, and heat. B) DIRECTV offers better HD options than cable or other satellite operators. C) NBA phenomenon LeBron James pitching Nike, Sprite, and McDonald's. D) Pringles advertised "Once You Pop, the Fun Don't Stop" for years. E) Excedrin stops the toughest headache pain.

D

93) ________ software provides a set of Web-based applications that automate and integrate project management, campaign management, budget management, asset management, brand management, customer relationship management, and knowledge management. A) Marketing dashboard B) Enterprise resource planning C) Supply chain management D) Marketing resource management E) Enterprise campaign management

D

133) ________ was originally pioneered by Japanese firms such as Toyota to produce goods with minimal waste of time, materials, and money. A) Electronic funds transfer (EFT) B) Market logistics C) Electronic data interchange (EDI) D) Lean manufacturing E) Supply chain management

D Diff: 2 LO: 18.6: What are some important issues in logistics? AACSB: Reflective thinking

2) Which of the following is true for retailing? A) Manufacturers are not considered to be retailers as they are engaged in producing the product. B) Vending machines are considered to be retailing only if they are located within stores. C) Retailing deals only with goods; it does not include services. D) Selling from a consumer's home is direct selling, but not retailing. E) Wholesalers are only considered to be retailers if they are selling to final consumers.

E

24) Telemarketing is a type of ________. A) direct selling B) network marketing C) multilevel selling D) close-range marketing E) direct marketing

E

25) Electronic shopping is a type of ________. A) direct selling B) network marketing C) multilevel selling D) corporate selling E) direct marketing

E

32) Aisha is a marketing manager with Injoos, a company that manufactures packaged fruit juices. Knowing that several other companies exist in the market that offer similar products, Aisha decides to build a customer base from among those who prefer to avoid mass media and other targeted promotions. Which of the following marketing communications tools would be her best option to build a favorable impression among the prospective customers? A) advertising B) personal selling C) sales promotions D) direct marketing E) public relations

E

50) Which of the following is an advantage of using radio as an advertising medium? A) higher attention than television B) standardized rate structures C) long duration of ad exposure D) high quality reproduction E) high geographic selectivity

E

51) An intensive distribution strategy serves well for ________. A) premium cars B) commercial trucks C) private label products D) industrial equipment E) newspapers

E

64) According to the hierarchy-of-effects model, which of the following corresponds to the affective stage that a buyer passes through? A) attention B) exposure C) reception D) adoption E) conviction

E

122) Sales promotion tools that typically are not brand building include price-off packs, contests and sweepstakes, consumer refund offers, and trade allowances.

TRUE

62) "Pop-up" stores let retailers promote brands to seasonal shoppers for a limited time.

TRUE

65) Supercenters retail grocery items as well as a huge selection of nonfood merchandise.

TRUE

82) Raising the level of socially responsible marketing calls for a three-pronged attack that relies on proper legal, ethical, and social responsibility behavior.

TRUE

87) Channel power is the ability to alter channel members' behavior so that they take actions they would not have taken otherwise.

TRUE

88) A manufacturer is using reward power when it offers intermediaries an extra benefit for performing specific acts or functions.

TRUE

89) Companies must adopt and disseminate a written code of ethics, build a company tradition of ethical behavior, and hold their people fully responsible for observing ethics and legal guidelines if they wish to demonstrate ethical behavior.

TRUE Page Ref: 633 Objective: 3 AACSB: Ethical understanding and reasoning abilities Difficulty: Easy

112) To succeed in the future, marketing must be more holistic and less departmental.

TRUE Page Ref: 645 Objective: 4 Difficulty: Easy

40) Eureka Forbes, an Asian consumer appliances company, sells its vacuum cleaners through door-to door sales. This allows the company to obtain a high conversion ratio. Comment on the length of the channel in the case of Eureka Forbes' vacuum cleaners.

The company is using a zero-level channel, i.e. the manufacturer sells directly to the final customer. This is the shortest channel.

34) A group of college graduates decide to start a business. Though they are knowledgeable in various business domains, they are unable to arrive at a valuable business idea. They decide to search for ideas in a structured manner. They meet together and start discussing everyone's ideas. Each idea is recorded and then the thoughts that come up in relation to the ideas are written down and discussed. This process helps them to finalize a business plan. What technique is used here? A) morphological analysis B) forced relationship analysis C) reverse assumption analysis D) attribute listing E) mind mapping

E Page Ref: 580 Objective: 3 AACSB: Analytic skills Difficulty: Moderate

In supermarkets and other retail outlets, RFID is used to ________. A) change prices instantaneously B) check for spoilage or damage to goods C) advertise special offers and discounts D) run continual promotional messages E) monitor inventory and track goods

E) monitor inventory and track goods

The elapsed time between an order's receipt, delivery, and payment is called the ________ cycle. A) variable-costs-to-payment B) product-to-payment C) inventory-to-sale D) order-to-inventory E) order-to-payment

E) order-to-payment

The owner of supermarket chain Reynold's has realized that customers want a wider variety of goods than is currently available. However, Reynold's cannot afford the costs of storing excess inventory. Additionally, the owner is not willing to take the risk that the new products will remain unsold. Which of the following types of wholesalers can help Reynold's meet customer demand while minimizing costs? A) producers' cooperatives B) cash and carry wholesalers C) truck wholesalers D) drop shippers E) rack jobbers

E) rack jobbers

141) The first step in conducting a marketing profitability analysis involves assigning functional expenses to marketing entities.

FALSE

25) In this new communication environment, advertising is often the single most important element of a marketing communications program for sales and building brand and customer equity.

FALSE

26) Personal selling refers to person-to-person oral, written, or electronic communications that relate to the merits or experiences of purchasing or using products or services.

FALSE

33) The product-management organization replaces the functional organization in the firm.

FALSE

34) A zero-level marketing channel typically uses a wholesaler and a retailer.

FALSE

34) An advantage of the product- and brand-management system is that product and brand managers focus the company on building market share rather than customer relationships.

FALSE

66) Growth in the retail market is centered firmly in the middle market, leaving luxury retailers and discounting specialists struggling.

FALSE

66) The less the sender's field of experience overlaps that of the receiver, the more effective the message is likely to be.

FALSE

67) All the response hierarchy models assume that a buyer passes through affective, cognitive, and behavioral stages, in that order.

FALSE

76) Brands in highly differentiated product classes require heavy advertising to establish a unique image.

FALSE

76) Cash donations are the most productive contribution that businesses can make to a nonprofit or community group.

FALSE

79) Social marketing programs designed to motivate people to donate blood or attract people for mass immunization are examples of cognitive campaigns.

FALSE

93) Accepting telephone and mail orders is an example of the ancillary services that a retailer offers.

FALSE

81) Price is one of the two elements of the marketing mix that produces revenue.

FALSE Page Ref: 383 Objective: 1 Difficulty: Easy

82) Traditionally, price was never a major determinant of buyer choice.

FALSE Page Ref: 384 Objective: 1 Difficulty: Easy

86) Skunkworks are formal workplaces where intrapreneurial teams attempt to develop new products.

FALSE Page Ref: 574 Objective: 2 Difficulty: Easy

115) At the interest stage of consumer-adoption process, the consumer becomes aware of the innovation but lacks information about it.

FALSE Page Ref: 591 Objective: 5 Difficulty: Easy

77) A functional organization allows for adequate planning as the number of products and markets of a firm increases.

FALSE Page Ref: 627 Objective: 2 Difficulty: Easy

11) Which of the following is the most accurate description of a value network? A) A system of partnerships and alliances that a firm creates to source, augment, and deliver its offerings. B) A system of organizations and resources involved in moving a product from supplier to customer. C) An arrangement whereby an organization transforms inputs into finished goods. D) A network that allows an organization take the finished products to the end-users. E) A communication network that allows an organization to transfer information to end-customers.

a

17) Which of the following entities is present in a zero-level marketing channel? A) consumers B) retailers C) brokers D) jobbers E) wholesalers

a

2) Which of the following entities in the marketing channel is a merchant? A) wholesalers B) brokers C) sales agents D) warehouses E) advertising agencies

a

21) Toyota has an advantage over Lexus due to the fact that there are more Toyota dealers, which helps customers save on transportation and search costs in buying and repairing an automobile. Which of the following service outputs relates to this competitive advantage? A) spatial convenience B) service backup C) lot size D) waiting time E) delivery time

a

27) Which of the following channel alternatives is most suited to handle complex products and transactions? A) sales forces B) the Internet C) dealers D) telemarketers E) direct mails

a

107) In a retailer cooperative, ________. A) profits are equally divided among members B) members plan their advertising jointly C) nonmembers cannot buy through the co-op D) members rely on distribution programming E) members standardize their selling practices

B

113) Which of the following is considered a wholesaler? A) retailers B) brokers C) producers D) manufacturers E) farmers

B

21) Which of the following is an example of direct marketing? A) E&OE sells its herbal skincare products exclusively through its standalone stores. B) TCJ is a telemarketing firm that sells products from a number of different suppliers. C) Jayne's sells most of its products to customers through home sales parties. D) J3 is a storeless retailer that organizes the retail activity of the employees of four firms. E) Reynold's tries to minimize its staff costs by installing vending machines in its stores.

B

27) Transforming into a true market-driven company requires organizing around ________. A) sales B) customer segments C) products D) functions E) brands

B

30) Which of the following activities is a reverse-flow channel of marketing? A) raw materials movement B) product recycling C) materials ordering D) finished goods storage E) customer order placement

B

102) Which of the following products is most likely to undergo alpha and beta testing? A) food products B) industrial goods C) consumer products D) commodities E) FMCG products

B Diff: 2 LO: 15.6: What is the best way to manage concept and strategy development? AACSB: Application of knowledge

86) With ________, respondents see different hypothetical offers formed by combining varying levels of the attributes, then rank the various offers. A) gap level analysis B) conjoint analysis C) perceptual mapping D) concept testing E) morphological analysis

B Diff: 2 LO: 15.6: What is the best way to manage concept and strategy development? AACSB: Reflective thinking

21) Which of the following is an example of direct marketing? A) E&OE sells its herbal skincare products exclusively through its standalone stores. B) TCJ is a telemarketing firm that sells products from a number of different suppliers. C) Jayne's sells most of its products to customers through home sales parties. D) J3 is a storeless retailer that organizes the retail activity of the employees of four firms. E) Reynold's tries to minimize its staff costs by installing vending machines in its stores.

B Diff: 3 LO: 18.1: What major types of marketing intermediaries occupy this sector? AACSB: Analytical thinking; Application of knowledge

8) Many consumers are willing to pay $100 for a perfume that contains $10 worth of scent because the perfume is from a well-known brand. What kind of a pricing is the company depending on? A) going-rate pricing B) image pricing C) market-skimming pricing D) target pricing E) markup pricing

B Page Ref: 388 Objective: 1 Difficulty: Easy

56) ROC Engineering, a Chinese shipbuilding company, agrees to build a fleet of submarines for the Sri Lankan navy, for which it will be paid in the local Sri Lankan currency. As per the agreement, ROC must also spend a substantial amount of the money it generates through this deal within the country. In accordance with the contract, ROC buys Sri Lankan tea at a reduced rate. This is an example of which of the following forms of countertrade? A) descending bid B) offset C) barter D) compensation deal E) buyback arrangement

B Page Ref: 404 Objective: 3 AACSB: Analytic skills Difficulty: Easy

54) A Japanese firm is ready to sell its recent technological innovation to the U.S. government. But it has asked for 80 percent in cash and the rest in mica. The Japanese firm is looking to enter into a(n) ________ with the U.S. government. A) functional discount B) compensation deal C) buyback arrangement D) offset agreement E) barter deal

B Page Ref: 404 Objective: 3 AACSB: Analytic skills Difficulty: Moderate

20) The ________ uses observational, anthropological, and ethnographic methods or consumer self-reports to map consumer needs, wants, and even beyond. A) business analysis B) demand landscape C) opportunity space D) strategic blueprint E) internal design analysis

B Page Ref: 576 Objective: 3 Difficulty: Easy

33) Increasingly, new-product ideas arise from ________ that combines two product concepts or ideas to create a new offering. A) reverse assumption analysis B) lateral marketing C) attribute listing D) forced relationships E) morphological analysis

B Page Ref: 580 Objective: 3 Difficulty: Easy

33) Jake wants to open a Subway franchise in his small town. To do this, he must pay the company a ________ fee. A) slotting B) title C) royalty D) merchandising E) residual

C

105) In addition to its store brands and nationally well-known brands of detergents, Reynold's also carries much cheaper varieties of detergents that are not advertised and have little-known names. They are often manufactured from lower-quality ingredients and save on packaging and advertising costs. These are known as ________. A) common carriers B) shills C) generics D) private labels E) marques

C

55) In an attempt to improve the product, a company that manufactures screwdrivers discusses the modification of each attribute, such as replacing the wooden handle with plastic, providing torque power, adding different screw heads, and so on. This creativity technique is called ________. A) attribute listing B) mind mapping C) morphological analysis D) lateral analysis E) reverse analysis

C Diff: 2 LO: 15.4: What are the main stages in developing new products and services? AACSB: Application of knowledge

________ is an element of the marketing communications mix that involves people-to-people oral, written, or electronic communications that relate to the merits or experiences of purchasing or using products or services. A) Personal selling B) Sales promotion C) Word-of-mouth marketing D) Public relations E) Advertising

C) Word-of-mouth marketing

37) Which marketing channel is associated with the highest value added per sale? A) retail stores B) sales force C) distributors D) Internet E) telemarketing

b

The degree to which the innovation can be tried on a limited basis is its ________.

Divisibility

_______ is additional income to a company as a result of a new product.

Dragalong Income

112) Which of the following is a best practice in business and marketing? A) end-product orientation B) reacting to competitors C) vertical integration D) stockholder driven E) teamwork

E

When a manufacturer wants to earn a specific percent on sales, what should be determined to calculate the sale price?

Markup Price

132) Corporate philanthropy can pose problems even when done with the best intentions. Explain.

Philanthropic efforts by companies can be overlooked—even resented—if the company is seen as exploitive or fails to live up to a "good guys" image. Page Ref: 634 Objective: 3 AACSB: Ethical understanding and reasoning abilities Difficulty: Moderate

When a museum charges a lower admission fee for students and senior citizens, it is engaging in _______ pricing

Customer-Segment

103) The purpose of strategic control is to ________. A) examine whether the planned results are being achieved B) examine where the company is making and losing money C) evaluate and improve the spending efficiency and impact of marketing expenditures D) examine whether the company is pursuing its best opportunities with respect to markets, products, and channels E) understand the efficiency of the sales force, advertising, sales promotion, and distribution

D

103) Which of the following is true for distributor brands? A) They sell at higher volumes than national brands and are also known as generics. B) They are usually sold at higher prices than national brands because production costs are higher. C) They are always of better quality than national brands as production is strictly monitored. D) Distributor brands can be sold at lower prices yet generate a higher profit margin because of their lower cost structure. E) Advertising and sales promotion costs for distributor brands are much higher than those for national brands.

D

133) ________ was originally pioneered by Japanese firms such as Toyota to produce goods with minimal waste of time, materials, and money. A) Electronic funds transfer (EFT) B) Market logistics C) Electronic data interchange (EDI) D) Lean manufacturing E) Supply chain management

D

51) Which of the following is a disadvantage of using direct mail as an advertising medium? A) low audience selectivity B) lack of flexibility C) high competition within same medium D) relatively high cost E) lack of personalization of advertising message

D

143) Which of the following describes the public relations function of lobbying? A) sponsoring efforts to publicize specific products B) advising management about public issues, and company positions and image during good times and bad C) presenting news and information about the organization in the most positive light D) dealing with legislators and government officials to promote or defeat legislation and regulation E) promoting understanding of the organization through internal and external communications

D

6) Which of the following business practices focuses specifically on designing the organization and setting up processes to respond quickly to changes in the environment? A) benchmarking B) outsourcing C) focusing D) accelerating E) globalizing

D

98) Top and middle management are primarily responsible for ________ control. A) annual-plan B) efficiency C) profitability D) technological E) innovation

A

56) Which of the following factors found in the macromodel of the communications process refers to random and competing messages that may interfere with the intended communication? A) negative feedback B) noise C) attenuation D) phase lag E) selective distortion

B

How does Proctor and Gamble's "Share Your Thoughts" section of its Web site draw new ideas from its customer?

By allowing users to offer advice and feedback

14) Which of the following is an example of an events and experiences platform? A) fairs and trade shows B) continuity programs C) factory tours D) sales presentations E) community relations

C

75) Mass merchandisers and discount stores typically fall into the ________ group with respect to margins and volume. A) mixed markup, high-volume B) low-volume, mixed markup C) low-volume, low-markup D) high-volume, low-markup E) high-markup, low-volume

D

Which of the following has led to increased success of new products, even though new products continue to fail at estimated rates of 50% to 95% in the United States and 90% in Europe?

Disruptive Technologies

118) The sales gap due to reduced volume is ________ percent. A) 0.4 B) 2.5 C) 28.5 D) 63 E) 71.4

E

The questions "Do other products currently meet this need and satisfy you?" is asked during the concept testing process and measures the _____.

Gap Level

60) A franchise organization is an example of a(n) ________ vertical marketing system. A) corporate B) administered C) contractual D) regulatory E) controlled

c

116) Operating management is most effective in controlling direct costs and traceable common costs.

TRUE Page Ref: 659 Objective: APP Difficulty: Easy

6) ________ advertising aims to stimulate repeat purchase of products and services. A) Reinforcement B) Comparative C) Persuasive D) Informational E) Reminder

E

1) Which of the following is most closely related with the organic growth of an organization? A) acquiring a product or service brand B) entering new marketplaces C) increasing the operational profitability D) increasing productivity of employees E) developing new products from within

E Diff: 2 LO: 15.1: How can new products be categorized? AACSB: Reflective thinking

20) Companies that aim to ________ strive to be affordable luxuries. A) survive in the market B) partially recover their costs C) maximize their market share D) pursue value pricing E) be product-quality leaders

E Page Ref: 390 Objective: 2 Difficulty: Moderate

64) In ________, the seller charges a separate price to each customer depending on the intensity of his or her demand. A) second-degree price discrimination B) third-degree price discrimination C) psychological discounting D) special-customer pricing E) first-degree price discrimination

E Page Ref: 406 Objective: 3 Difficulty: Easy

The trucking firm hired by your transportation manager provides a trucking service between the city of Sacramento and the surrounding towns on a regular schedule and at fixed prices. The trucking firm saves transportation costs by transporting the goods using trains as well as trucks, instead of trucks alone. Your transportation manager has hired a(n) ________ carrier. A) airship B) airtruck C) trainship D) fishyback E) piggyback

E) piggyback

152) Jaycee was using ________ when his organization tried to win the support of the leaders of one of his distributors by including them in advisory councils, boards of directors, and the like. A) dual compensation B) employee exchange C) co-optation D) joint membership E) strategic justification

C

31) Which of the following is true for franchisees? A) The franchisee is paid by the franchisor to be part of the franchise system. B) The franchisee licenses a trademark to the franchisor. C) The franchisee must change its operations to suit those of the franchisor. D) The franchisee collects royalty payments from the franchisor. E) The franchisee owns the trade or service mark.

C

70) Flighting as an advertising timing pattern is most useful when ________. A) purchase cycle is rather frequent B) substantial advertising budget is available C) items are seasonal D) tightly defined buyer categories exist E) there are expanding market situations

C

93) Which of the following personal communication channels consist of company salespeople contacting buyers in the target market? A) expert channels B) social channels C) advocate channels D) independent channels E) informal channels

C

According to Narus and​ Anderson, which of the following threatens wholesalers instead of strengthening the relationship with​ them? A. They gain insight into the​ manufacturers' requirements by visiting their plants and attending manufacturer association conventions and trade shows. B. They seek a clear agreement with their manufacturers about their expected functions in the marketing channel. C. They winnow out suppliers based on cost and quality. D. They fulfill their commitments to manufacturers by meeting the volume​ targets, paying bills​ promptly, and feeding back customer information to their manufacturers. E. They identify and offer​ value-added services to help their suppliers.

C

Airfare (Scenario) The airfare for an economy class, one-way ticket from Los Angeles to New York is $500. Due to the recession, the airline manages to fill only 100 out of the 150 seats at $400 per seat. 117) How much of the sales performance gap is due to price decline? A) 0.4 percent B) 2.5 percent C) 28.5 percent D) 37 percent E) 71.4 percent

C

103) ________ are consumer promotion tools that provide a price reduction after purchase rather than at the retail shop. A) Rebates B) Cents-off deals C) Price packs D) Coupons E) Premiums

A

83) In your neighborhood there is a small men's store that has a limited selection of clothing, but the selection that is carried is of very high quality and price. Services include free alterations and tailoring, personalized recordkeeping, and free dry cleaning. Inside the store are deep leather chairs and couches and thick pile carpet. Upon entering the store, a customer feels "special and rich." In terms of differentiation, what is this retailer trying to communicate by its decorations and service level? A) differentiation based on services mix and atmosphere B) differentiation based on prepurchase services C) differentiation based on postpurchase services and atmosphere D) differentiation based on ancillary services and atmosphere E) differentiation based on prepurchase and postpurchase services

A

Stores are using​ ________ to measure a​ product's handling costs from the time it reaches the warehouse until a customer buys it in the retail store. A. direct product profitability​ (DPP) B. ​radio-frequency identification​ (RFID) C. global positioning systems​ (GPS) D. compounded annual growth rate​ (CAGR) E. electronic data interchange​ (EDI)

A

63) ________ is market share in comparison to the largest competitor. A) Relative market share B) Served market share C) Overall market share D) Market value E) Target market share

A Page Ref: 654 Objective: APP Difficulty: Easy

140) What is a brand-positioning map?

A brand-positioning map is a perceptual map showing the current positions of various existing brands competing with each other. Page Ref: 582 Objective: 4 Difficulty: Easy

47) Define a category-management organization.

A category-management organization is where a company focuses on product categories to manage its brands.

128) Define a category-management organization.

A category-management organization is where a company focuses on product categories to manage its brands. Page Ref: 628 Objective: 2 Difficulty: Easy

Which of the following is an example of a word-of-mouth marketing communication platform? A) chat rooms B) billboards C) factory tours D) incentive programs E) trade shows

A) Chat Rooms

Which of the following is an example of a category killer? A) PETCO B) Walmart C) Kohl's D) The Limited E) Tesco

A) PETCO

Which of the following is NOT one of the four important contributions an effectively trained company sales force can make to consumer marketing? A) remind end-consumers about the product B) increase stock position C) build enthusiasm D) conduct missionary selling E) manage key accounts

A) Remind end consumers about the product

Beyond the optimal order quantity, total cost per unit increases because ________. A) inventory-carrying cost per unit increases B) inventory-carrying cost per unit decreases C) order-processing cost per unit increases D) order-processing cost per unit increases though inventory cost decreases E) inventory-processing cost per unit falls slowly

A) inventory-carrying cost per unit increases

52) Many people believe that customer focus does not help to create better, or new, products. This belief rests on which of the following ideas? A) Such new product developments decrease the chances of success. B) Customers do not consider the cost of design when making suggestions. C) Customers are at times unaware of what they really want. D) It increases the cost of testing a product or service. E) Having too much customer focus leads to a negative brand image.

C Diff: 2 LO: 15.4: What are the main stages in developing new products and services? AACSB: Reflective thinking

53) The ________ technique for stimulating creativity lists all the characteristics of an object and then modifies each to try to arrive at a new idea. A) reverse assumption analysis B) lateral marketing C) attribute listing D) forced relationship E) morphological analysis

C Diff: 2 LO: 15.4: What are the main stages in developing new products and services? AACSB: Reflective thinking

26) A(n) ________ is a storeless retailer serving a specific clientele — usually employees of large organizations — who are authorized to buy from a list of retailers that have agreed to give discounts in return for inclusion on the list. A) direct-selling vendor B) direct marketing vendor C) buying service D) automatic vendor E) corporate retailer

C Diff: 2 LO: 18.1: What major types of marketing intermediaries occupy this sector? AACSB: Analytical thinking; Application of knowledge

5) Reynold's is a grocery chain that has always catered to mid-market customers. However, the owner, Mal, has noticed that an influx of new residents are buying mostly the lower-cost and discounted products. To attract customers, Mal decides to make a gradual switch to the discount store format, but to do this, he will have to cut costs wherever possible. Which of the following types of services should Mal avoid in order to lower costs? A) limited service B) self-selection C) full-service D) self-service E) limited-selection

C Diff: 2 LO: 18.1: What major types of marketing intermediaries occupy this sector? AACSB: Analytical thinking; Application of knowledge

26) Many believe that customer focus does not help to create better or new products. This belief rests on which of the following ideas? A) Such new product developments decrease the chances of success. B) Customers do not consider the cost of design when suggesting. C) Customers are at times unaware of what they really want. D) It increases the cost of testing a product or service. E) Having too much customer focus leads to a negative brand image.

C Page Ref: 578 Objective: 3 Difficulty: Moderate

27) The ________ technique for stimulating creativity lists all the characteristics of an object and then modifies each to try to arrive at a new idea. A) reverse assumption analysis B) lateral marketing C) attribute listing D) forced relationship E) morphological analysis

C Page Ref: 579 Objective: 3 Difficulty: Easy

Which of the following personal communication channels consist of company salespeople contacting buyers in the target market? A) expert channels B) social channels C) advocate channels D) independent channels E) informal channels

C) Advocate Channels

Which of the following is an example of a business and sales force promotion? A) advertising allowance B) free samples C) contests for sales reps D) display allowance E) discount coupons

C) Contests for Sales Reps

Which of the following sources of a spokesperson's credibility describes his or her attractiveness? A) expertise B) trustworthiness C) likability D) integrity E) experience

C) Likability

16) Aldi, Lidl, Dollar General, and Family Dollar are examples of ________, as they carry a more restricted merchandise mix than discount stores at even lower prices. A) off-price retailers B) extreme value stores C) superstores D) convenience stores E) specialty stores

B

28) Which of the following is true of building a creative marketing organization? A) It is enough if firms are customer-oriented. B) Companies must watch trends and be ready to capitalize on them. C) Firms should focus more on efficiency rather than innovation. D) Companies should attempt to minimize risks as much as possible. E) Firms should focus on protecting their existing markets and physical resources.

B

30) Place advertising, or out-of-home advertising, is a broad category that includes many creative and unexpected forms to grab consumers' attention, including all of the following EXCEPT ________. A) billboards B) yellow pages C) public spaces D) product placement E) point of purchase

B

42) As a service output produced by marketing channels, product variety refers to the ________. A) units the channel permits a customer to purchase at once B) assortment provided by the marketing channel C) add-on services provided by the channel D) ability of a product to provide incremental value E) degree to which the channel makes it easy for customers to purchase a product

B

77) Broomer threatens to withdraw all its other products from the retailers' stores if they are unwilling to push products from the "Inducer" line. This is an example of ________ power. A) reward B) coercive C) legitimate D) expert E) referent

B

8) As a business practice, broadening involves ________. A) acquiring or merging with firms in the same or complementary industries to gain economies of scale and scope B) factoring the interests of customers, employees, shareholders, and other stakeholders into the activities of the enterprise C) buying more goods and services from outside domestic or foreign vendors D) appointing teams to manage customer-value-building processes and break down walls between departments E) becoming more accountable by measuring, analyzing, and documenting the effects of marketing actions

B

8) Which of the following elements of the marketing communications mix includes a variety of programs directed internally to employees of the company or externally to consumers, other firms, the government, and media to promote or protect a company's image or its individual product communications? A) direct marketing B) public relations and publicity C) personal selling D) advertising E) sales promotion

B

83) One of the possible objectives of marketing communications is helping consumers evaluate a brand's perceived ability to meet a currently relevant need. Which of the following relevant brand needs is most likely emphasized by an advertisement for a luxury car? A) problem removal B) sensory gratification C) normal depletion D) intellectual stimulation E) problem avoidance

B

32) ________ are informal workplaces, sometimes garages, where intrapreneurial teams attempt to develop new products. A) Stage-gate systems B) Skunkworks C) Funnels D) Opportunity spaces E) Research centers

B Diff: 2 LO: 15.3: What organizational structures and processes do managers use to oversee new-product development? AACSB: Reflective thinking

22) The ________ marketing sales system works by recruiting independent businesspeople who act as distributors. A) catalog B) multilevel C) direct-response D) corporate E) direct

B Diff: 2 LO: 18.1: What major types of marketing intermediaries occupy this sector? AACSB: Analytical thinking

106) Because shelf space is scarce, many supermarkets now charge a ________ for accepting a new brand, to cover the cost of listing and stocking it. A) retainer B) slotting fee C) residual fee D) contingent fee E) royalty

B Diff: 2 LO: 18.4: What does the future hold for private label brands? AACSB: Analytical thinking

118) Agricultural assemblers, petroleum bulk plants and terminals, and auction companies are examples of ________. A) full-service wholesalers B) specialized wholesalers C) limited-service wholesalers D) merchant wholesalers E) brokers

B Diff: 2 LO: 18.5: What are some of the important issues in wholesaling? AACSB: Analytical thinking; Application of knowledge

30) A company must make payments each month for rent, heat, interest, and salaries. These are ________. A) total costs B) fixed costs C) variable costs D) opportunity costs E) target costs

B Page Ref: 393 Objective: 2 Difficulty: Easy

25) Transforming into a true market-driven company requires organizing around ________. A) sales B) customer segments C) products D) functions E) brands

B Page Ref: 630 Objective: 2 Difficulty: Easy

26) Which of the following is true of building a creative marketing organization? A) It is enough if firms are customer-oriented. B) Companies must watch trends and be ready to capitalize on them. C) Firms should focus more on efficiency rather than innovation. D) Companies should attempt to minimize risks as much as possible. E) Firms should focus on protecting their existing markets and physical resources.

B Page Ref: 630-631 Objective: 2 Difficulty: Moderate

Which of the following tools or combinations of tools is most influential at the comprehension stage of buyer readiness? A) sales promotion and advertising B) advertising and personal selling C) publicity and personal selling D) reminder advertising and publicity E) sales promotion and personal selling

B) Advertising and personal selling

159) What are the ten deadly marketing sins? Please include the signs that the sin exists and a suggested solution for each.

Deadly Sin #1: The company is not sufficiently market focused and customer driven. Signs: There is evidence of poor identification of market segments, poor prioritization of market segments, no market segment managers, employees who think it is the job of marketing and sales to serve customers, no training program to create a customer culture, and no incentives to treat the customer especially well. Solutions: Use more advanced segmentation techniques, prioritize segments, specialize the sales force, develop a clear hierarchy of company values, foster more "customer consciousness" in employees and company agents, and make it easy for customers to reach the company and respond quickly to any communication. Deadly Sin #2: The company does not fully understand its target customers. Signs: The latest study of customers is three years old; customers are not buying your product like they once did; competitors' products are selling better; and there is a high level of customer returns and complaints. Solutions: Do more sophisticated consumer research, use more analytical techniques, establish customer and dealer panels, use customer relationship software, and do data mining. Deadly Sin #3: The company needs to better define and monitor its competitors. Signs: The company focuses on near competitors, misses distant competitors and disruptive technologies, and has no system for gathering and distributing competitive intelligence. Solutions: Establish an office for competitive intelligence, hire competitors' people, watch for technology that might affect the company, and prepare offerings like those of competitors. Deadly Sin #4: The company does not properly manage relationships with stakeholders. Signs: Employees, dealers, and investors are not happy; and good suppliers do not come. Solutions: Move from zero-sum thinking to positive-sum thinking; and do a better job of managing employees, supplier relations, distributors, dealers, and investors. Deadly Sin #5: The company is not good at finding new opportunities. Signs: The company has not identified any exciting new opportunities for years, and the new ideas the company has launched have largely failed. Solutions: Set up a system for stimulating the flow of new ideas. Deadly Sin #6: The company's marketing planning process is deficient. Signs: The marketing plan format does not have the right components, there is no way to estimate the financial implications of different strategies, and there is no contingency planning. Solutions: Establish a standard format including situational analysis, SWOT, major issues, objectives, strategy, tactics, budgets, and controls; ask marketers what changes they would make if they were given 20 percent more or less budget; and run an annual marketing awards program with prizes for best plans and performance. Deadly Sin #7: Product and service policies need tightening. Signs: There are too many products and many are losing money; the company is giving away too many services; and the company is poor at cross-selling products and services. Solutions: Establish a system to track weak products and fix or drop them; offer and price services at different levels; and improve processes for cross-selling and up-selling. Deadly Sin #8: The company's brand-building and communications skills are weak. Signs: The target market does not know much about the company; the brand is not seen as distinctive; the company allocates its budget to the same marketing tools in about the same proportion each year; and there is little evaluation of the ROI impact of marketing communications and activities. Solutions: Improve brand-building strategies and measurement of results; shift money into effective marketing instruments; and require marketers to estimate the ROI impact in advance of funding requests. Deadly Sin #9: The company is not organized for effective and efficient marketing. Signs: Staff lacks 21st-century marketing skills, and there are bad vibes between marketing/sales and other departments. Solutions: Appoint a strong leader and build new skills in the marketing department, and improve relationships between marketing and other departments. Deadly Sin #10: The company has not made maximum use of technology. Signs: There is evidence of minimal use of the Internet, an outdated sales automation system, no market automation, no decision-support models, and no marketing dashboards. Solutions: Use the Internet more, improve the sales automation system, apply market automation to routine decisions, and develop formal marketing decision models and marketing dashboards.

127) What does the term organic growth mean?

Organic growth refers to the development of new products from within. Page Ref: 569 Objective: 1 Difficulty: Easy

The trap associated with a price-cutting strategy where a higher-priced competitor matches lower prices but has longer staying power because of deeper cash reserves is the _______.

Shallow-Pockets Trap

______ are informal workplaces, sometimes garages, where entrepreneurial teams attempt to develop new products.

Skunkworks

36) What are the various functions performed by members of a marketing channel? Provide examples.

Some of the functions that the members of a marketing channel perform (storage and movement, title, and communications) constitute a forward flow of activity from the company to the customer; other functions (ordering and payment) constitute a backward flow from customers to the company. Still others (information, negotiation, finance, and risk taking) occur in both directions.

112) What should manufacturers do to compete against or collaborate with private labels?

• Fight selectively: manufacturers should focus on products where they can win against private labels and add value for consumers, retailers, and shareholders. • Partner effectively: seek win-win relationships with retailers through strategies that complement the retailer's private labels. • Innovate brilliantly: Add new products to beat private labels. • Create winning value propositions.

133) Repurchase rate is a sales metric that is used to evaluate the performance of marketing plans.

FALSE

137) A marketing audit only benefits a company that is in trouble; companies in good health do not need to conduct them.

FALSE

148) If setup costs are high, the manufacturer can produce the item often, and the average cost per item is stable and equal to the running costs.

FALSE

150) The public relations function of lobbying involves advising management about public issues as well as company positions and image during good times and bad.

FALSE

155) If a company has too many products and many are losing money; the company is giving away too many services; and the company is poor at cross-selling products and services, it is a sign that the company's brand-building and communication skills are weak.

FALSE

23) The pervasive nature of advanced communications technologies, such as the Internet, has resulted in greater exposure of customers to advertising content than it was in the 1960s.

FALSE

42) A retailer cooperative is a retail firm that is owned by its customers.

FALSE

46) Corporate chain stores such as Subway, Jiffy-Lube, Holiday Inn, Supercuts, and 7-Eleven account for $1 trillion of annual US sales and 40 percent of all retail transactions.

FALSE

46) Public relations is incapable of reaching prospects who prefer to avoid mass media and targeted promotions.

FALSE

67) Selective distribution severely limits the number of intermediaries and is appropriate when the producer wants to maintain control over the service level and outputs offered by the resellers.

FALSE

96) A small change in price of a product within the price indifference band causes a substantial change in the demand of that product.

FALSE Page Ref: 392 Objective: 2 Difficulty: Easy

98) In target-return pricing, the firm adds a standard markup to the product's cost.

FALSE Page Ref: 397 Objective: 2 Difficulty: Easy

99) The key to effectively using perceived-value pricing is to deliver value that is on par with your competitors.

FALSE Page Ref: 399 Objective: 2 Difficulty: Easy

101) In high-low pricing, retailers charge low prices on an everyday basis with occasional price increases.

FALSE Page Ref: 401 Objective: 2 Difficulty: Easy

103) In a compensation deal, the seller sells a plant, equipment, or technology to another country and agrees to accept as partial payment products manufactured with the supplied equipment.

FALSE Page Ref: 404 Objective: 3 Difficulty: Easy

112) Price discrimination in all forms is illegal in the United States.

FALSE Page Ref: 407 Objective: 3 Difficulty: Easy

113) Predatory pricing, which refers to the concept of selling below cost with the intention of destroying competition, is lawful under certain conditions.

FALSE Page Ref: 407 Objective: 3 Difficulty: Easy

80) High-tech firms that function in a market with high technological uncertainty, high market uncertainty, and high investment costs are not likely to seek radical innovation.

FALSE Page Ref: 570 Objective: 1 Difficulty: Easy

85) Almost all companies use the conventional percentage-of-sales figure while budgeting for new product development.

FALSE Page Ref: 573 Objective: 2 Difficulty: Easy

88) A business analysis is performed mainly to identify if a company has got a technically and commercially sound product.

FALSE Page Ref: 575 Objective: 2 Difficulty: Easy

104) A major drawback of conjoint analysis is that it cannot be used to measure objective attributes such as estimated market share and profit.

FALSE Page Ref: 583 Objective: 4 Difficulty: Easy

106) After a test, the most customer-appealing offer will be the most profitable offer to make.

FALSE Page Ref: 584 Objective: 4 Difficulty: Easy

110) Modular function deployment methodology takes the list of desired customer attributes generated by market research and turns them into a list of engineering attributes that engineers can use.

FALSE Page Ref: 587 Objective: 4 Difficulty: Easy

111) Beta testing tests the product within the firm to see how it performs in different applications.

FALSE Page Ref: 587 Objective: 4 Difficulty: Easy

114) Some firms might delay new product launch until after the competitor has borne the cost of educating the market and its product may reveal flaws the late entrant can avoid. This can be classified as a parallel entry.

FALSE Page Ref: 590 Objective: 4 Difficulty: Easy

116) Compatibility refers to the degree to which the innovation matches the values and experiences of the individuals.

FALSE Page Ref: 592 Objective: 5 Difficulty: Easy

79) The product-management organization replaces the functional organization in the firm.

FALSE Page Ref: 627 Objective: 2 Difficulty: Easy

80) An advantage of the product- and brand-management system is that product and brand managers focus the company on building market share rather than customer relationships.

FALSE Page Ref: 628 Objective: 2 Difficulty: Easy

83) When customers fall into different user groups with distinct buying preferences and practices, a product team structure is desirable.

FALSE Page Ref: 629 Objective: 2 Difficulty: Easy

86) Many companies now focus on departments as opposed to processes, because processes can be a barrier to smooth performance.

FALSE Page Ref: 629 Objective: 2 Difficulty: Easy

97) Cash donations are the most productive contribution that businesses can make to a nonprofit or community group.

FALSE Page Ref: 638 Objective: 3 AACSB: Ethical understanding and reasoning abilities Difficulty: Easy

98) Many companies focus on multiple cause-related marketing programs to simplify execution and maximize impact.

FALSE Page Ref: 638 Objective: 3 AACSB: Ethical understanding and reasoning abilities Difficulty: Easy

100) Social marketing programs designed to motivate people to donate blood or attract people for mass immunization are examples of cognitive campaigns.

FALSE Page Ref: 640 Objective: 3 AACSB: Ethical understanding and reasoning abilities Difficulty: Easy

104) Profitability control is the prime responsibility of line and staff management.

FALSE Page Ref: 643 Objective: 5 Difficulty: Easy

148) When Gina's company printed the ad for their Perfume in the newspapers, the caption read, "WAS $100, NOW $75". What kind of a promotional pricing did her company use?

Gina's company used psychological discounting. This strategy sets an artificially high price and then offers the product at substantial savings. Page Ref: 405 Objective: 3 Difficulty: Moderate

Which of the following is one of the questions a company needs to consider in a homogeneous product market?

How can we enhance our augmented product?

135) What is crowdsourcing?

Crowdsourcing means inviting the Internet community to help create content or software, often with prize money or a moment of glory as an incentive. Page Ref: 577 Objective: 3 Difficulty: Easy

16) Which of the following is an example of an interactive marketing communication platform? A) product demonstrations B) factory tours C) company museums D) TV shopping E) community relations

D

44) What is the gross rating points (GRP) for a media schedule that reaches 60 percent of homes with an average exposure frequency of 4 and impact of 1.5? A) 10 B) 15 C) 160 D) 240 E) 360

D

Cabot (Scenario) Cabot, a large car manufacturer, has four popular car brands in different segments. It has a manufacturing facility near Detroit, MI, where parts common to all the four brands are manufactured. Other specific parts like the brake system, windshield, bonnet, locks, and so on are manufactured in separate plants. Each make has its own product manager and support staff. All product managers report to the CEO of the company. 124) Cabot's CEO's annual compensation is an example of a ________ cost. A) direct B) variable C) traceable common D) nontraceable common E) manufacturing

D

​________ such as​ Subway, Jiffy-Lube, Holiday​ Inn, Supercuts, and​ 7-Eleven account for​ $1 trillion of annual U.S. sales and​ 40% of all retail transactions. A. Retailer cooperatives B. Corporate chain stores C. Merchandising conglomerates D. Franchise businesses E. Consumer cooperatives

D

115) ________ serve bulk industries such as coal, lumber, and heavy equipment, assuming title and risk from the time an order is accepted to its delivery. A) Producers' cooperatives B) Cash and carry wholesalers C) Truck wholesalers D) Drop shippers E) Rack jobbers

D Diff: 2 LO: 18.5: What are some of the important issues in wholesaling? AACSB: Reflective thinking

78) American businessman King Gillette pioneered the sales model in which razor handles were given away for free or sold at a loss, but sales of disposable razor blades were very profitable. This is known as the ________ model. A) two-tiered pricing B) predatory pricing C) cross selling D) loss leading E) product churning

D Diff: 3 LO: 18.3: What marketing decisions do marketing intermediaries make? AACSB: Analytical thinking; Application of knowledge

70) Troma Inc., is a famous manufacturer of cookware, that follows a traditional distributor-retailer system to distribute its products. The company abstains from the use of automated supply chain management (SCM) system mainly due to the fear of unknown. However, rapidly escalating operational costs and inefficiencies have made it absolutely necessary for the company to implement an SCM system. The company goes for a big-bang installation of SCM system to become more competitive and cost effective. Identify the adopter group to which Troma belongs. A) innovator B) early adopter C) early majority D) laggard E) late majority

D Page Ref: 591 Objective: 5 AACSB: Analytic skills Difficulty: Moderate

27) Rising customer expectations, evolving employee goals and ambitions, and tighter government legislation and pressure are driving companies to ________. A) operate leaner manufacturing facilities B) manage shorter supply chains C) operate flatter organizations D) practice a higher level of corporate social responsibility E) vertically integrate

D Page Ref: 631 Objective: 3 AACSB: Ethical understanding and reasoning abilities Difficulty: Easy

12) Which of the following elements of the marketing communications mix involves face-to-face interaction with one or more prospective purchasers for the purpose of making presentations, answering questions, and procuring orders? A) advertising B) sales promotion C) word-of-mouth marketing D) public relations E) personal selling

E

125) A large retail chain in the United States decides to expand its operations by adding an online site for e-commerce. This is called a(n) ________ company. A) B2B B) brick-and-mortar C) m-commerce D) pure-click E) brick-and-click

E

7) Jordan's firm enters new markets by tweaking products for new customers, uses variations on a core product to stay one step ahead of the market, and creates interim solutions for industry-wide products. In other words, it uses ________. A) disruptive technologies B) incremental innovation C) complex innovations D) discontinuous innovations E) radical innovations

E Diff: 2 LO: 15.1: How can new products be categorized? AACSB: Application of knowledge

49) ________ means inviting the Internet community to help create content or software, often with prize money or a moment of glory involved. A) Stage-gating B) Cocreation C) Microstocking D) Buzzing E) Crowdsourcing

E Diff: 2 LO: 15.4: What are the main stages in developing new products and services? AACSB: Reflective thinking

120) For a frequently purchased new product, the seller estimates repeat sales as well as first-time sales. A high rate of repeat purchasing means customers ________. A) value price more than differentiation B) do not support innovation on brands C) value differentiation more than price D) prefer personalized products rather than standard ones E) are satisfied with the product

E Diff: 2 LO: 15.7: What is the best way to manage the commercialization of new products? AACSB: Reflective thinking

142) Which of the following characteristics is closely associated with late majority adopter groups? A) superior technological knowledge B) low price sensitiveness C) opinion leadership D) deliberate pragmatism E) high risk aversion

E Diff: 2 LO: 15.8: What factors affect the rate of diffusion and consumer adoption of newly launched products and services? AACSB: Reflective thinking

145) ________ is the effect one person has on another's attitude or purchase probability. A) Sharing power B) Collaborative power C) Market influence D) Brand power E) Personal influence

E Diff: 2 LO: 15.8: What factors affect the rate of diffusion and consumer adoption of newly launched products and services? AACSB: Reflective thinking

25) Electronic shopping is a type of ________. A) direct selling B) network marketing C) multilevel selling D) corporate selling E) direct marketing

E Diff: 2 LO: 18.1: What major types of marketing intermediaries occupy this sector? AACSB: Analytical thinking

143) The trucking firm hired by your transportation manager provides a trucking service between the city of Sacramento and the surrounding towns on a regular schedule and at fixed prices. The trucking firm saves transportation costs by transporting the goods using trains as well as trucks, instead of trucks alone. Your transportation manager has hired a(n) ________ carrier. A) airship B) airtruck C) trainship D) fishyback E) piggyback

E Diff: 2 LO: 18.6: What are some important issues in logistics? AACSB: Analytical thinking; Application of knowledge

13) After estimating the demand and costs associated with alternative prices, a company has chosen to price its product in such a way that it gains the highest rate of return on its investment. The company is looking to ________. A) maximize its market share B) skim the market C) become a product-quality leader D) survive in the market E) maximize its current profit

E Page Ref: 389 Objective: 2 Difficulty: Easy

21) Starbucks, Aveda, and BMW have been able to position themselves within their categories by combining quality, luxury, and premium prices with an intensely loyal customer base. These companies are employing a ________ strategy. A) market-skimming B) market-penetration C) survival D) market share maximization E) product-quality leadership

E Page Ref: 390 Objective: 2 Difficulty: Easy

39) Despite its weaknesses, markup pricing remains popular for which of the following reasons? A) Sellers can determine demand much more easily than they can estimate costs. B) By tying the price to cost, the pricing task becomes more sophisticated. C) When all firms in the industry use markup pricing, price competition flourishes. D) Sellers take advantage of buyers when the latter's demand becomes acute. E) Many people feel that cost-plus pricing is fairer to both buyers and sellers.

E Page Ref: 396-397 Objective: 2 Difficulty: Moderate

50) In which of the following auctions does the auctioneer first announce a high price for a product and then slowly decreases the price until a bidder accepts? A) a Dutch auction with one buyer and many sellers B) an English auction with one seller and many buyers C) an ascending bid auction D) a sealed-bid auction E) a Dutch auction with one seller and many buyers

E Page Ref: 402 Objective: 2 Difficulty: Moderate

60) ________ are granted for turning in old item when buying a new one. A) Promotional allowances B) Quantity discounts C) Functional discounts D) Seasonal discounts E) Trade-in allowances

E Page Ref: 404 Objective: 3 Difficulty: Easy

76) When higher-priced competitors match the lower prices but have longer staying power because of deeper cash reserves, it leads to a(n) ________. A) low-quality trap B) fragile-market-share trap C) price war trap D) escalator trap E) shallow-pockets trap

E Page Ref: 408 Objective: 4 Difficulty: Easy

80) In markets that are characterized by products that are highly homogeneous, how should a firm react to a competitor's reduction in price? A) shrink the amount of the product available B) substitute expensive materials or ingredients C) reduce product features D) reduce product services E) augment the product

E Page Ref: 409 Objective: 5 Difficulty: Moderate

1) Which of the following is most closely related with the organic growth of an organization? A) acquiring a product or service brand B) entering new market places C) increasing the operational profitability D) increasing productivity of employees E) developing new products from within

E Page Ref: 569 Objective: 1 Difficulty: Easy

5) Which of the following is the best example of a new-to-the-world product? A) Walmart, the retail giant, opens new stores in an underdeveloped African country. B) Pestorica, a publishing company, decides to launch a new sports magazine. C) Tata Motors, an Indian automobile company, acquires Jaguar to extend its business. D) An Asian company licenses a U.S. apparel brand name though the brand is not familiar in Asia. E) Kids-Med, a company that produces childcare products, launches a non-contact thermometer.

E Page Ref: 570 Objective: 1 AACSB: Analytic skills Difficulty: Moderate

21) Which of the following uses conceptual lens and structured innovative-thinking tools to achieve market perspectives from different angles? A) business analysis B) strategic blueprint C) internal design analysis D) demand landscape E) opportunity space

E Page Ref: 576 Objective: 3 Difficulty: Easy

24) ________ means inviting the Internet community to help create content or software, often with prize money or a moment of glory involved. A) Stage-gating B) Cocreation C) Microstocking D) Buzzing E) Crowdsourcing

E Page Ref: 577 Objective: 3 Difficulty: Easy

71) Which of the following characteristics is closely associated with late majority adopter groups? A) superior technological knowledge B) low price sensitiveness C) opinion leadership D) deliberate pragmatism E) high risk aversion

E Page Ref: 591 Objective: 5 Difficulty: Easy

74) ________ is the effect one person has on another's attitude or purchase probability. A) Sharing power B) Collaborative power C) Market influence D) Brand power E) Personal influence

E Page Ref: 592 Objective: 5 Difficulty: Easy

17) A disadvantage of the product-management organization is that ________. A) it marginalizes a company's smaller brands B) it reduces organizational responsiveness to new products in the market place C) product managers generally exercise authority outside their areas of responsibility D) it prevents product managers from gaining sufficient expertise in their product areas E) it fails to build long-term strengths as brand managers normally manage brands for only short periods

E Page Ref: 628 Objective: 2 Difficulty: Moderate

20) Which of the following is true about a brand-asset management team (BAMT)? A) A BAMT is part of the triangular and vertical product team structures. B) BAMTs often report directly to the organization's chief branding officer. C) BAMTs are a traditional, tried and tested means of managing brands. D) Companies with a product-management organization often have only one BAMT. E) The BAMT consists of key representatives from functions affecting the brand's performance.

E Page Ref: 628 Objective: 2 Difficulty: Moderate

28) In order to promote ethical cultures, companies should do all of the following EXCEPT ________. A) disseminate a written code of ethics B) build a company tradition of ethical behavior C) hold people responsible for observing ethical and legal guidelines D) ensure every employee knows and observes relevant laws E) encourage business practices that are not clearly ethical or unethical

E Page Ref: 633 Objective: 3 AACSB: Ethical understanding and reasoning abilities Difficulty: Easy

60) Which of the following is a best practice in business and marketing? A) end-product orientation B) reacting to competitors C) vertical integration D) stockholder driven E) teamwork

E Page Ref: 648 Objective: 5 Difficulty: Moderate

68) Annual-plan control requires making sure the company isn't overspending to achieve sales goals. The key ratio to watch is ________. A) stock turnover B) gross margin C) return on capital D) cash flow return on investment E) marketing expense-to-sales

E Page Ref: 655 Objective: APP Difficulty: Easy

72) Suppose the manufacturer pays a commission on every car sold. Then, the salesperson's commission is classified as a(n) ________. A) cost of labor B) traceable common cost C) non-traceable common cost D) advertising cost E) direct cost

E Page Ref: 659 Objective: APP AACSB: Analytic skills Difficulty: Moderate

An example of a restaurant with a narrow and deep assortment is a ________. A) small lunch counter B) cafeteria C) large restaurant D) casual dining restaurant chain E) delicatessen

E) delicatessen

Electronic shopping is a type of ________. A) direct selling B) network marketing C) multilevel selling D) corporate selling E) direct marketing

E) direct marketing

Telemarketing is a type of ________. A) direct selling B) network marketing C) multilevel selling D) close-range marketing E) direct marketing

E) direct marketing

A store selling expensive artwork and luxury goods typically falls into the ________ group with respect to margins and volume. A) mixed markup, high-volume B) low-volume, mixed markup C) low-volume, low-markup D) high-volume, low-markup E) high-markup, low-volume

E) high-markup, low-volume

Though it is sold only in Walmart stores, Walmart's Ol'Roy dog food has surpassed Nestlé's Purina brand as the top-selling dog food. Ol'Roy is an example of a ________. A) generic product B) national brand C) franchise D) copy-cat brand E) private label

E) private label

A brand developed by a retailer and/or wholesaler that is available only in selected retail outlets is called a ________ brand. A) generic B) franchisee C) marque D) national E) private-label

E) private-label

118) DMM Industries, a manufacturer of composite metal products, sells its products using a conventional marketing channel. The company decides to adopt a vertical marketing system to improve its performance. What advantages could this provide?

Each entity in a conventional channel is a separate business seeking to maximize its own profits, even if this goal reduces profit for the system as a whole. A vertical marketing system, by contrast, acts as a unified system. This would make the company more profitable. VMSs achieve economies through size, bargaining power, and elimination of duplicated services.

140) Ellie's manager has asked her to come up with ways to reduce costs of their new product by utilizing a process called "target costing." What should Ellie do?

Ellie will have to ask the marketing research department to establish the new product's desired functions and the price at which the product will sell, given its appeal and competitors' prices. Deducting the desired profit margin from this price leaves the target cost that must be achieved. Her company will then have to examine each cost element like the design, engineering, manufacturing, and sales and bring down costs so that the final cost projections are in the target range. Page Ref: 394 Objective: 2 AACSB: Analytic skills Difficulty: Moderate

61) Which of the following is an example of a category killer? A) PETCO B) Walmart C) Kohl's D) The Limited E) Tesco

A

37) Competitors are most likely to react to a price change, when ________. A) the firm has a weak value proposition B) the firm enjoys a monopoly C) there are few competing firms D) the product is heterogeneous E) buyers have limited information

C Page Ref: 395 Objective: 2 Difficulty: Moderate

140) In the context of overall market share, customer loyalty refers to the percentage of all customers who buy from the company.

FALSE

133) What is a spiral development process?

A spiral development process recognizes the value of returning to an earlier stage to make improvements before moving forward. Page Ref: 575 Objective: 2 Difficulty: Easy

151) The order-processing cost per unit increases with the number of units ordered because the order costs are spread over more units.

FALSE

153) Conflicts between various franchisees of a company are an example of vertical channel conflict.

FALSE

123) Identify some of the brand benefits that can accrue to a company that engages in cause marketing.

A successful cause-marketing program can improve social welfare, create differentiated brand positioning, build strong consumer bonds, enhance the company's public image, create a reservoir of goodwill, boost internal morale and galvanize employees, drive sales, and increase the firm's market value. Consumers may develop a strong, unique bond with the firm running the cause-marketing that transcends normal marketplace transactions. Specifically, cause marketing can (1) build brand awareness, (2) enhance brand image, (3) establish brand credibility, (4) evoke brand feelings, (5) create a sense of brand community, and (6) elicit brand engagement. Page Ref: 637 Objective: 3 AACSB: Ethical understanding and reasoning abilities Difficulty: Moderate

87) Transforming into a true market-driven company involves organizing around products.

FALSE Page Ref: 630 Objective: 2 Difficulty: Easy

50) The ________ is primarily responsible for strategic control. A) line and staff management B) marketing auditor C) marketing controller D) middle management E) BAMT

B Page Ref: 643 Objective: 5 Difficulty: Easy

9) Which of the following is true for self-service retailing? A) They carry more shopping goods and services such as credit and merchandise-return privileges. B) This service model is favored by discount stores and customers who want to save money. C) It results in high staffing costs compared to other forms of retailing. D) Salespeople are ready to assist in every phase of the "locate-compare-select" process. E) Customers need more information and assistance than in other forms of retailing.

B Diff: 1 LO: 18.1: What major types of marketing intermediaries occupy this sector? AACSB: Analytical thinking

15) E&OE produces and markets its own brand of skincare products using herbal remedies and natural ingredients through standalone stores as well as an online portal. E&OE is a(n) ________ retailer. A) off-price B) specialty C) discount D) department E) extreme value

B Diff: 1 LO: 18.1: What major types of marketing intermediaries occupy this sector? AACSB: Analytical thinking; Application of knowledge

34) Which of the following wholesaler functions reduces inventory costs and risks to suppliers and customers? A) selling and promoting B) warehousing C) transportation D) market information E) assortment building

B Diff: 1 LO: 18.1: What major types of marketing intermediaries occupy this sector? AACSB: Reflective thinking

113) Which of the following is considered a wholesaler? A) retailers B) brokers C) producers D) manufacturers E) farmers

B Diff: 1 LO: 18.5: What are some of the important issues in wholesaling? AACSB: Application of knowledge

96) Apple Inc., together with subsidiaries, designs, manufactures, and markets personal computers, mobile phones, and media devices. It also sells software, services, and third-party digital content. Apple is a highly respected brand across the globe. What kind of channel power does this respect give Apple?

Apple is so highly respected that intermediaries are proud to be associated with it. This gives Apple referent power.

14) The most common form of marketing organization consists of ________ reporting to a marketing vice president who coordinates their activities. A) zonal managers B) functional specialists C) product managers D) area market specialists E) brand managers

B

A(n) ________ appeal is a creative strategy that elaborates on a nonproduct-related benefit or image. A) logical B) transformational C) reasonable D) informational E) rational

B) Transformational

141) Alcart Solutions is a large distributor of Aldor phones in Canada. The company distributes products to various retailers in the New Brunswick province. Recently Aldor received several complaints from its retailers that their orders are not delivered on time. They also complain that Alcart offers preferential treatment to some of the other retailers in the region. This is an example of ________ conflict. A) multichannel B) horizontal C) vertical D) intermediate E) parallel

C

22) The first step in estimating demand is to ________. A) analyze competitors' cost B) select a pricing method C) understand what affects price sensitivity D) calculate fixed costs E) decipher the experience curve

C Page Ref: 390 Objective: 2 Difficulty: Moderate

11) Which of the following statements is true of the factors that affect an advertising budget? A) High-market-share brands usually require proportionately high advertising expenditure as a percentage of sales to maintain share. B) Brands in less-differentiated or commodity-like product classes require less advertising to establish a unique image. C) New products typically merit large advertising budgets to build awareness and to gain consumer trial. D) In a market with few competitors and moderate advertising spending, a brand must advertise more heavily to be heard. E) Established brands usually are supported with high advertising budgets, measured as a ratio to sales.

C

28) A ________ is a retail firm owned by its customers. Members contribute money to open their own store, vote on its policies, elect a group to manage it, and receive dividends. A) retailer cooperative B) voluntary chain C) consumer cooperative D) merchandising conglomerate E) franchise organization

C

According to recommendations from Paco​ Underhill, a pioneer in the field of improving the shopper​ experience, which of the following rewards busy shoppers and encourages leisurely shippers to look​ more? A. Make kids welcome. B. Honor the​ "transition zone." C. ​Don't make customers hunt. D. Note that men do not ask questions. E. Avoid overdesign.

C

88) How can firms promote ethical behavior among their employees?

Companies must adopt and disseminate a written code of ethics, build a company tradition of ethical behavior, and hold their people fully responsible for observing ethical and legal guidelines. Organizations must also ensure that every employee knows and observes relevant laws.

_______ is a method for deriving the utility values that consumers attach to varying levels of a product's attributes.

Conjoint Analysis

The three C's model for price setting includes ________.

Costs Competitors' Prices Customers' assessments of unique features

103) Which of the following is a disadvantage of using a percentage-of-sales method to determine the marketing communications budget? A) It discourages stability when competing firms spend approximately the same percentage of their sales on communications. B) By using a percentage-of-sales method, communication expenditures tend to be extremely high irrespective of what a company can afford. C) It discourages management from thinking of the relationship among communication cost, selling price, and profit per unit. D) Dependence of the percentage-of-sales method on year-to-year sales fluctuations interferes with long-range planning. E) The percentage-of-sales method views sales as the result in itself rather than the determiner of communications.

D

Rack​ jobbers, cash-and-carry​ wholesalers, truck​ wholesalers, and​ producers' cooperatives are examples of​ ________. A. brokers B. merchant wholesalers C. specialized wholesalers D. ​full-service wholesalers E. ​limited-service wholesalers

E

146) A customer survey at a mall revealed that Zoe is considered to be an early adopter. What does this mean?

Early adopters are opinion leaders who carefully search for new technologies that might give them a dramatic competitive advantage. They are less price sensitive and willing to adopt the product if given personalized solutions and good service support. Page Ref: 591 Objective: 5 Difficulty: Easy

122) The most highly credible source would score high on at least two of the three dimensions — expertise, trustworthiness, and likability.

FALSE

48) Advertising and publicity play the most important roles in the conviction stage of buyer readiness.

FALSE

59) Lot size refers to the total number of units a channel can transmit from the manufacturer's place to the service outlet.

FALSE

62) When economic conditions are depressed, producers want to move goods to market using longer channels.

FALSE

85) Retailers can target their customers more effectively by using only a single channel to reach them.

FALSE

90) Although salespeople are legally prohibited from saying things about their products that are not true, they may legally suggest things about competitors' products that are not true.

FALSE Page Ref: 633 Objective: 3 AACSB: Ethical understanding and reasoning abilities Difficulty: Easy

91) Some business practices, such as deceptive advertising, exclusive dealing, and predatory competition, sharply divide critics regarding whether they are clearly unethical or illegal.

FALSE Page Ref: 633 Objective: 3 AACSB: Ethical understanding and reasoning abilities Difficulty: Moderate

96) Cause-related marketing efforts are unlikely to backfire as customers generally view the company's motives as genuine.

FALSE Page Ref: 637 Objective: 3 AACSB: Ethical understanding and reasoning abilities Difficulty: Easy

106) The annual plan control process begins with measuring performance.

FALSE Page Ref: 643-644 Objective: 5 Difficulty: Easy

If a calculator company produces 100,000 hand calculators at a cost of $10, but the cost drops to $9 when it produces 200,000 and $8 when it produces 400,000 hand calculators, the decline in average cost with accumulated product experience is call the ____________.

Learning Curve

When the seller absorbs part or all of the risk of a purchase, in the event the purchase does not deliver the full promised value, what type of pricing is being used?

Gain-and-Risk-Sharing Pricing

139) A group of young students run an Internet center. In an attempt to increase their profitability they start a cafe alongside the existing business. Explain the type of marketing used here.

Lateral marketing is used here. It combines two product concepts or ideas to create a new offering. Page Ref: 580 Objective: 3 AACSB: Analytic skills Difficulty: Moderate

Most established companies focus on _______, entering new markets by tweaking products for new customers, using variations on a core product to stay one step ahead of the market, and creating interim solutions for industry-wide products.

Incremental Innovation

95) Describe the challenges and opportunities that international markets pose.

International markets pose distinct challenges, including variations in customers' shopping habits, but opportunities at the same time. Behavior of markets and preferences for intermediaries differ across countries. Foreign markets provide an opportunity for growth and expansion. Many pitfalls also exist in global expansion, and retailers must also be able to defend their home turf from the entry of foreign retailers.

135) Bella's Inc. has estimated the demand and costs associated with alternative prices. It has finally chosen to price its new offering in such a way that it will maximize the rate of return on investment. What can be deduced about the company's objective?

It can be deduced that the company's main objective is to maximize its maximum current profit. Page Ref: 389 Objective: 2 AACSB: Analytic skills Difficulty: Moderate

In the adopter categories, the _______ are skeptical conservatives who are risk averse, technology shy, and price sensitive.

Late Majority

156) Why are marketers concerned about the cost of logistics?

Management has become concerned about the total cost of market logistics, which can amount to as much as 30 percent to 40 percent of the product's cost. In the US grocery business, waste or "shrink" affects 8 percent to 10 percent of perishable goods, costing $20 billion annually. Stop and Shop/Giant Landover, a $16 billion grocery chain, saved an estimated annual $100 million by reducing shrink and improving customer satisfaction with a redesigned perishables department. One farmer who was not able to sell 70 percent of his carrots because of their irregular shape found that he could sell them all, and for $0.50 per pound compared with $0.17, by cutting them into "baby carrots."

Caterpillar uses _________ to set prices on its construction equipment, and justifies a higher price by showing lower lifetime operating costs.

Perceived-Value Pricing

140) Identify three different types of personal communication channels.

Personal communications channels derive their effectiveness from individualized presentation and feedback and include direct marketing, personal selling, and word of mouth. Advocate channels consist of company salespeople contacting buyers in the target market. Expert channels consist of independent experts making statements to target buyers. Social channels consist of neighbors, friends, family members, and associates talking to target buyers.

133) Differentiate between pure-click companies and brick-and-click companies.

Pure-click companies are those that have launched a Web site without any previous existence as a firm, while brick-and-click companies are existing companies that have added an online site for information or e-commerce. There are several kinds of pure-click companies: search engines, Internet service providers (ISPs), commerce sites, transaction sites, content sites, and enabler sites. Brick-and-click companies are formed by adding an e-commerce channel to an existing business.

Customers of a proposed truck might want a certain acceleration rate, which is a desired customer attribute. Engineers can turn this into the required horsepower and other engineering attributes through a process known as _______.

Quality-Function Development

148) Define sale-variance analysis and microsales analysis.

Sales-variance analysis measures the relative contribution of different factors to a gap in sales performance. Microsales analysis looks at specific products, territories, and so forth that failed to produce expected sales.

142) Define sale-variance analysis and microsales analysis.

Sales-variance analysis measures the relative contribution of different factors to a gap in sales performance. Microsales analysis looks at specific products, territories, and so forth that failed to produce expected sales. Page Ref: 654 Objective: APP Difficulty: Easy

143) GameTech International Inc. engages in the design, development, and marketing of interactive electronic bingo systems primarily in the United States. Some top managers want to collect sales-wave research, but others disagree, offering several disadvantages. List these disadvantages.

Sales-wave research can be implemented quickly, conducted with a fair amount of security, and carried out without final packaging and advertising. However, because customers are preselected, it does not indicate trial rates the product would achieve with different sales incentives, nor does it indicate the brand's power to gain distribution and favorable shelf position. Page Ref: 588 Objective: 4 Difficulty: Easy

149) A public is any group that has an actual or potential interest in or impact on a company's ability to achieve its objectives.

TRUE

68) Companies must adopt and disseminate a written code of ethics, build a company tradition of ethical behavior, and hold their people fully responsible for observing ethics and legal guidelines if they wish to demonstrate ethical behavior.

TRUE

128) What are some of the reasons sales promotion expenditures might increase as a percentage of budget expenditure?

The promotion becomes more accepted by top management as an effective sales tool; competitors used promotions frequently; consumers became more price-oriented, and trade demanded more deals from manufacturers.

150) When the airline industries offer discounted but limited early purchases, higher-priced late purchases, and the lowest rates on unsold inventory just before it expires, what kind of a pricing technique are they said to be using?

The airline industries are using yield pricing. Page Ref: 406 Objective: 3 Difficulty: Moderate

155) What is the role of information technology in the logistics system?

The market logistics task calls for integrated logistics systems (ILS), which include materials management, material flow systems, and physical distribution, aided by information technology (IT). Information systems play a critical role in managing market logistics, especially via computers, point-of-sale terminals, uniform product barcodes, satellite tracking, electronic data interchange (EDI), and electronic funds transfer (EFT).

49) A producer must modify its channel design and arrangements if ________. A) consumer buying patterns change B) the competition in the market stabilizes C) the product is in the growth stage of its life cycle D) the market size remains unchanged for a particular period E) the firm's profits stabilize

a

42) Describe the functional organization of a marketing department in terms of its structure, advantages, and disadvantages.

The most common form of marketing organization consists of functional specialists reporting to a marketing vice president. The main advantage of a functional marketing organization is its administrative simplicity. It can be quite a challenge for the department to develop smooth working relationships, however. This form also can result in inadequate planning as the number of products and markets increases and each functional group vies for budget and status. The marketing vice president constantly weighs competing claims and faces a difficult coordination problem.

134) Andersen, the CEO of a home appliance manufacturer, believes in controlled idea generation techniques such as brainstorming when creating new designs and ideas for product revisions. He does not engage in observational methods. What part of DIG framework is eminently used here?

The opportunity space is used here. It uses conceptual lens and structured innovative-thinking tools to achieve market perspectives from different angles. Page Ref: 576 Objective: 3 AACSB: Analytic skills Difficulty: Moderate

61) In a ________ marketing system, two or more unrelated companies put together resources or programs to exploit an emerging marketing opportunity. A) reverse flow B) vertical C) horizontal D) lateral E) forward flow

c

139) A company that pays its bills each month for its rent, heat, interest, and salaries regardless of its output is said to be incurring what type of costs?

These are said to be a company's fixed costs. Page Ref: 393 Objective: 2 AACSB: Analytic skills Difficulty: Moderate

64) Alcart Solutions is a large distributor of Aldor phones in Canada. The company distributes products to various retailers in the New Brunswick province. Recently Aldor received several complaints from its retailers that their orders are not delivered on time. They also complain that Alcart offers preferential treatment to some of the other retailers in the region. This is an example of ________ conflict. A) multichannel B) horizontal C) vertical D) intermediate E) parallel

c

54) Rotter Garder Inc. is a large-scale paint manufacturer and is known for its wide range of decorative paint products and industrial coatings. In addition to making paints, the company also owns and operates the retail stores which sell its products. This is an example of a(n) ________ vertical marketing system. A) administered B) contractual C) referent D) corporate E) regulated

d

65) A franchisee owner is unsatisfied because the manufacturer provides more benefits to a wholesaler. This conflict is an example of a(n) ________ conflict. A) horizontal B) vertical C) intermediate D) multichannel E) parallel

d

126) An Internet service provider (ISP) is a(n) ________ company. A) pure-click B) brick and click C) brick-and-mortar D) m-commerce E) one-level

A

129) Consumer surveys suggest that one of the most significant inhibitors of online shopping is the absence of ________. A) pleasurable experiences B) competitive prices C) adequate technical information D) after-sales service E) facilities to compare offerings

A

44) ________ is a measure of communications effectiveness that describes the percentage of target market exposed to a communication. A) Frequency B) Reach C) Width D) Depth E) Range

B

52) ________ gives products the appearance of being more environmentally friendly without living up to that promise. A) Ambush marketing B) Greenwashing C) Astroturfing D) Viral marketing E) Green politics

B

55) Which of the following is an advantage of using Yellow Pages as an advertising medium? A) low competition B) high believability C) short ad purchase lead time D) greater scope for creativity E) lack of clutter

B

7) The "Got Milk" campaign was intended to boost sagging milk consumption among Californians in the 1990s. The campaign ads highlighted the inconvenience of running out of milk when intended to be used with certain foods, such as cookies or muffins, advising consumers to stock up on milk to avoid such inconveniences. The "Got Milk?" campaign is an example of ________ advertising. A) informational B) reminder C) institutional D) comparative E) reinforcement

B

7) Using the push strategy is most appropriate when ________. A) consumers are able to perceive differences between brands B) the product being sold is an impulse item C) there is high brand loyalty for the product D) the product is a high involvement purchase E) consumers choose the brand before they go to the store

B

Use of Power (Scenario) Broomer manufactures fashion apparel for women, men, and children. Its products are in high demand and apparel stores are more than willing to carry and sell Broomer's garments. The company recently introduced a new clothing line named "Inducer," targeted at youth. However, the new line is not well received by the market, and sales fail to take off even three months after its launch. 75) In an effort to boost sales, Broomer offers its retailers a higher margin for promoting and selling products from the "Inducer" line to customers. This is an example of ________ power. A) coercive B) reward C) passive D) expert E) referent

B

100) A(n) ________ includes the producer, wholesaler(s), and retailer(s) acting as a unified system. A) parallel marketing channel B) vertical marketing system C) extensive marketing channel D) internal marketing system E) conventional marketing channel

B

127) The cost of operating the common manufacturing facility is a(n) ________ cost. A) opportunity B) traceable C) nontraceable D) sunk E) differential

B

129) Becoming part of a personally relevant moment in consumers' lives through ________ can broaden and deepen a company or brand's relationship with the target market. A) advertisements B) events and experiences C) sales promotions D) public relations E) direct marketing efforts

B

38) The number of times within a specified time period that an average person or household is exposed to an advertising message is known as ________. A) impact B) frequency C) amplitude D) reach E) depth

B

57) Which marketing channel is associated with the highest value added per sale? A) retail stores B) sales force C) distributors D) Internet E) telemarketing

B

7) ________, or reducing the number of organizational levels to get closer to the customer, is one of the important shifts in marketing and business practices. A) Merging B) Flattening C) Globalizing D) Focusing E) Justifying

B

8) In the ________ type of retailing, customers usually find their own goods, although they can ask salespeople for assistance. A) self-service B) self-selection C) full-service D) limited service E) limited-selection

B

A) Direct-cost B) Full-cost C) Traceable-cost D) Activity-based costing E) Fixed cost

B

A) Samples B) Coupons C) Rebates D) Price packs E) Premiums

B

33) ________ is the cost per unit at that level of production. A) Target cost B) Average cost C) Marginal cost D) Opportunity cost E) Fixed cost

B Page Ref: 393 Objective: 2 Difficulty: Easy

144) When Estee Lauder set up a Web site to sell its Clinique and Bobbi Brown brands, Dayton Hudson reduced space for Estee Lauder products in its department stores in response to the ________ conflict. A) horizontal channel B) multichannel C) vertical channel D) grid channel E) end-customer

C

104) ________ are unbranded, plainly packaged, less expensive versions of common products such as spaghetti, paper towels, and canned peaches. A) Common carriers B) Shills C) Generics D) Private labels E) Marques

C

120) Which of the following is true about market share? A) Outside forces affect all companies in the same way. B) A company's performance should be judged against the average performance of all companies. C) A decline in market share does not necessarily mean the company is performing worse than are other companies. D) A decline in market share cannot be deliberately engineered. E) All shifts in market share have marketing significance.

C

13) H&A is a retail chain that specializes in selling goods at very low prices. To achieve this, it stocks a very narrow assortment of basic necessities and offers customers a "no-frills" shopping experience. H&A is an example of a(n) ________ store. A) off-price B) specialty C) hard-discount D) superstore E) convenience

C

6) Which of the following is an example of a business and sales force promotion? A) advertising allowance B) free samples C) contests for sales reps D) display allowance E) discount coupons

C

96) Personal influence in marketing communications carries great weight when the ________. A) product being marketed is a convenience item B) purchase of the product is considered to be safe and risk-free C) product suggests something about the user's status or taste D) product being marketed is purchased on a frequent basis E) product or service in questions is used without being recommended by others

C

2) ________ involves buying more goods and services from external domestic or foreign vendors. A) Merging B) Broadening C) Outsourcing D) Globalizing E) Accelerating

C Page Ref: 624 Objective: 1 Difficulty: Easy

33) Cadbury's "Sports for Schools" promotion offered sports and fitness equipment for schools in exchange for vouchers. The problem was that the public and media saw a perverse incentive for children to eat more chocolate, a product associated with obesity. Which of the following best summarizes Cadbury's problem? A) Customers felt that the cause was not in sync with the company's brand image. B) Consumers did not value the cause Cadbury was promoting. C) Customers questioned the link between the product and the cause and saw the firm as self-serving and exploitive. D) Consumers resented being sold an inferior product on the back of a cause-marketing program. E) Consumers felt that the campaign did not make a sufficient attempt to change the target audience's behavior.

C Page Ref: 637 Objective: 3 AACSB: Analytic skills Difficulty: Moderate

65) The sales gap due to reduced volume is ________. A) 0.4 percent B) 28.5 percent C) 71.4 percent D) 2.5 percent E) 63 percent

C Page Ref: 654 Objective: APP AACSB: Analytic skills Difficulty: Moderate

Which element of the marketing communications mix allows for amplified expressiveness? A) sales promotion B) public relations C) advertising D) direct and interactive marketing E) personal selling

C) Advertising

Which of the following marketing communications tools is most influential at the maturity stage of a product's life cycle? A) sales promotions B) direct marketing C) advertising D) publicity E) interactive marketing

C) Advertising

Personal influence in marketing communications carries great weight when the ________. A) product being marketed is a convenience item B) purchase of the product is considered to be safe and risk-free C) product suggests something about the user's status or taste D) product being marketed is purchased on a frequent basis E) product or service in questions is used without being recommended by others

C) Product suggests something about the user's status or taste.

92) Give an example of a cognitive social marketing campaign.

Cognitive campaigns try to educate and inform people. A cognitive campaign might explain the nutritional value of different foods or demonstrate the importance of conservation.

101) Which of the following consumer promotion tools offers a free amount of a product or service delivered door-to-door, sent in the mail, picked up in a store, attached to another product, or featured in an advertising offer? A) coupons B) rebates C) premiums D) samples E) price packs

D

20) Along which of the following parameters should marketers evaluate communication options when building brand equity? A) popularity B) innovativeness C) technological sophistication D) efficiency E) novelty

D

25) Companies that produce many products for many markets may adopt a ________ marketing organization. A) flat B) brand C) product D) matrix E) top-down

D

131) The elapsed time between an order's receipt, delivery, and payment is called the ________ cycle. A) variable-costs-to-payment B) product-to-payment C) inventory-to-sale D) order-to-inventory E) order-to-payment

E

143) The trucking firm hired by your transportation manager provides a trucking service between the city of Sacramento and the surrounding towns on a regular schedule and at fixed prices. The trucking firm saves transportation costs by transporting the goods using trains as well as trucks, instead of trucks alone. Your transportation manager has hired a(n) ________ carrier. A) airship B) airtruck C) trainship D) fishyback E) piggyback

E

38) Advertising and publicity tools play the most important roles in influencing buying decisions at the ________ stage of buyer readiness. A) comprehension B) conviction C) ordering D) reordering E) awareness-building

E

6) The minimum price that most consumers would pay for a given product is known as the ________ price. A) everyday low B) usual discounted C) fair D) typical E) lower-bound

E Page Ref: 387 Objective: 1 Difficulty: Easy

111) Problem removal and normal depletion are examples of positively oriented relevant brand needs.

FALSE

22) The rapid diffusion of multipurpose smart phones, broadband and wireless Internet connections, and ad-skipping digital video recorders (DVRs) have augmented the effectiveness of the mass media.

FALSE

The stage in the new-product development process that occurs first and has a pass ratio of 1:4 is the ______ stage.

Idea Screening

When a company combines two product concepts or ideas to create a new offering, it is called _________.

Lateral Marketing

127) How would a merchant wholesaler operate?

Merchant wholesalers are independently owned businesses that take title to the merchandise they handle. They are full-service and limited-service jobbers, distributors, and mill supply houses.

The brainstorming technique that might ask a customer to think of a car, write it on a piece of paper, think of the next thought that comes up (say Mercedes), link it to car, thing of the next association (Germany), and do this with all associations that come up with each new word it called __________.

Mind Mapping

143) In oligopolistic industries, all firms normally charge the same price. What kind of a pricing method are they said to be following?

Oligopolistic industries follow going-rate pricing. Firms following this pricing method, base their prices largely on competitors' prices. Page Ref: 401 Objective: 2 Difficulty: Moderate

Gabrielle is the chief marketing officer of Boyd Pharmaceuticals. She is meeting with Trent, the chief financial officer to decide on the company's marketing communications budget. After extensive discussions, they decide that the size of the budget will be calculated as a fraction of the overall turnover. What method did Gabrielle and Trent use to arrive at the marketing communications budget? A) affordable method B) objective-and-task method C) competitive-parity method D) activity-based method E) percentage-of-sales method

Percentage-of-Sales method

New-product ideas have a process that is not always linear. Many firms use a(n) ________ process that recognizes the value of returning to an earlier stage to make improvements before moving forward.

Spiral Development

54) Sandy's Stores is a small chain of grocery stores located in a few neighboring towns. The stores have always been largely self-service, but the company is considering making a switch to full-service stores. Offer one possible reason why this move is justified.

Student answers will vary. Full-service retailing is preferred by customers who like to be waited upon. If most of Sandy's customers show a preference for being waited upon, this can justify the costs of full-service retailing. If Sandy's has a number of products or services for which customers need information or guidance, or stocks high-value or specialized product, full-service retailing will be justified.

111) Supermarket chain Reynold's is considering stocking a number of private-label products in order to offer customers the lowest possible prices. Explain how this strategy could benefit the company.

Student answers will vary. Private brands can be more profitable for Reynold's if it searches for manufacturers with excess capacity that will produce private label goods at low cost. Other costs, such as research and development, advertising, sales promotion, and physical distribution, are also much lower, so private labels can generate a higher profit margin. Exclusive store brands can also help Reynold's differentiate itself from competitors. Many price-sensitive consumers prefer store brands in certain categories. These preferences could give Reynold's increased bargaining power with marketers of national brands.

99) Give one example of a retailer that uses standalone stores as opposed to stores located in malls. Why do you think retailers opt for this channel?

Student answers will vary. Retailers may opt for standalone stores so that they are not associated with other retailers. It can avoid competition from conflicting attractions in a mall. Customers will be less motivated to check other options available at other retailers. Standalone stores may give a retailer an upscale image and prevent dilution of the brand identity.

86) Identify three key success factors in developing and implementing a social marketing program.

Students may choose three of the following key success factors presented in the text: • choose target markets that are most ready to respond • promote a single, doable behavior in clear, simple terms • explain the benefits in compelling terms • make it easy to adopt the behavior • develop attention-grabbing messages and media • consider an education-entertainment approach

148) Media coordination can occur across and within media types, but marketers should combine personal and nonpersonal communications channels through multiple-vehicle, multiple-stage campaigns to achieve maximum impact and increase message reach and impact.

TRUE

35) The flow of materials to refurbish products for resale constitutes a reverse flow.

TRUE

36) Because the retail trade tends to think of profitability in terms of product categories, some companies are switching to a category management organizational model.

TRUE

60) Add-on services such as credit, delivery, installation, and repairs provided by the marketing channel are referred to as service backup.

TRUE

71) Often the more committed a company is to sustainability and environmental protection, the more dilemmas that can arise.

TRUE

73) Ravake is a manufacturer of high-end designer apparel. Competition in the apparel industry is severe and the market is driven by price. What distribution strategy would be best suited to help Ravake obtain an edge over its competitors?

The company can use exclusive distribution. It severely limits the number of intermediaries and reduces costs. This would help the company maintain greater control over service levels.

142) A toaster manufacturer who has invested $1 million in the business wants to set a price to earn a 20 percent return on investment, specifically $200,000. What pricing method should it choose?

The toaster manufacture should go for a target-return pricing. While using this pricing method, companies determine the price that yield its target rate of return on investment. Page Ref: 397 Objective: 2 Difficulty: Moderate

26) Which of the following allows a firm to maintain control over the service level and obtain more dedicated and knowledgeable selling? A) selective distribution B) intensive distribution C) push strategy D) exclusive distribution E) pull strategy

d

29) What are the four types of advertising timing patterns available to marketers when launching a new product?

In launching a new product, the advertiser must choose among continuity, concentration, flighting, and pulsing. • Continuity means exposures appear evenly throughout a given period. Generally, advertisers use continuous advertising in expanding market situations, with frequently purchased items, and in tightly defined buyer categories. • Concentration calls for spending all the advertising dollars in a single period. This makes sense for products with one selling season or related holiday. • Flighting calls for advertising during a period, followed by a period with no advertising, followed by a second period of advertising activity. It is useful when funding is limited, the purchase cycle is relatively infrequent, or items are seasonal. • Pulsing is continuous advertising at low-weight levels, reinforced periodically by waves of heavier activity. It draws on the strength of continuous advertising and flights to create a compromise scheduling strategy. Those who favor pulsing believe the audience will learn the message more thoroughly, and at a lower cost to the firm.

141) What should a company do if its competitor's product contains some features that are not available in its product?

In such a situation, the company should subtract the value of those features from the price of its product. Page Ref: 395 Objective: 2 Difficulty: Moderate

53) How do the stages in a product's life cycle influence the marketing communications mix?

In the introduction stage of the product life cycle, advertising, events and experiences, and publicity have the highest cost-effectiveness, followed by personal selling to gain distribution coverage and sales promotion and direct marketing to induce trial. In the growth stage, demand has its own momentum through word of mouth and interactive marketing. Advertising, events and experiences, and personal selling all become more important in the maturity stage. In the decline stage, sales promotion continues strong, other communication tools are reduced, and salespeople give the product only minimal attention.

138) How does American Express utilize m-commerce?

It runs a "Link-Like-Love" social commerce program, which sends cardmembers couponless personalized offers from merchants based on their Facebook "likes" and Facebook Places check-ins that are automatically redeemed through card use. Via a partnership with Foursquare, cardmembers could also automatically receive and redeem promotional offers from merchants based on their Foursquare activity.

157) What are the major functions performed by marketing public relations? Illustrate with examples.

MPR goes beyond simple publicity and plays an important role in the following tasks: • Launching new products: The amazing commercial success of toys such as LeapFrog, Beanie Babies, and even the latest kids' craze, Silly Bandz, owes a great deal to strong publicity. • Repositioning a mature product: In a classic PR case study, New York City had extremely bad press in the 1970s until the "I Love New York" campaign. • Building interest in a product category: Companies and trade associations have used MPR to rebuild interest in declining commodities such as eggs, milk, beef, and potatoes and to expand consumption of such products as tea, pork, and orange juice. • Influencing specific target groups: McDonald's sponsors special neighborhood events in Latino and African American communities to build goodwill. • Defending products that have encountered public problems: PR professionals must be adept at managing crises, such as those weathered by such well-established brands as Tylenol, Toyota, and BP in 2010. • Building the corporate image in a way that reflects favorably on its products: Steve Jobs's heavily anticipated Macworld keynote speeches have helped to create an innovative, iconoclastic image for Apple Corporation.

145) What is marketing control? List the four types of marketing control.

Marketing control is the process by which firms assess the effects of their marketing activities and programs and make necessary changes and adjustments. The four types of needed marketing control are: annual-plan control, profitability control, efficiency control, and strategic control.

89) Corporate philanthropy can pose problems even when done with the best intentions. Explain.

Philanthropic efforts by companies can be overlooked — even resented — if the company is seen as exploitive or fails to live up to a "good guys" image.

55) Briefly describe the process of measuring the results of marketing communications.

Senior managers want to know the outcomes and revenues resulting from their communications investments. Too often, however, their communications directors supply only inputs and expenses: press clipping counts, numbers of ads placed, media costs. In fairness, communications directors try to translate inputs into intermediate outputs such as reach and frequency (the percentage of target market exposed to a communication and the number of exposures), recall and recognition scores, persuasion changes, and cost-per-thousand calculations. Ultimately, behavior-change measures capture the real payoff. After implementing the communications plan, the communications director must measure its impact. Members of the target audience are asked whether they recognize or recall the message, how many times they saw it, what points they recall, how they felt about the message, and what their previous and current attitudes are toward the product and the company. The communicator should also collect behavioral measures of audience response, such as how many people bought the product, liked it, and talked to others about it.

137) When Sony introduced the world's first high-definition television to the Japanese market in 1990, it was priced at $43,000. This helped Sony to scoop the maximum amount of revenue from the various segments of the market. The price dropped steadily through the years—a 28-inch Sony HDTV cost just over $6,000 in 1993, but a 40-inch Sony HDTV cost only $600 in 2010. What pricing strategy did Sony use here?

Sony used a market-skimming pricing strategy. This is a favorite for companies unveiling a new technology. Companies using this introduce their product at a high price and slowly drop the price over time. Page Ref: 390 Objective: 2 Difficulty: Moderate

Back-to-school sales in August are an example of _____________.

Special-Event Pricing

71) HCN is a wholesaler that supplies consumer good products to a number of retailers. HCN has witnessed the worrying trend of its competitors losing out on suppliers who approach retailers directly. How can HCN strengthen its relationship with manufacturers and prevent this?

Student answers will vary. HCN can work to reach a clear agreement with their manufacturers about their expected functions in the marketing channel. It can gain insight into the manufacturers' requirements by visiting their plants and attending manufacturer association conventions and trade shows. HCN must also fulfill its commitments to the manufacturer by meeting the volume targets, paying bills promptly, and feeding back customer information to the manufacturers. It can also identify and offer value-added services, such as financing, on-site inventory management, parts-tracking software, and chip programming, to help its suppliers.

126) Reynold's, a supermarket chain, carries the K-Nine range of dog food manufactured by JGB. However, the chain does not interact directly with JGB, but obtains stocks from wholesalers. It has been suggested that Reynold's save costs by sourcing products directly from JGB. However, Mal, the CEO of Reynold's, insists that wholesalers are the most hassle-free option for Reynold's. What can Mal say to justify this?

Student answers will vary. Mal can justify this using any or all of the following reasons: • Wholesalers are able to select items and build the assortments Reynold's needs, saving Reynold's considerable work. • Wholesalers achieve savings for Reynold's by buying large carload lots and breaking the bulk into smaller units. • Wholesalers hold inventories, thereby reducing inventory costs and risks for Reynold's, who needn't hold the inventory for itself. • Wholesalers finance Reynold's by granting credit. • Wholesalers supply information regarding competitors' activities, new products, price developments, and so on.

132) Marco is working on promoting his company's Glazer brand of electronic razors. Preliminary surveys have revealed that even though a sizable portion of the target market has developed a liking for the product due to innovative advertising, few customers would actually consider replacing their current razors with Glazers. How can Marco modify the communications program to get customers to favor Glazers over other brands?

Student answers will vary. Since customers are comfortable with their present electronic razors, Marco's communications strategy should aim to build preference for Glazers. Customers have already developed a liking for Glazers. So, Marco should focus on comparing the quality of Glazers with that of competing brands. This can be done by illustrating the improved features of Glazers that provide clean and hassle-free use. Marco could do well to compare the Glazers with other brands in terms of value, utility, and performance factors as well.

118) If messages are too conflicting, audiences will counterargue and disbelieve them.

TRUE

130) ________ encompass sales forecasting, production planning, and inbound materials transportation. A) Market logistics B) Containerization C) Transportation D) Nonstore retailing E) Wholesaling

A

134) As inventory draws down, management must know at what stock level to request additional stock. This stock level is called the ________. A) reorder point B) least fixed point C) point of divergence D) inflection point E) critical point

A

140) An item described as low-risk and low-opportunity is a ________. A) nuisance item B) bottleneck item C) variable item D) critical item E) commodity

A

76) Because of the acceptance that the other Broomer products have in the market, retailers are willing to stock items from the new "Inducer" line of clothing. This is an example of ________ power. A) referent B) passive C) legitimate D) coercive E) reward

A

80) A manufacturer is using legitimate power when it ________. A) requests a behavior that is warranted under the selling contract B) threatens to withdraw a resource or terminate a relationship C) offers intermediaries an extra benefit for performing specific acts or functions D) makes the intermediaries sell more of a particular product by offering rewards E) sells more products by making use of its reputation in the market

A

81) Hewlett-Packard is a highly respected brand. Many retailers want to be associated with the brand because of this reputation. What kind of power does Hewlett-Packard obtain due to this reputation? A) referent B) functional C) legitimate D) coercive E) reward

A

84) Who will most likely be willing to pay for high-value-added channels? A) early buyers of a product B) internal customers of a company C) small and matured buyers of an industry D) consumers of low involvement products E) repeat customers of a product

A

126) What is a marketing audit? Explain the four characteristics of a marketing audit.

A marketing audit is a comprehensive, systematic, independent, and periodic examination of a company's or business unit's marketing environment, objectives, strategies, and activities, with a view to determining problem areas and opportunities and recommending a plan of action to improve the company's marketing performance. A marketing audit should be: (1) comprehensive it covers all the major marketing activities of a business; (2) systematic it is an orderly examination of the organization's macro- and micromarketing environments, marketing objectives and strategies, marketing systems, and specific activities; (3) independent outside consultants bring the necessary objectivity, broad experience in a number of industries, familiarity with the industry being audited, and undivided time and attention; and (4) periodic firms typically initiate marketing audits only after failing to review their marketing operations during good times, with resulting problems. A periodic marketing audit can benefit companies in good health as well as those in trouble. Page Ref: 645 Objective: 5 Difficulty: Moderate

Tracy works for a cellular phone company that offers trade-in allowances toward the upgrade of new phones, which is an example of ________. A) a sales promotion B) interactive marketing C) word-of-mouth marketing D) personal selling E) events and experiences

A) a sales promotion

85) Identify some of the brand benefits that can accrue to a company that engages in cause marketing.

A successful cause-marketing program can improve social welfare, create differentiated brand positioning, build strong consumer bonds, enhance the company's public image, create a reservoir of goodwill, boost internal morale and galvanize employees, drive sales, and increase the firm's market value. Consumers may develop a strong, unique bond with the firm running the cause-marketing that transcends normal marketplace transactions. Specifically, cause marketing can build brand awareness, enhance brand image, establish brand credibility, evoke brand feelings, create a sense of brand community, and elicit brand engagement.

Which of the following is an advantage of using the percentage-of-sales method to determine the marketing communications budget? A) The percentage-of-sales method encourages stability when competing firms spend approximately the same portion of their sales on communications. B) The percentage-of-sales method views sales as the determiner of communications rather than as the result. C) The percentage-of-sales method leads to a budget set by market opportunities rather than the availability of funds. D) The percentage-of-sales method encourages experimentation with countercyclical communication or aggressive spending. E) The percentage-of-sales method encourages building the communication budget by determining what each product and territory deserves.

A) The percentage-of-sales method encourages stability when competing firms spend approximately the same portion of their sales on communications.

125) The cost of land where a plant that manufactures common parts is set up falls under ________ costs. A) traceable common B) nontraceable common C) variable D) manufacturing E) material

B

120) Dominant brands offer sales promotion deals less frequently because most deals subsidize only current users.

TRUE

144) If the shipper owns its own truck or air fleet, it becomes a ________ carrier. A) containerized B) private C) contract D) common E) diversified

B

121) Wholesaling includes all the activities in selling goods or services to those who buy for resale or business use.

TRUE

64) Consumers now receive sales offers through direct-mail letters and catalogs, television, cell phones, and the Internet. The nonstore-based retailers are taking business away from store-based retailers.

TRUE

86) After a company has chosen a channel system, it must select, train, motivate, and evaluate individual intermediaries for each channel.

TRUE

85) A(n) ________ appeal is a creative strategy that elaborates on product or service attributes or benefits. A) aesthetic B) informational C) bandwagon D) emotional E) transformational

B

147) The degree to which the beneficial results of an innovation's use are observable or describable to others is called ________. A) divisibility B) communicability C) compatibility D) relative advantage E) plainness

B Diff: 2 LO: 15.8: What factors affect the rate of diffusion and consumer adoption of newly launched products and services? AACSB: Reflective thinking

19) Companies that sell products door-to-door or at home sales parties are engaging in ________. A) franchising B) network marketing C) direct-response marketing D) corporate selling E) direct marketing

B Diff: 2 LO: 18.1: What major types of marketing intermediaries occupy this sector? AACSB: Analytical thinking; Application of knowledge

143) Briefly explain the concept of annual-plan control.

Annual-plan control ensures the company achieves the sales, profits, and other goals established in its annual plan. At its heart is management by objectives. First, management sets monthly or quarterly goals. Second, it monitors performance in the marketplace. Third, management determines the causes of serious performance deviations. Fourth, it takes corrective action to close gaps between goals and performance. This control model applies to all levels of the organization. Top management sets annual sales and profit goals; each product manager, regional district manager, sales manager, and sales rep is committed to attaining specified levels of sales and costs. Each period, top management reviews and interprets the results. Marketers today have better marketing metrics for measuring the performance of marketing plans. Four tools for the purpose are sales analysis, market share analysis, marketing expense-to-sales analysis, and financial analysis.

102) The most advanced supply-distributor arrangements for ________ vertical marketing systems rely on distribution programming. A) corporate B) administered C) contractual D) regulatory E) controlled

B

105) A(n) ________ VMS consists of independent firms at different levels of production and distribution, integrating their programs on a contractual basis to obtain more economies or sales impact than they could achieve alone. A) administered B) contractual C) corporate D) regulated E) referent

B

119) If Ming was interested in capturing distribution metrics for her retail organization, which of the following might she consider? A) customer complaints B) share of shelf C) effective reach D) trial rate E) market share

B

62) The popular music talent show American Idol has been generally acknowledged as the most profitable TV series in US history, in terms of advertising and merchandising revenue. Major sponsors of the show include Coca-Cola, AT&T Wireless, and iTunes, among others. Cups bearing the Coca-Cola logo were a prominent prop found on the show's judges' tables. The show also urged viewers to vote for contestants using AT&T's sms service. Contestants were routinely shown rehearsing for their performances with the help of Apple iPods. Which of the following advertising practices is apparent in this example? A) ambush advertising B) product placement C) angel dusting D) co-branding E) subliminal advertising

B

63) Which of the following represents the objective of a social marketing campaign aimed at changing people's values? A) Motivate people with obesity to eat healthy and exercise more often. B) Change public attitudes and stereotypes associated with people who are obese. C) Explain the different causes of obesity and how it can be prevented. D) Encourage people to participate in a walkathon aimed at promoting awareness about obesity. E) Help people with obesity to implement lifestyle changes.

B

64) ________ is the number of times the average buyer buys a product during the period. A) Buyer turnover B) Purchase frequency C) Customer retention rate D) Advertising impressions E) Total number of exposures

B

9) Which of the following is true for self-service retailing? A) They carry more shopping goods and services such as credit and merchandise-return privileges. B) This service model is favored by discount stores and customers who want to save money. C) It results in high staffing costs compared to other forms of retailing. D) Salespeople are ready to assist in every phase of the "locate-compare-select" process. E) Customers need more information and assistance than in other forms of retailing.

B

A) expresses the company's sales as a percentage of total market sales B) is sales as a percentage of the total sales to the market C) is market share in relationship to the largest competitor D) is always smaller than overall market share E) measures the relative contribution of different factors to a gap in sales performance

B

133) A(n) ________ is any good, service, or idea that is perceived as new, no matter how long its history. A) commodity B) innovation C) adoption D) invention E) novel product

B Diff: 1 LO: 15.8: What factors affect the rate of diffusion and consumer adoption of newly launched products and services? AACSB: Reflective thinking

12) ________ teams are cross-functional groups charged with developing a specific product or business. A) Virtual B) Venture C) Fundamental D) Elemental E) Transitory

B Page Ref: 574 Objective: 2 Difficulty: Easy

Which of the following is NOT true about the two-step process by which mass communications affect personal attitudes? A) The influence of mass media is mediated by opinion leaders, people whose opinions others seek or who carry their opinions to others. B) The influence of mass media on public opinion is not as direct, powerful, and automatic as marketers have supposed. C) The two-step flow supports the notion that consumption styles are primarily influenced by a trickle-up or trickle-down effect from mass media. D) People interact primarily within their own social groups and acquire ideas from opinion leaders in their groups. E) Two-step communication suggests that mass communicators should direct messages specifically to opinion leaders and let them carry the message to others.

C) The two-step flow supports the notion that consumption styles are primarily influenced by a trickle-up or trickle-down effect from mass media

Jake wants to open a Subway franchise in his small town. To do this, he must pay the company a ________ fee. A) slotting B) title C) royalty D) merchandising E) residual

C) royalty

134) Define cause-related marketing. What is the difference between cause-related marketing and social marketing?

Cause-related marketing links the firm's contributions to a designated cause to customers' engaging directly or indirectly in revenue-producing transactions with the firm. Cause-related marketing supports a cause whereas social marketing by nonprofit or government organizations furthers a cause. Page Ref: 636; 640 Objective: 3 AACSB: Ethical understanding and reasoning abilities Difficulty: Moderate

115) Retailer cooperatives allow nonmember retailers to buy through them and share the profits.

FALSE

52) How do the stages of buyer readiness affect the composition of a product's marketing communications mix?

Communication tools vary in cost-effectiveness at different stages of buyer readiness. Advertising and publicity play the most important roles in the awareness-building stage. Customer comprehension is primarily affected by advertising and personal selling. Customer conviction is influenced mostly by personal selling. Closing the sale is influenced mostly by personal selling and sales promotion. Reordering is also affected mostly by personal selling and sales promotion, and somewhat by reminder advertising.

125) Briefly explain the three choices available to companies when deciding the timing of market entry.

Companies face three choices when deciding the timing. (1) First entry: The first firm entering a market usually enjoys the "first mover advantages" of locking up key distributors and customers and gaining leadership. But if rushed to market before it has been thoroughly debugged, the first entry can backfire. (2) Parallel entry: The firm might time its entry to coincide with the competitor's entry. The market may pay more attention when two companies are advertising the new product. (3) Late entry: The firm might delay its launch until after the competitor has borne the cost of educating the market, and its product may reveal flaws the late entrant can avoid. The late entrant can also learn the size of the market. Page Ref: 590 Objective: 4 Difficulty: Easy

131) How can firms promote ethical behavior among their employees?

Companies must adopt and disseminate a written code of ethics, build a company tradition of ethical behavior, and hold their people fully responsible for observing ethical and legal guidelines. Organizations must also ensure that every employee knows and observes relevant laws. Page Ref: 633 Objective: 3 AACSB: Ethical understanding and reasoning abilities Difficulty: Easy

123) What is concept testing? What is its importance? Explain two modern techniques used for concept testing.

Concept testing means presenting the product concept to target consumers, physically or symbolically, and getting their reactions. The more the tested concepts resemble the final product or experience, the more dependable concept testing is. Concept testing of prototypes can help avoid costly mistakes, but it may be especially challenging with radically different, new-to-the-world products. Rapid prototyping can be used to design products on a computer and then produce rough models to show potential consumers for their reactions. Companies are also using virtual reality to test product concepts. Virtual reality programs use computers and sensory devices (such as gloves or goggles) to simulate reality. Page Ref: 582 Objective: 4 Difficulty: Easy

49) Rising customer expectations, evolving employee goals and ambitions, and tighter government legislation and pressure are driving companies to ________. A) operate leaner manufacturing facilities B) manage shorter supply chains C) operate flatter organizations D) practice a higher level of corporate social responsibility E) vertically integrate

D

54) ________ call(s) for the producer to establish a schedule of discounts and allowances that intermediaries see as equitable and sufficient. A) Exclusive dealings B) Mutual services C) Territorial rights D) Price policy E) Tying agreements

D

60) Staples is a giant retailer that concentrates on selling office supplies. Staples is an example of a(n) ________. A) ambush marketer B) supercenter C) megamarketer D) category killer E) guerilla marketer

D

62) Which of the following steps in the innovation-adoption model of marketing communications corresponds to the cognitive stage that a buyer passes through? A) interest B) evaluation C) trial D) awareness E) adoption

D

98) Which of the following statements is true of the two-step approach to mass communications? A) The influence of mass media on public opinion is more direct, powerful, and automatic than marketers have supposed. B) Communications through mass media bypasses opinion leaders and reaches the individual buyers. C) The two-step flow supports the notion that consumption styles are primarily influenced by a "trickle-down" or "trickle-up" effect from mass media. D) According to the two-step flow, people interact primarily within their own social groups and acquire ideas from opinion leaders in their groups. E) Two-step communication suggests that mass communicators should direct messages to groups of buyers who interpret the message and act accordingly.

D

10) ________ is an element of the marketing communications mix that involves online activities and programs designed to engage customers or prospects and directly or indirectly raise awareness, improve image, or elicit sales of products and services. A) Personal selling B) Direct marketing C) Sales promotion D) Interactive marketing E) Public relations

D

106) Which of the following consumer promotion tools involves using one brand to advertise another noncompeting brand? A) tie-in promotion B) frequency programs C) specialty advertising D) cross-promotion E) patronage awards

D

117) The owner of supermarket chain Reynold's has realized that customers want a wider variety of goods than is currently available. However, Reynold's cannot afford the costs of storing excess inventory. Additionally, the owner is not willing to take the risk that the new products will remain unsold. Which of the following types of wholesalers can help Reynold's meet customer demand while minimizing costs? A) producers' cooperatives B) cash and carry wholesalers C) truck wholesalers D) drop shippers E) rack jobbers

E

122) Annual-plan control requires making sure the company isn't overspending to achieve sales goals. The key ratio to watch is ________. A) stock turnover B) gross margin C) return on capital D) cash flow return on investment E) marketing expense-to-sales

E

14) An example of a restaurant with a narrow and deep assortment is a ________. A) small lunch counter B) cafeteria C) large restaurant D) casual dining restaurant chain E) delicatessen

E

142) Which of the following functions of public relations departments involves presenting news and information about the organization in the most positive light? A) corporate communications B) product publicity C) lobbying D) counseling E) press relations

E

A) breakfast cereal B) detergents C) beer D) electric bulbs E) refrigerator

E

Which of the following is least likely to help a retailer develop its​ product-differentiation strategy? A. Feature the latest or newest merchandise first. B. Feature exclusive national brands that are not available at competing retailers. C. Feature blockbuster distinctive merchandise events. D. Offer​ merchandise-customizing services. E. Feature mostly​ manufacturer-brand merchandise.

E

________ increases at an accelerating rate as the​ customer-service level approaches​ 100%. A. Running cost B. Setup cost C. Obsolescence cost D. ​Order-processing cost E. Inventory cost

E

________ was originally pioneered by Japanese firms such as Toyota to produce goods with minimal waste of​ time, materials, and money. A. ​Supply-chain management B. Electronic data interchange​ (EDI) C. Market logistics D. Electronic funds transfer​ (EFT) E. Lean manufacturing

E

2) Which of the following is true for retailing? A) Manufacturers are not considered to be retailers as they are engaged in producing the product. B) Vending machines are considered to be retailing only if they are located within stores. C) Retailing deals only with goods; it does not include services. D) Selling from a consumer's home is direct selling, but not retailing. E) Wholesalers are only considered to be retailers if they are selling to final consumers.

E Diff: 1 LO: 18.1: What major types of marketing intermediaries occupy this sector? AACSB: Analytical thinking

72) What are the characteristics of an ideal ad campaign?

In an ideal ad campaign, the right consumer is exposed to the right message at the right place and at the right time. The consumer is not distracted from the intended message. The ad properly reflects the consumer's level of understanding of and behaviors with the product and the brand. The ad correctly positions the brand in terms of desirable and deliverable points-of-difference and points-of-parity. The ad motivates consumers to consider purchase of the brand.

124) Explain the concepts of sales-wave research and simulated test marketing.

In sales-wave research, consumers who initially try the product at no cost are reoffered it, or a competitor's product, at slightly reduced prices. The offer may be made as many as five times (sales waves), while the company notes how many customers select it again and their reported level of satisfaction. During simulated test marketing, thirty to forty qualified shoppers are asked about brand familiarity and preferences in a specific product category and attend a brief screening of both well-known and new TV commercials and print ads. One ad advertises the new product but is not singled out for attention. Consumers receive a small amount of money and are invited into a store where they may buy any items. Page Ref: 588 Objective: 4 Difficulty: Easy

110) The marketing communications objective of "brand attitude" refers to helping consumers evaluate the brand's perceived ability to meet a currently relevant need.

TRUE

112) Vertical marketing systems achieve economies through size, bargaining power, and elimination of duplicated services.

TRUE

132) Interno Computers Inc., together with subsidiaries, designs, manufactures, and markets personal computers and mobile communication and media devices, as well as selling related software, services, peripherals, networking solutions, and applications worldwide. The company is planning to launch a high-end portable digital music player worldwide. What is the best way to organize this product launch?

Large companies, like Interno, often establish a new-product department headed by a manager with substantial authority and access to top management whose responsibilities include generating and screening new ideas, working with the R&D department, and carrying out field testing and commercialization. This model is necessary here as the product is going to be launched worldwide. Page Ref: 574 Objective: 2 AACSB: Analytic skills Difficulty: Moderate

150) What is integrated marketing communications? What is its significance in the current marketing environment?

Many companies still rely on only one or two communication tools. This practice persists in spite of the fragmenting of mass markets into a multitude of minimarkets, each requiring its own approach; the proliferation of new types of media; and the growing sophistication of consumers. The wide range of communication tools, messages, and audiences makes it imperative that companies move toward integrated marketing communications. Companies must adopt a "360-degree view" of consumers to fully understand all the different ways that communications can affect consumer behavior in their daily lives. The American Marketing Association defines integrated marketing communications (IMC) as "a planning process designed to assure that all brand contacts received by a customer or prospect for a product, service, or organization are relevant to that person and consistent over time." This planning process evaluates the strategic roles of a variety of communications disciplines — for example, general advertising, direct response, sales promotion, and public relations — and skillfully combines these disciplines to provide clarity, consistency, and maximum impact through the seamless integration of messages.

16) What are marketing channels? Briefly explain some of the different types of intermediaries.

Marketing channels are sets of interdependent organizations participating in the process of making a product or service available for use or consumption. They are the set of pathways a product or service follows after production, culminating in purchase and consumption by the final end user. Some intermediaries — such as wholesalers and retailers — buy, take title to, and resell the merchandise; they are called merchants. Others — brokers, manufacturers' representatives, sales agents — search for customers and may negotiate on the producer's behalf but do not take title to the goods; they are called agents. Still others — transportation companies, independent warehouses, banks, advertising agencies — assist in the distribution process but neither take title to goods nor negotiate purchases or sales; they are called facilitators.

113) A corporate vertical marketing system combines successive stages of production and distribution under single ownership.

TRUE

113) Volkswagen's famed "Drivers Wanted" advertising campaign aimed at attracting active, youthful people uses a transformational appeal as its creative strategy.

TRUE

115) Sales promotions often attract brand switchers, who are primarily looking for low price, good value, or premiums.

TRUE

126) Compare and contrast advertising and sales promotion as marketing communication tools.

Sales promotion expenditures increased as a percentage of budget expenditure for a number of years, although its growth has recently slowed. Several factors contributed to this growth, particularly in consumer markets. Promotion became more accepted by top management as an effective sales tool, the number of brands increased, competitors used promotions frequently, many brands were seen as similar, consumers became more price-oriented, the trade demanded more deals from manufacturers, and advertising efficiency declined. But the rapid growth of sales promotion created clutter. Loyal brand buyers tend not to change their buying patterns as a result of competitive promotions. Advertising appears to be more effective at deepening brand loyalty, although we can distinguish added-value promotions from price promotions. Price promotions may not build permanent total-category volume. Small-share competitors may find it advantageous to use sales promotion because they cannot afford to match the market leaders' large advertising budgets, nor can they obtain shelf space without offering trade allowances or stimulate consumer trial without offering incentives. Dominant brands offer deals less frequently, because most deals subsidize only current users.

137) Horon Furniture is a manufacturer of office furniture. The company is known for its innovative multi-purpose furniture. When designing a new product, the designers list several ideas and consider each in relationship to others. For example, The company considers various options such as an office table, foldable chairs, and filing cabinet, etc. Finally the company may come up with a foldable office table with chairs attached and with a filing cabinet. Describe the creativity technique used at Horon Furniture.

The company uses forced relationship techniques. This method lists several ideas and considers each in relationship to each of the others. Page Ref: 579 Objective: 3 AACSB: Analytic skills Difficulty: Moderate

136) When Carl's company introduced its new product in the market, it introduced it at the lowest possible price assuming that the demand for the product is going to be highly responsive to the price it is being introduced at. It also believes that a higher sales volume will lead to lower unit costs and higher long-run profit. What can be said about the company's objective?

The company's objective is to maximize its market share. Page Ref: 389 Objective: 2 AACSB: Analytic skills Difficulty: Moderate

131) Dormentor, Inc. produces converts and markets packaging products including boxboard, container board and numerous other specialty packaging products. In an attempt to increase its organic growth, the company decides to introduce new products. It asks the managers and employees to send in ideas for new products. Before launching products based on any of these ideas, what are the stages that it has to go through?

The following are the steps that Dormentor has to go through before launching: (1) It must screen the ideas that are obtained. (2) The ideas that pass the screening should be concept tested. (3) After concept testing a few are screened out and the remaining should be placed for product development. (4) After developing the product, product has to be test marketed. Ideas that are successfully test marketed can be launched. Page Ref: 573 Objective: 2 AACSB: Analytic skills Difficulty: Moderate

139) What is geofencing? Provide an example.

The idea of geofencing is to target customers with a mobile promotion when they are within a defined geographical space, typically near or in a store. The local-based service requires just an app and GPS coordinates, but consumers have to opt in. Neiman-Marcus is piloting geofencing in its stores so its salespeople know when their more valuable customers are on the premises and can look at their purchase history to provide more personalized service.

149) Movie matinees are priced lower than the evening shows; television advertising costs less when run after midnight. These are examples of what type of price discrimination?

These are examples of time pricing or price discrimination based on time. Page Ref: 406 Objective: 3 AACSB: Analytic skills Difficulty: Moderate

95) List some differentiation strategies retailers can use to compete successfully in the market.

To better differentiate themselves and generate consumer interest, retailers can use the following strategies: • feature exclusive national brands that are not available at competing retailers • feature mostly private-label merchandise • feature blockbuster distinctive merchandise events • feature surprise or ever-changing merchandise • feature the latest or newest merchandise first • offer merchandise-customizing services • offer a highly targeted assortment

75) JSE Securities Exchange is the largest stock exchange in Africa. The JSE provides a market where securities can be traded freely under a regulated procedure. The company acts as an intermediary between the traders. JSE is an example of a(n) ________. A) internal broker B) infomediary C) customer community D) market maker E) third party arbitrator

d

provided by marketing channels with reference to Orion.

• Lot size — Customers may be allowed to book as many tickets as they want, subject to availability. • Waiting and delivery time — This would refer to the average time customers take to book their tickets. • Spatial convenience — This refers to the ease of booking tickets. The location of Orion's retail ticketing outlets and whether the company offers online ticket booking services would be classified under spatial convenience. • Product variety — This refers to the assortment provided by the marketing channel in terms of travel destinations, holiday packages, hotel bookings, car rental services, and the like. • Service backup — This refers to the add-on services provided by the channel such as credit, discounts, delivery, refunds, and so on.

39) Dell computers is a manufacturer of computers. Dell accepts orders for computers online and ships the products to the customer. Explain the likely physical flow of materials in this case.

Suppliers to Transporters to Warehouses to Dell to Transporters to Customers

86) Concentrated advertising is well suited for products with one selling season or related holiday.

TRUE

82) Most established companies focus on incremental innovation rather than radical innovation.

TRUE Page Ref: 571 Objective: 1 Difficulty: Easy

89) A spiral development process recognizes the value of returning to an earlier stage to make improvements before moving forward.

TRUE Page Ref: 575 Objective: 2 Difficulty: Easy

91) The strategic blueprint thinks about how the new product can fit into customers' lives and how it can be distinguished from competitors.

TRUE Page Ref: 576 Objective: 3 Difficulty: Easy

81) There are three types of product-teams structures: vertical, triangular, and horizontal.

TRUE Page Ref: 628 Objective: 2 Difficulty: Easy

95) Corporate philanthropy as a whole is on the rise.

TRUE Page Ref: 636 Objective: 3 AACSB: Ethical understanding and reasoning abilities Difficulty: Easy

71) Briefly describe the macromodel of the marketing communications process.

The macromodel of the communications process has nine key factors in effective communication. Two represent the major parties — sender and receiver. Two represent the major tools — message and media. Four represent major communication functions — encoding, decoding, response, and feedback. The last element in the system is noise, random and competing messages that may interfere with the intended communication. Senders must know what audiences they want to reach and what responses they want to get. They must encode their messages so the target audience can decode them. They must transmit the message through media that reach the target audience and develop feedback channels to monitor the responses. The more the sender's field of experience overlaps that of the receiver, the more effective the message is likely to be.

45) Many companies are beginning to realize that they are not really market and customer driven, they are product and sales driven. In the attempt to transform themselves into true market-driven companies, many firms must change. Describe and explain what changes are necessary.

To be truly market-driven, companies need to develop a company-wide passion for customers, organize around customer segments instead of around products, and develop a deep understanding of customers through qualitative and quantitative research. Additionally, the organization must be creative; the firm must build capability in strategic innovation and imagination. This capability comes from assembling tools, processes, skills, and measures that let the firm generate more and better new ideas than its competitors.

131) When Abe goes shopping, he comes across a T-shirt that is priced at $35. Although he wants to buy it, judging from the material used, he feels that the T-shirt should only cost $20. What reference price is Abe using here?

$20 is what Abe perceives to be the "fair price" for the product. Page Ref: 387 Objective: 1 AACSB: Analytic skills Difficulty: Moderate

140) ________ channel conflict occurs between channel members at the same level. A) Horizontal B) Vertical C) Multichannel D) Administrative E) Contractual

A

17) Which of the following is an example of a word-of-mouth marketing communication platform? A) chat rooms B) billboards C) factory tours D) incentive programs E) trade shows

A

140) An item described as low-risk and low-opportunity is a ________. A) nuisance item B) bottleneck item C) variable item D) critical item E) commodity

A Diff: 2 LO: 18.6: What are some important issues in logistics? AACSB: Analytical thinking

29) Costs that do not vary with production levels or sales revenue are known as ________. A) overhead costs B) variable costs C) average costs D) opportunity costs E) total costs

A Page Ref: 393 Objective: 2 Difficulty: Easy

44) ________ pricing is a matter of reengineering the company's operations to become a low-cost producer without sacrificing quality. A) Value B) Going-rate C) Auction-type D) Markup E) Perceived-value

A Page Ref: 400 Objective: 2 Difficulty: Easy

51) In a(n) ________, the buyer announces something he or she wants to buy, and potential sellers compete to offer the lowest price. A) Dutch auction with one buyer and many sellers B) English auction with one buyer and many sellers C) English auction with one seller and many buyers D) sealed-bid auction E) ascending auction

A Page Ref: 402 Objective: 2 Difficulty: Easy

55) Armac Ltd., is a sluice-box manufacturer based in China. A sluice-box is used for gold prospecting. Armac is interested in selling a few of its machines to an American mining company, but it wants 95 percent of the machines' price in gold and the rest in ores recovered by using the machines. This is an example of a ________. A) buyback arrangement B) functional discount C) barter deal D) compensation deal E) sealed bid

A Page Ref: 404 Objective: 3 AACSB: Analytic skills Difficulty: Moderate

65) In second-degree price discrimination, the seller charges ________. A) less to buyers of larger volumes B) different prices depending on the season, day, or hour C) a separate price to each customer depending on the intensity of his or her demand D) different prices for different versions of the same product E) different prices for the same product depending on the channel through which it is sold

A Page Ref: 406 Objective: 3 Difficulty: Moderate

77) A company does not set a final price until the product is finished or delivered. This is known as ________. A) delayed quotation pricing B) an escalator clause C) special-event pricing D) time pricing E) the shallow-pockets trap

A Page Ref: 408 Objective: 4 Difficulty: Easy

6) Most established companies focus on ________ innovation when they aim to enter new markets by tweaking existing products, or they want to stay one step ahead in the market by using variations on a core product. A) incremental B) continuous C) spontaneous D) radical E) competitive

A Page Ref: 571 Objective: 1 Difficulty: Easy

9) Pager, a simple personal device for short messages, became famous in the 1990s. Troveron Communications launched a pager in the early twenty-first century. Due to the introduction of mobile phones and text messaging, the pager industry was on a decline. The company's innovations were not well received by the market and the product became a failure. Which of the following is the most likely reason for the product's failure in this case? A) poor launch timing of the product B) a small and fragmented target market C) high cost of development D) social and economic constraints E) hasty product development

A Page Ref: 572 Objective: 1 AACSB: Analytic skills Difficulty: Moderate

8) Which of the following strategies for new-product development incorporates the buyers' preferences in the final design of the product? A) customer-driven engineering B) market leadership C) cost leadership D) incremental innovation E) disruptive technology

A Page Ref: 572 Objective: 1 Difficulty: Easy

14) A team formed at Intercom, Inc. to generate ideas for new products, conducts frequent meetings and engages in activities such as mind mapping and brainstorming. Most of the meetings are conducted at informal locations away from office. These workplaces are called ________. A) skunkworks B) idea funnels C) research centers D) stage gate systems E) contextual bases

A Page Ref: 574 Objective: 2 AACSB: Analytic skills Difficulty: Moderate

19) ________ process recognizes the value of returning to an earlier stage to make improvements before moving forward. A) Spiral development B) Reactive development C) Market testing D) Proactive development E) Concept testing

A Page Ref: 575 Objective: 2 Difficulty: Easy

17) Explain the concept of a value network.

A company can be seen as being at the center of a value network, a system of partnerships and alliances that a firm creates to source, augment, and deliver its offerings. A value network includes a firm's suppliers and its suppliers' suppliers, and its immediate customers and their end customers. The value network includes valued relationships with others such as university researchers and government approval agencies. A company needs to orchestrate these parties in order to deliver superior value to the target market. Managing a value network means making increasing investments in information technology (IT) and software. Firms have introduced supply chain management (SCM) software and invited such software firms as SAP and Oracle to design comprehensive enterprise resource planning (ERP) systems to manage cash flow, manufacturing, human resources, purchasing, and other major functions within a unified framework.

136) What is the purpose of appointing an idea manager?

A company can motivate its employees to submit new ideas to an idea manager whose name and contact information are widely circulated. Page Ref: 578 Objective: 3 Difficulty: Easy

Which of the following types of retailing generally entails the highest costs? A) full-service B) self-service C) limited-selection D) limited service E) self-selection

A) full-service

An item described as low-risk and low-opportunity is a ________. A) nuisance item B) bottleneck item C) variable item D) critical item E) commodity

A) nuisance item

Creative strategies refer to the ________. A) way marketers translate their messages into a specific communication B) amount of creative content in a communications message C) degree of innovation involved in the marketing of a product D) novelty of a marketing communication E) type of medium used to deliver a marketing communication

A) way marketers translate their messages into a specific communication

1) An advertising ________ is a specific communications task and achievement level to be accomplished with a specific audience in a specific period of time. A) medium B) objective C) channel D) budget E) copy

B

22) Which of the following channel functions constitute only a backward flow? A) movement of physical goods B) placing orders with manufacturers C) persuasive communication D) storage of physical goods E) overseeing actual transfer of ownership

B

35) ________ includes all the activities in selling goods or services to those who buy for resale or business use. A) Retailing B) Wholesaling C) Procurement D) Promoting E) Warehousing

B

26) If demand changes considerably, with a small change in price, the demand is said to be ________. A) unit elastic B) elastic C) inelastic D) marginal E) strained

B Page Ref: 392 Objective: 2 Difficulty: Easy

36) Deducting the desired profit margin from the price at which a product will sell, given its appeal and competitors' prices, is known as ________. A) overhead costing B) target costing C) activity based costing D) benefit analysis E) estimate costing

B Page Ref: 394 Objective: 2 Difficulty: Easy

43) The key to perceived-value pricing is to ________. A) reengineer the company's operations B) deliver more unique value than competitors C) adopt subtle marketing tactics compared to competitors D) deliver more value but at a lower cost E) invest heavily in advertising in order to convey superior value

B Page Ref: 399 Objective: 2 Difficulty: Moderate

49) Which of the following auctions is characterized by one seller and many buyers? A) Walrasian auctions B) ascending bid auctions C) closed auctions D) sealed-bid auctions E) reverse auctions

B Page Ref: 402 Objective: 2 Difficulty: Easy

59) A(n) ________ is an extra payment designed to gain reseller participation in special programs. A) seasonal discount B) allowance C) discount D) quantity discount E) functional discount

B Page Ref: 404 Objective: 3 Difficulty: Easy

62) When supermarkets and department stores drop the price on well-known brands to stimulate store traffic, they are said to be following ________. A) value pricing B) loss-leader pricing C) special event pricing D) high-low pricing E) everyday low pricing

B Page Ref: 405 Objective: 3 Difficulty: Easy

13) Intercom, Inc. together with its subsidiaries, primarily engages in the generation, transmission, and distribution of electric power in the United States. The company observes that its growth has stagnated over a period of two years. In an attempt to promote growth, it considers adding new features to the existing products and introducing a few new products. The company forms a committee consisting of three top executives, one of the production mangers, a few operational managers, and a representative of the HR department to generate ideas. This team is called a(n) ________. A) virtual team B) venture team C) fundamental team D) elemental team E) transitory team

B Page Ref: 574 Objective: 2 AACSB: Analytic skills Difficulty: Moderate

40) Modern firms use the ________ concept testing tool to design products on a computer and then produce rough models to show potential consumers for their reactions. A) morphological analysis B) rapid prototyping C) concept testing D) perceptual mapping E) conjoint analysis

B Page Ref: 582 Objective: 4 Difficulty: Easy

43) With ________, respondents see different hypothetical offers formed by combining varying levels of the attributes, then rank the various offers. A) gap level analysis B) conjoint analysis C) perceptual mapping D) concept testing E) morphological analysis

B Page Ref: 583 Objective: 4 Difficulty: Easy

53) Which of the following methodologies takes the list of desired customer attributes (CAs) generated by market research and turns them into a list of engineering attributes (EAs) that engineers can use? A) quality control processes B) quality function deployment C) rapid prototyping D) marketing control E) control system formation

B Page Ref: 587 Objective: 4 Difficulty: Easy

59) Which of the following products is most likely to undergo alpha and beta testing? A) food products B) industrial goods C) consumer products D) commodities E) FMCG products

B Page Ref: 589 Objective: 4 Difficulty: Easy

73) Five characteristics influence the rate of adoption of an innovation. One of these is ________. A) marketing expertise B) relative advantage C) packaging attractiveness D) government regulations E) place of value exchange

B Page Ref: 592 Objective: 5 Difficulty: Easy

76) The degree to which the beneficial results of an innovation's use are observable or describable to others is called ________. A) divisibility B) communicability C) compatibility D) relative advantage E) plainness

B Page Ref: 592 Objective: 5 Difficulty: Easy

14) A company selling in a national market often organizes its sales force along ________. A) functional groups B) geographic lines C) product teams D) brand groups E) product categories

B Page Ref: 627 Objective: 2 Difficulty: Easy

Stacey wants to use a highly relevant, engaging, and implicit communications mix mode to use a "soft sell" approach for her new makeup line. The communications mix mode that has these characteristics is ________. A) sales promotions B) events and experiences C) advertising D) direct and interactive marketing E) personal selling

B) Events and Experiences

Which of the following factors found in the macromodel of the communications process refers to random and competing messages that may interfere with the intended communication? A) negative feedback B) noise C) attenuation D) phase lag E) selective distortion

B) Noise

The most effective communications mix tool at later stages of the buying process is ________ because it is particularly effective at building buyer preference, conviction, and action. A) sales promotions B) personal selling C) advertising D) direct and interactive marketing E) events

B) Personal Selling

Which of the following is true for self-service retailing? A) They carry more shopping goods and services such as credit and merchandise-return privileges. B) This service model is favored by discount stores and customers who want to save money. C) It results in high staffing costs compared to other forms of retailing. D) Salespeople are ready to assist in every phase of the "locate-compare-select" process. E) Customers need more information and assistance than in other forms of retailing.

B) This service model is favored by discount stores and customers who want to save money.

If the shipper owns its own truck or air fleet, it becomes a ________ carrier. A) containerized B) private C) contract D) common E) diversified

B) private

Agricultural assemblers, petroleum bulk plants and terminals, and auction companies are examples of ________. A) full-service wholesalers B) specialized wholesalers C) limited-service wholesalers D) merchant wholesalers E) brokers

B) specialized wholesalers

A ________ is a wholesaler-sponsored group of independent retailers engaged in bulk buying and common merchandising. A) retailer cooperative B) voluntary chain C) consumer cooperative D) merchandising conglomerate E) franchise organization

B) voluntary chain

136) Briefly describe "brand awareness" as an objective of marketing communications.

Brand awareness refers to fostering the consumer's ability to recognize or recall the brand within the category, in sufficient detail to make a purchase. Recognition is easier to achieve than recall. Brand recall is important outside the store; brand recognition is important inside the store. Brand awareness provides a foundation for brand equity.

142) What is the significance of performing business analysis?

Business analysis is performed to study a proposal's business attractiveness. Management prepares sales, cost, and profit projections to determine whether they satisfy company objectives. Page Ref: 585 Objective: 4 Difficulty: Easy

18) Avon, Tupperware, and Southwestern Company of Nashville are among companies in the multibillion-dollar ________ industry, which involves selling door-to-door or at home sales parties. A) direct marketing B) catalog showroom C) direct selling D) automatic vending E) buying services

C

2) ________ involves buying more goods and services from external domestic or foreign vendors. A) Merging B) Broadening C) Outsourcing D) Globalizing E) Accelerating

C

20) A product-management organization ________. A) often proves to be cost-effective B) simplifies the process of developing a national strategy C) focuses on building market share rather than customer relationships D) reduces an organization's staffing requirements E) allows product managers to achieve functional expertise

C

3) When Lola asked what the advertising campaign should say, she was concerned with which of the five Ms? A) Mission B) Money C) Message D) Media E) Measurement

C

Which of the following wholesaler functions reduces inventory costs and risks to suppliers and​ customers? A. Market information B. Transportation C. Warehousing D. Assortment building E. Selling and promoting

C

31) A ________ divides the innovation process into stages with a checkpoint at the end of each stage. A) reverse assumption analysis technique B) skunkworks system C) stage-gate system D) spiral development process E) new-product department technique

C Diff: 1 LO: 15.3: What organizational structures and processes do managers use to oversee new-product development? AACSB: Reflective thinking

119) Which of the following is true of brokers? A) Brokers represent buyers or sellers on a semipermanent basis. B) Most brokers are small businesses with a few skilled salespeople. C) Brokers bring buyers and sellers together and assist in negotiation. D) Selling brokers have contractual authority to sell a manufacturer's entire output. E) Purchasing brokers make purchases for buyers and often receive, inspect, warehouse, and ship merchandise.

C Diff: 2 LO: 18.5: What are some of the important issues in wholesaling? AACSB: Analytical thinking

116) ________ sell and deliver a limited line of semiperishable goods to supermarkets, grocery stores, hospitals, restaurants, and hotels. A) Producers' cooperatives B) Cash and carry wholesalers C) Truck wholesalers D) Drop shippers E) Rack jobbers

C Diff: 2 LO: 18.5: What are some of the important issues in wholesaling? AACSB: Reflective thinking

136) Optimal order quantities exist when the curves for the order-processing cost per unit and inventory-carrying cost per unit ________. A) are collinear B) are diagonal to each other C) intersect D) are parallel to each other E) equal zero

C Diff: 2 LO: 18.6: What are some important issues in logistics? AACSB: Analytical thinking

77) Which of the following terms refers to the degree to which the innovation can be tried on a limited basis? A) compatibility B) relative advantage C) divisibility D) communicability E) complexity

C Page Ref: 592 Objective: 5 Difficulty: Easy

21) One of the options in a product-management organization is to eliminate product manager positions for minor products and assign two or more products to each remaining manager. Under what conditions is this alternative feasible? A) when the product mix is highly diverse B) when there are very few products in the company's portfolio C) when two or more products serve a similar set of needs D) when customers fall into different user groups E) when the company produces many products for many markets

C Page Ref: 628 Objective: 2 Difficulty: Moderate

36) Social marketing programs designed to discourage cigarette smoking or excessive consumption of alcohol are examples of ________. A) cognitive campaigns B) active campaigns C) behavioral campaigns D) value campaigns E) normative campaigns

C Page Ref: 640 Objective: 3 AACSB: Ethical understanding and reasoning abilities Difficulty: Moderate

Which of the following is an example of a trade promotion? A) free samples B) discount coupons C) display allowances D) contests for sales reps E) premiums

C) Display Allowances

Which of the following is true for direct product profitability? A) It is highly correlated with the gross margin on a product. B) It is negligible compared to the gross margin on a product. C) It bears little relation to the gross margin on a product. D) It is significantly lower than the gross margin on a product. E) It is exactly the same as the gross margin on a product.

C) It bears little relation to the gross margin on a product.

Which of the following is an example of direct selling? A) E&OE sells its herbal skincare products exclusively through its standalone stores. B) TCJ is a telemarketing firm that sells products from a number of different suppliers. C) Jayne's sells most of its products to customers through home sales parties. D) J3 is an online shopping portal where customers can buy directly from manufacturers. E) Reynold's tries to minimize its staff costs by installing vending machines in its stores. Answer: C

C) Jayne's sells most of its products to customers through home sales parties.

E&OE is trying to minimize its inventory costs, which are extremely high. The company has realized that it can achieve this by maintaining a near-zero inventory and producing more products only once it is ordered. Which of the following is true for E&OE? A) Inventory costs are lower than order-processing costs. B) Running costs are higher than inventory-carrying costs. C) Setup costs for the products are low. D) Order-processing costs are high. E) Order-processing costs are lower than setup costs.

C) Setup costs for the products are low.

In ________ retailing, salespeople are ready to assist in every phase of the "locate-compare-select" process. A) self-service B) self-selection C) full-service D) limited service E) limited-selection

C) full-service

Reynold's is a grocery chain that has always catered to mid-market customers. However, the owner, Mal, has noticed that an influx of new residents are buying mostly the lower-cost and discounted products. To attract customers, Mal decides to make a gradual switch to the discount store format, but to do this, he will have to cut costs wherever possible. Which of the following types of services should Mal avoid in order to lower costs? A) limited service B) self-selection C) full-service D) self-service E) limited-selection

C) full-service

In addition to its store brands and nationally well-known brands of detergents, Reynold's also carries much cheaper varieties of detergents that are not advertised and have little-known names. They are often manufactured from lower-quality ingredients and save on packaging and advertising costs. These are known as ________. A) common carriers B) shills C) generics D) private labels E) marques

C) generics

Optimal order quantities exist when the curves for the order-processing cost per unit and inventory-carrying cost per unit ________. A) are collinear B) are diagonal to each other C) intersect D) are parallel to each other E) equal zero

C) intersect

135) Give an example of a cognitive social marketing campaign.

Cognitive campaigns try to educate and inform people. A cognitive campaign might explain the nutritional value of different foods or demonstrate the importance of conservation. Page Ref: 640 Objective: 3 AACSB: Ethical understanding and reasoning abilities Difficulty: Easy

34) Which of the following statements is true of the role of advertising in business markets? A) Advertising is unsuitable for explaining any new features that a product might have. B) Sales calls are more economical than reminder advertisements. C) Sales calls are more effective than advertisements at reminding customers how to use a product and reassure them about their purchase. D) Sales representatives can use copies of the company's ads to legitimize their company and products. E) Advertisements are the least preferred tools when intended to generate leads for sales representatives.

D

65) In which of the following types of advertising timing patterns do exposures appear evenly throughout a given period? A) concentration B) pulsing C) flighting D) continuity E) frequency capping

D

103) Rotter Garder Inc. is a large-scale paint manufacturer and is known for its wide range of decorative paint products and industrial coatings. In addition to making paints, the company also owns and operates the retail stores that sell its products. This is an example of a(n) ________ vertical marketing system. A) administered B) contractual C) referent D) corporate E) regulated

D

104) Premiums, as a consumer promotion tool, are defined as ________. A) offers to consumers of savings off the regular price of a product, flagged on the label or package B) certificates entitling the bearer to a stated saving on the purchase of a specific product C) programs providing rewards related to the consumer's frequency and intensity in purchasing the company's products or services D) merchandise offered at a relatively low cost or free as an incentive to purchase a particular product E) values in cash or in other forms that are proportional to patronage of a certain vendor or group of vendors

D

39) The qualitative value of an exposure through a given medium is known as ________. A) frequency B) reach C) amplitude D) impact E) range

D

97) Which of the following is an example of a manufacturer promotion? A) price cuts B) feature advertising C) retailer coupons D) high-value trade-in credit E) retailer contests or premiums

D

4) Most new-product activities are devoted to ________. A) changing the target markets B) developing new-to-the-world products C) introducing backward integration D) improving existing products E) changing the existing market dynamics

D Diff: 2 LO: 15.1: How can new products be categorized? AACSB: Reflective thinking

33) Which of the following questions is answered during the business analysis of ideas? A) Will this product meet our profit goals? B) Have we got a technically and commercially sound product? C) Can we find a cost-effective, affordable marketing strategy? D) Can this product meet sales expectations? E) Can we find a good concept consumers say they would try?

D Diff: 2 LO: 15.3: What organizational structures and processes do managers use to oversee new-product development? AACSB: Reflective thinking

51) Surgeons and ER nurses would be considered ________ for surgical equipment. A) venture agents B) internal customers C) buzz agents D) lead users E) connectors

D Diff: 2 LO: 15.4: What are the main stages in developing new products and services? AACSB: Application of knowledge

62) A ________ error occurs when the company dismisses a good idea. A) probability B) performance C) double counting D) DROP E) GO

D Diff: 2 LO: 15.4: What are the main stages in developing new products and services? AACSB: Reflective thinking

76) A(n) ________ is a possible product the company might offer to the market. A) test brand B) alpha product C) beta version product D) product idea E) product concept

D Diff: 2 LO: 15.5: What is the best way to manage the generation of new ideas? AACSB: Analytical thinking

101) The least costly way of consumer-goods market testing is ________. A) simulated test marketing B) controlled test marketing C) a few test markets D) sales-wave research E) test marketing in 25 percent of the country

D Diff: 2 LO: 15.6: What is the best way to manage concept and strategy development? AACSB: Analytical thinking

122) Some firms might delay the launch of their products until after the competitor has borne the cost of educating the market. Such an entry is called ________ entry. A) strategic payoff B) parallel C) balancing D) late E) compensating

D Diff: 2 LO: 15.7: What is the best way to manage the commercialization of new products? AACSB: Reflective thinking

132) ________ costs for a manufacturer consist of setup costs and running costs. A) Inventory-carrying B) Containerization C) Wholesaling D) Order-processing E) Transportation

D Diff: 2 LO: 18.6: What are some important issues in logistics? AACSB: Reflective thinking

47) Everyday low pricing is most suitable if ________. A) consumers are willing to perform activities such as clip coupons to avail of discounts B) consumers tend to associate price with quality C) customers are insensitive to changes in price D) the cost of conducting frequent sales and promotions is high E) consumers have sufficient time to find the best prices

D Page Ref: 401 Objective: 2 Difficulty: Moderate

25) Surgeons and ER nurses would be considered ________ for surgical equipment. A) venture agents B) internal customers C) buzz agents D) lead users E) connectors

D Page Ref: 577 Objective: 3 Difficulty: Easy

39) Which of the following represents the objective of a social marketing campaign aimed at changing people's actions? A) Motivate people with obesity to eat healthy and exercise more often. B) Change public attitudes and stereotypes associated with people who are obese. C) Explain the different causes of obesity and how it can be prevented. D) Encourage people to participate in a walkathon aimed at promoting awareness about obesity. E) Help people with obesity to implement lifestyle changes.

D Page Ref: 640 Objective: 3 AACSB: Analytic skills Difficulty: Moderate

35) ________ by nonprofit or government organizations furthers a cause. A) Corporate societal marketing B) Brand marketing C) Causal marketing D) Social marketing E) Place marketing

D Page Ref: 640 Objective: 3 AACSB: Ethical understanding and reasoning abilities Difficulty: Easy

________ is an element of the marketing communications mix that involves online activities and programs designed to engage customers or prospects and directly or indirectly raise awareness, improve image, or elicit sales of products and services. A) Personal selling B) Direct marketing C) Sales promotion D) Interactive marketing E) Public relations

D) Interactive Marketing

________ costs for a manufacturer consist of setup costs and running costs. A) Inventory-carrying B) Containerization C) Wholesaling D) Order-processing E) Transportation

D) Order-processing

Which of the following ads depict a transformational appeal? A) Thompson Water Seal can withstand intense rain, snow, and heat. B) DIRECTV offers better HD options than cable or other satellite operators. C) NBA phenomenon LeBron James pitching Nike, Sprite, and McDonald's. D) Pringles advertised "Once You Pop, the Fun Don't Stop" for years. E) Excedrin stops the toughest headache pain.

D) Pringles advertised "Once You Pop, the Fun Don't Stop" for years.

Marketing communications budgets tend to be higher when there ________. A) is high channel support B) exists hardly any change in the marketing program over time C) are infrequent product purchases in large quantities D) are differentiated products and nonhomogeneous customer needs E) are many easily reachable customers spread over small geographic territories

D) are differentiated products and nonhomogeneous customer needs

Which of the following is an ancillary service offered by retailers? A) accepting orders over the telephone B) advertising and window displays C) delivery to the customer's doorstep D) general information E) alterations and tailoring

D) general information

Which of the following marketing communications principles implies that communicators can use their good image to reduce some negative feelings toward a brand but in the process might lose some esteem with the audience? A) principle of closure B) principle of duality C) principle of delegation D) principle of congruity E) principle of neutrality

D) principle of congruity

Discount stores that try to keep prices as low as possible are more likely to function using ________ operations. A) limited service B) self-selection C) full-service D) self-service E) limited-selection

D) self-service

4) Which of the following elements of the marketing communications mix involves a variety of short-term incentives to encourage trial or purchase of a product or service? A) advertising B) direct marketing C) public relations D) personal selling E) sales promotion

E

Which of the following is an example of a category​ killer? A. ​Kohl's B. Tesco C. The Limited D. Walmart E. PETCO

E

23) Consumers are less price sensitive ________. A) to high cost items B) when they frequently change their buying habits C) when there are more substitutes D) when there are more competitors E) when they do not readily notice higher prices

E Page Ref: 390-391 Objective: 2 Difficulty: Moderate

Which of the following is the marketing communications objective for a new-to-the-world product, such as electric cars? A) enhancing brand awareness B) developing brand attitude C) increasing brand purchase intention D) encouraging repeat purchases E) establishing category need

E) Establishing category need

Which of the following marketing communications tools has the highest cost-effectiveness in the introduction stage of the product life cycle? A) personal selling B) sales promotion C) interactive marketing D) direct marketing E) events and experiences

E) Events and Experiences

________ refer to the marketing communications element that involves company-sponsored activities and programs designed to create daily or special brand-related interactions with consumers. A) Publicity campaigns B) Trade promotions C) Advertisements D) Public relations E) Events and experiences

E) Events and experiences

Which of the following elements of the marketing communications mix involves a variety of short-term incentives to encourage trial or purchase of a product or service? A) advertising B) direct marketing C) public relations D) personal selling E) sales promotion

E) Sales promotion

131) Brick-and-click companies are those that have launched a Web site without any previous existence as a firm.

FALSE

52) Sandy's Stores is a small chain of grocery stores located in a few neighboring towns. The stores have always been largely self-service, but the company is considering making a switch to full-service stores. Offer reasons why Sandy's should stick with its current system.

Student answers will vary. As Sandy's is primarily a grocery chain, it is unlikely that customers will need much help in locating products. The high staffing cost of full-service retailing will only increase operational costs for Sandy's, without conferring any benefit.

55) Skincare company E&OE has realized that its customers are very loyal to the brand and play a large part in popularizing it by word of mouth. E&OE wants to leverage this customer loyalty by using it to generate sales outside its store format. How can E&OE achieve this?

Student answers will vary. E&OE can opt for the direct selling method, using multilevel or network marketing. A salesperson can go to the home of a loyal customer who has invited friends; the salesperson demonstrates the products and takes orders. The company can also recruit independent customers who act as distributors. The distributor's compensation includes a percentage of sales made by those he or she recruits, as well as earnings on direct sales to customers.

154) E&OE wants to minimize inventory costs as far as possible. Explain one way by which it can achieve a near-zero inventory.

Student answers will vary. Manufacturers can achieve a near-zero inventory by building for order, not for stock, i.e., manufacturing a product only when it receives an order. However, this system will work only when the customers do not need a product immediately.

110) Supermarket chain Reynold's is considering making a switch to stocking almost exclusively private-label products in order to offer customers the lowest prices. Offer reasons why Reynold's should think twice before opting to stock exclusively private-label products.

Student answers will vary. Reynold's should think twice about opting for private-label products because consumers prefer certain national brands, and many product categories are not feasible or attractive on a private-label basis.

When ConAgra foods decided to cut $250 million in costs to return to a $1 price point (after sales dropped as a result of raising prices $0.25 to cover higher commodity costs), it was using _______.

Target Costing

126) What are the different types of price discounts and allowances?

The different types of price discounts and allowances are: • Discount - This is a price reduction given to buyers who pay their bills promptly. • Quantity discount - This is a price reduction offered to those who buy in large volumes. • Functional discount - This is offered by a manufacturer to trade-channel members if they perform certain functions like selling, storing, and record keeping. • Seasonal discount - This is a price reduction given to those who buy merchandise or services out of season. • Allowance - This is an extra payment designed to gain reseller participation in special programs. These are of two types: i. Trade-in allowances - These are granted for turning in an old item when buying a new one. ii. Promotional allowances - These reward dealers for participating in advertising and sales support programs. Page Ref: 404 Objective: 3 Difficulty: Moderate

118) Identify the major reasons why new product failures occur.

The following are some of the possible reasons for the failure of new products. (1) Shortage of important ideas in certain areas, (2) fragmented markets, (3) social, economic, and governmental constraints, (4) high cost of development, (4) capital shortages, (5) shorter required development time, (6) poor launch timing, shorter product life cycles, and (7) organizational support. Page Ref: 572 Objective: 1 Difficulty: Easy

134) Agatha's Inc. is about to introduce a new product in the market, but is not sure as to how it should price the product. The company is facing intense competition from 5 other companies. In the past, it has also failed to keep up with the changing consumer wants. In such a situation, what should be its main objective?

The main objective for Agatha's Inc. should be survival. As long as prices cover variable costs and some fixed costs, the company will stay in business. But, it is worthwhile to remember that survival is a short-run objective. In the long run, the company has to add value to its product or face extinction. Page Ref: 389 Objective: 2 AACSB: Analytic skills Difficulty: Moderate

140) List the major objectives of events and experiences as promotion tools.

The major objectives of events and experiences as promotion tools are: 1. To identify with a particular target market or lifestyle 2. To increase salience of company or product name 3. To create or reinforce perceptions of key brand image associations 4. To enhance corporate image 5. To create experiences and evoke feelings 6. To express commitment to the community or on social issues 7. To entertain key clients or reward key employees 8. To permit merchandising or promotional opportunities

118) Describe the functional organization of a marketing department in terms of its structure, advantages, and disadvantages.

The most common form of marketing organization consists of functional specialists reporting to a marketing vice president. The main advantage of a functional marketing organization is its administrative simplicity. It can be quite a challenge for the department to develop smooth working relationships, however. This form also can result in inadequate planning as the number of products and markets increases and each functional group vies for budget and status. The marketing vice president constantly weighs competing claims and faces a difficult coordination problem. Page Ref: 627 Objective: 2 Difficulty: Moderate

149) What are the steps involved in marketing profitability analysis?

The steps involved in marketing profitability analysis are as follows: Step 1: Identifying functional expenses Step 2: Assigning functional expenses to marketing entities Step 3: Preparing a profit-and-loss statement for each marketing entity

143) What are the steps involved in marketing profitability analysis?

The steps involved in marketing profitability analysis are as follows: Step 1: Identifying functional expenses Step 2: Assigning functional expenses to marketing entities Step 3: Preparing a profit-and-loss statement for each marketing entity Page Ref: 656-657 Objective: APP Difficulty: Moderate

121) Briefly explain the three parts of the demand-first innovation and growth (DIG) framework.

The three parts of the DIG framework are the following. (1) The demand landscape: Use observational, anthropological, and ethnographic methods or consumer self-reports to map consumer needs, wants, and even beyond. (2) The opportunity space: Use conceptual lens and structured innovative-thinking tools to achieve market perspectives from different angles. (3) The strategic blueprint: Think about how the new product can fit into customers' lives and how it can be distinguished from competitors. Page Ref: 575-576 Objective: 3 Difficulty: Easy

46) What is a brand-asset management team (BAMT)?

Triangular and horizontal product-team approaches let each major brand be run by a brand-asset management team (BAMT) consisting of key representatives from functions that affect the brand's performance. The company consists of several BAMTs that periodically report to a BAMT directors committee, which itself reports to a chief branding officer.

127) What is a brand-asset management team (BAMT)?

Triangular and horizontal product-team approaches let each major brand be run by a brand-asset management team (BAMT) consisting of key representatives from functions that affect the brand's performance. The company consists of several BAMTs that periodically report to a BAMT directors committee, which itself reports to a chief branding officer. Page Ref: 628 Objective: 2 Difficulty: Easy

When would pricing cues such as sale signs and prices that end in "9" be less meaningful for consumers?

When consumers are experienced in the category

10) A computer manufacturing company allows customers to place orders online, which they can later pick up from a convenient retail location. Which of the following terms best represents this practice? A) channel integration B) mass customization C) online personalization D) push strategy E) internal marketing

a

24) Atburex is a furniture manufacturing company in the United States. The company provides a sixty day credit period and EMI options to customers and also offers on-site delivery and installation. These special benefits refer to which of the following service outputs? A) good service backup B) large product variety C) spatial convenience D) large lot size E) short waiting time

a

3) A(n) ________ is a facilitator who assists in the distribution process. A) advertising agency B) sales agent C) manufacturer's representative D) broker E) wholesaler

a

32) Which of the following products is most likely to be sold using an exclusive distribution strategy? A) designer luggage B) cigarettes C) alcoholic beverages D) car fuel E) medicine

a

33) Which of the following covers payment terms and producer guarantees? A) conditions of sale B) pricing policies C) exclusive dealings D) mutual services E) territorial rights

a

36) Which of the following channels is associated with the lowest cost per transaction? A) Internet B) telemarketing C) retail stores D) distributor E) sales force

a

38) Which of the following problems is most likely when a sales agency is used instead of company salespeople? A) The company will find it difficult to control the sales process. B) They do not take title to goods or negotiate purchases or sales. C) The value-added per sale is the lowest for sales agencies. D) Agencies will pay less attention to customers who buy the most or in large volumes. E) Sales agencies are often difficult to access due to strict contractual obligations.

a

44) A manufacturer is using legitimate power when it ________. A) requests a behavior that is warranted under the selling contract B) threatens to withdraw a resource or terminate a relationship C) offers intermediaries an extra benefit for performing specific acts or functions D) makes the intermediaries sell more of a particular product by offering rewards E) sells more products by making use of its reputation in the market

a

45) Hewlett-Packard is a highly respected brand. Many retailers want to be associated with the brand because of this reputation. What kind of power does Hewlett-Packard obtain due to this reputation? A) referent B) functional C) legitimate D) coercive E) reward

a

46) Which of the following types of power is objectively observable? A) coercive power B) legitimate power C) group power D) expert power E) referent power

a

52) A(n) ________ vertical marketing system combines successive stages of production and distribution under single ownership. A) corporate B) administered C) contractual D) regulatory E) controlled

a

66) A manufacturer wants to achieve rapid market penetration through a low-price policy. However, its dealers prefer to work with high margins and pursue short-run profitability. The major reason for this conflict is ________. A) goal incompatibility B) unclear roles C) ambiguous rights D) differences in perception E) dependence on the manufacturer

a

67) General Motors' executives work for a short time in some dealerships, and some dealership owners work in GM's dealer policy department. This strategy helps the company avoid conflicts with its dealers. This is an example of the ________ strategy. A) employee exchange B) dual compensation C) joint membership D) co-optation E) diplomacy

a

69) Co-optation is an effort by one organization to win the support of the leaders of another by ________. A) including them in advisory councils B) engaging in mediation and arbitration C) encouraging joint memberships in trade associations D) encouraging employee exchanges E) offering strategic justifications

a

55) An administered VMS coordinates successive stages of production and distribution through ________. A) an automated central control unit B) single ownership C) the combined efforts of all its members D) the establishment of contractual obligations E) the size and power of one of the members

e

1) ________ includes all the activities in selling goods or services directly to final consumers for personal, non-business use. A) Wholesaling B) Retailing C) Procurement D) Promoting E) Warehousing

B Diff: 1 LO: 18.1: What major types of marketing intermediaries occupy this sector? AACSB: Analytical thinking

35) ________ includes all the activities in selling goods or services to those who buy for resale or business use. A) Retailing B) Wholesaling C) Procurement D) Promoting E) Warehousing

B Diff: 1 LO: 18.1: What major types of marketing intermediaries occupy this sector? AACSB: Reflective thinking

144) If the shipper owns its own truck or air fleet, it becomes a ________ carrier. A) containerized B) private C) contract D) common E) diversified

B Diff: 1 LO: 18.6: What are some important issues in logistics? AACSB: Reflective thinking

28) ________ teams are cross-functional groups charged with developing a specific product or business. A) Virtual B) Venture C) Fundamental D) Elemental E) Transitory

B Diff: 2 LO: 15.3: What organizational structures and processes do managers use to oversee new-product development? AACSB: Interpersonal relations and teamwork

59) Increasingly, new-product ideas arise from ________ that combines two product concepts or ideas to create a new offering. A) reverse assumption analysis B) lateral marketing C) attribute listing D) forced relationships E) morphological analysis

B Diff: 2 LO: 15.4: What are the main stages in developing new products and services?

17) When a company introduces a product at a very high price and then gradually drops the price over time, it is pursuing a ________ strategy. A) market-penetration pricing B) market-skimming pricing C) value-pricing D) switching cost E) loss-leader pricing

B Page Ref: 390 Objective: 2 Difficulty: Easy

120) ________ refers to buying large carload lots and dividing them into smaller units before shipping them out to consumers. A) Bulk breaking B) Containerization C) Wholesaling D) Warehousing E) Broking

A

145) When a public relations department of a company advises management about public issues, and company positions and image during good times and bad, it is performing the function of ________. A) counseling B) corporate communications C) product publicity D) lobbying E) press relations

A

13) ________ requires that everyone in the organization accept the concepts and goals of marketing and engage in choosing, providing, and communicating customer value. A) Internal marketing B) Corporate communications C) Integrated marketing communications D) Supply chain management E) Employee engagement

A

66) The first step in the social marketing planning process is ________. A) determining the focus of the program B) selecting the target audience C) setting objectives and goals D) designing the market offering E) finding a source of funding

A

48) What is a customer-management organization? When should a customer-management organization be adopted?

A customer-management organization deals with individual customers rather than the mass market or even market segments. It is suitable when a close customer relationship is advantageous, such as when customers have diverse and complex requirements and buy an integrated bundle of products and services.

74) Which of the following is true for direct product profitability? A) It is highly correlated with the gross margin on a product. B) It is negligible compared to the gross margin on a product. C) It bears little relation to the gross margin on a product. D) It is significantly lower than the gross margin on a product. E) It is exactly the same as the gross margin on a product.

C Diff: 3 LO: 18.3: What marketing decisions do marketing intermediaries make? AACSB: Analytical thinking

18) When Apple introduced its iPhone, it was priced at $599. This allowed Apple to earn the maximum amount of revenue from the various segments of the market. Two months after the introduction, the price has come down to $399. What kind of a pricing did Apple adopt? A) loss-leader pricing B) market-penetration pricing C) market-skimming pricing D) target-return pricing E) value pricing

C Page Ref: 390 Objective: 2 AACSB: Analytic skills Difficulty: Moderate

25) If demand hardly changes with a small change in price, the demand is said to be ________. A) strained B) marginal C) inelastic D) flexible E) unit elastic

C Page Ref: 392 Objective: 2 Difficulty: Easy

Which of the following is true of brokers? A) Brokers represent buyers or sellers on a semipermanent basis. B) Most brokers are small businesses with a few skilled salespeople. C) Brokers bring buyers and sellers together and assist in negotiation. D) Selling brokers have contractual authority to sell a manufacturer's entire output. E) Purchasing brokers make purchases for buyers and often receive, inspect, warehouse, and ship merchandise.

C) Brokers bring buyers and sellers together and assist in negotiation.

Which of the following is an example of an events and experiences platform? A) fairs and trade shows B) continuity programs C) factory tours D) sales presentations E) community relations

C) Factory Tours

________ are unbranded, plainly packaged, less expensive versions of common products such as spaghetti, paper towels, and canned peaches. A) Common carriers B) Shills C) Generics D) Private labels E) Marques

C) Generics

________ are independently owned businesses that take title to the merchandise they handle. They are full-service and limited-service jobbers, distributors, and mill supply houses. A) Brokers B) Agents C) Merchant wholesalers D) Specialized wholesalers E) Retailers' branches

C) Merchant wholesalers

Which of the following is a form of mass communications channel? A) interactive marketing B) personal selling C) public relations D) word-of-mouth marketing E) sales presentations

C) Public Relations

In the corporate headquarters of a supermarket chain, ________ are responsible for developing brand assortments and listening to salespersons' presentations. A) central buyers B) brokers C) specialist buyers D) agents E) specialized wholesalers

C) specialist buyers

106) Marketing communications budgets tend to be higher when there ________. A) is high channel support B) exists hardly any change in the marketing program over time C) are infrequent product purchases in large quantities D) are differentiated products and nonhomogeneous customer needs E) are many easily reachable customers spread over small geographic territories

D

114) ________ are independently owned businesses that take title to the merchandise they handle. They are full-service and limited-service jobbers, distributors, and mill supply houses. A) Brokers B) Agents C) Merchant wholesalers D) Specialized wholesalers E) Retailers' branches

C

35) Which of the following circumstances are best suited for personal selling? A) when the products used are simple and easy-to-use B) when there is minimal risk involved in buying or using the products C) when the market has fewer and larger sellers D) when the products being marketed are inexpensive and easily available E) when prospective customers are spread across a wide geographic area

C

37) Which element of the marketing communications mix allows for amplified expressiveness? A) sales promotion B) public relations C) advertising D) direct and interactive marketing E) personal selling

C

54) Which of the following is a disadvantage of using outdoor media for advertising? A) low flexibility B) low repeat exposure C) limited audience selectivity D) high costs E) greater competition

C

91) Which of the following sources of a spokesperson's credibility describes his or her attractiveness? A) expertise B) trustworthiness C) likability D) integrity E) experience

C

Which of the following is a reason intermediaries sponsor their own private label​ brands? A. Private label brands give manufacturers increased bargaining power. B. Generic brands can help differentiate a retailer from a competitor. C. Private label brands can be more profitable. D. ​Price-sensitive consumers prefer store brands in certain categories. E. Private label brands offer​ lower-priced products as an alternative to generics.

C

13) H&A is a retail chain that specializes in selling goods at very low prices. To achieve this, it stocks a very narrow assortment of basic necessities and offers customers a "no-frills" shopping experience. H&A is an example of a(n) ________ store. A) off-price B) specialty C) hard-discount D) superstore E) convenience

C Diff: 2 LO: 18.1: What major types of marketing intermediaries occupy this sector? AACSB: Analytical thinking; Application of knowledge

18) Avon, Tupperware, and Southwestern Company of Nashville are among companies in the multibillion-dollar ________ industry, which involves selling door-to-door or at home sales parties. A) direct marketing B) catalog showroom C) direct selling D) automatic vending E) buying services

C Diff: 2 LO: 18.1: What major types of marketing intermediaries occupy this sector? AACSB: Application of knowledge

33) Jake wants to open a Subway franchise in his small town. To do this, he must pay the company a ________ fee. A) slotting B) title C) royalty D) merchandising E) residual

C Diff: 2 LO: 18.1: What major types of marketing intermediaries occupy this sector? AACSB: Application of knowledge

27) An independent retailer using a central buying organization and joint promotion efforts with other retailers is part of a ________. A) corporate chain store B) voluntary chain C) retailer cooperative D) merchandising conglomerate E) franchise organization

C Diff: 2 LO: 18.1: What major types of marketing intermediaries occupy this sector? AACSB: Reflective thinking

28) A ________ is a retail firm owned by its customers. Members contribute money to open their own store, vote on its policies, elect a group to manage it, and receive dividends. A) retailer cooperative B) voluntary chain C) consumer cooperative D) merchandising conglomerate E) franchise organization

C Diff: 2 LO: 18.1: What major types of marketing intermediaries occupy this sector? AACSB: Reflective thinking

31) Which of the following is true for franchisees? A) The franchisee is paid by the franchisor to be part of the franchise system. B) The franchisee licenses a trademark to the franchisor. C) The franchisee must change its operations to suit those of the franchisor. D) The franchisee collects royalty payments from the franchisor. E) The franchisee owns the trade or service mark.

C Diff: 2 LO: 18.1: What major types of marketing intermediaries occupy this sector? AACSB: Reflective thinking

42) Luke is considering the various options available to him to promote an energy drink, Turbozade, that has decreasing sales volumes after having peaked some time back. Which of the following marketing communications tools should Luke focus marketing efforts on to keep the sales volume up? A) advertising B) direct marketing C) events and experiences D) sales promotions E) publicity

D

92) Which of the following marketing communications principles implies that communicators can use their good image to reduce some negative feelings toward a brand but in the process might lose some esteem with the audience? A) principle of closure B) principle of duality C) principle of delegation D) principle of congruity E) principle of neutrality

D

96) As long as finance focuses on short-term profit, it will oppose major investments designed to build satisfied, loyal customers. Which of the following steps that a marketing CEO can take to create a market- and customer-focused company can help a CEO overcome this challenge? A) Empower the employees. B) Get outside help and guidance. C) Install a modern marketing planning system. D) Change the company's reward measurement and system. E) Develop strong in-house marketing training programs.

D

99) Which of the following is a characteristic of the affordable method of establishing a marketing communications budget? A) fixed annual budget B) suitable for long-range planning C) priority given to role of promotion as an investment D) calculated to reflect what the company can spare for marketing communications E) based on the immediate impact of promotion on sales volume

D

A full​ service, independently owned business that takes title to the merchandise being handled is​ a(n) ________ wholesaler. A. industrial B. ​producers' cooperatives C. ​full-service D. retail E. merchant

D

An example of a restaurant with a narrow and deep assortment is a​ ________. A. large restaurant B. cafeteria C. casual dining restaurant chain D. delicatessen E. small lunch counter

D

146) Relative advantage of an innovation refers to the degree to which ________. A) it matches the values and experiences of the individuals B) it is difficult to understand or use C) it can be tried on a limited basis D) it appears superior to existing products E) the benefits of use are observable or describable to others

D Diff: 2 LO: 15.8: What factors affect the rate of diffusion and consumer adoption of newly launched products and services? AACSB: Analytical thinking

71) When hotels drop their rates on the weekends, then this form of price discrimination is known as ________. A) channel pricing B) image pricing C) product-form pricing D) time pricing E) location pricing

D Page Ref: 406 Objective: 3 Difficulty: Easy

58) Which of the following is usually referred to as a full-blown test market? A) an internal focus group B) a discussion group C) a country D) a city or a few cities E) a laboratory

D Page Ref: 589 Objective: 4 Difficulty: Easy

59) Which of the following is likely to be an important trend in marketing in the future? A) marketing intuition B) free-spending marketing C) manual marketing D) marketing science E) mass marketing

D Page Ref: 648 Objective: 5 Difficulty: Moderate

102) Though it is sold only in Walmart stores, Walmart's Ol'Roy dog food has surpassed Nestlé's Purina brand as the top-selling dog food. Ol'Roy is an example of a ________. A) generic product B) national brand C) franchise D) copy-cat brand E) private label

E

________ consists of putting the goods in boxes or trailers that are easy to transfer between two transportation modes. A. Trainshipping B. Fishybacking C. Piggybacking D. Airtrucking E. Containerization

E

141) When does personal influence carry the most weight?

Personal influence carries especially great weight when products are expensive, risky, or purchased infrequently, and when products suggest something about the user's status or taste. People often ask others to recommend a doctor, plumber, hotel, lawyer, accountant, architect, insurance agent, interior decorator, or financial consultant. If we have confidence in the recommendation, we normally act on the referral. Service providers clearly have a strong interest in building referral sources.

71) Which of the following channel conflict resolution techniques is used only if everything else proves ineffective? A) legal recourse B) mediation C) arbitration D) co-option E) superordinate goals

a

74) An Internet service provider (ISP) is a(n) ________ company. A) pure-click B) brick and click C) brick-and-mortar D) m-commerce E) one-level

a

31) Costs that differ directly with the level of production are known as ________. A) fixed costs B) overhead costs C) opportunity costs D) target costs E) variable costs

E Page Ref: 393 Objective: 2 Difficulty: Easy

72) Joseph, a student of the Columbia University, finds many of his classmates have purchased the I-pad tablet PC from Apple Computers. I-pad, launched a few months before has been identified as a very useful product and many students in the U.S. have rated it highly. Considering all these, Joseph also decides to purchase an I-pad. Which of the following is the adopter group to which Joseph belongs? A) early adopter B) innovator C) late majority D) laggard E) early majority

E Page Ref: 591 Objective: 5 AACSB: Analytic skills Difficulty: Moderate

4) Merging involves ________. A) reducing the number of organizational levels to get closer to the customer B) removing barriers that separate organizational departments C) partnering with fewer but better value-adding suppliers D) working more closely with customers to add value to their operations E) acquiring firms in the same or complementary industries

E Page Ref: 624 Objective: 1 Difficulty: Moderate

Advertising and publicity tools play the most important roles in influencing buying decisions at the ________ stage of buyer readiness. A) comprehension B) conviction C) ordering D) reordering E) awareness-building

E) Awareness Building

According to the hierarchy-of-effects model, which of the following corresponds to the affective stage that a buyer passes through? A) attention B) exposure C) reception D) adoption E) conviction

E) Conviction

Which of the following results in a tangible increase in price?

Escalator Clauses

69) If a small Iowa college builds awareness among applicants from Nebraska, increases the target audience's knowledge about its offerings, communicates in a way that makes the target audience like it and prefer it to other colleges, builds intent to apply, and gets the target audience to attend, they are using an AIDA model of consumer responses.

FALSE

70) Some business practices, such as deceptive advertising, exclusive dealing, and predatory competition, sharply divide critics regarding whether they are clearly unethical or illegal.

FALSE

87) In order to clearly differentiate themselves from competitors, retailers must keep merchandise and prices consistent.

FALSE

108) A group of small grocery shops forms a new business entity to buy products directly from manufacturers. The group buys products in bulk that are then distributed among members. This helps the shops obtain better profit margins. Which of the following types of vertical marketing systems can be observed here? A) contractual B) corporate C) administered D) controlled E) regulatory

A

124) Which of the following is the most complete and accurate description of a pure-click company? A) These are companies that launch a Web site without any previous existence as a firm. B) These firms' business models are based on advertising revenue. C) These are established companies that have an online site for selling products. D) These companies get paid every time a user visits their Web site. E) These companies sell online space to other Web sites and are paid for the service.

A

147) Co-optation is an effort by one organization to win the support of the leaders of another by ________. A) including them in advisory councils B) engaging in mediation and arbitration C) encouraging joint memberships in trade associations D) encouraging employee exchanges E) offering strategic justifications

A

81) One of the possible objectives of marketing communications is helping consumers evaluate a brand's perceived ability to meet a currently relevant need. Which of the following is a negatively oriented relevant brand need? A) normal depletion B) sensory gratification C) social approval D) intellectual stimulation E) value enhancement

A

84) Creative strategies refer to the ________. A) way marketers translate their messages into a specific communication B) amount of creative content in a communications message C) degree of innovation involved in the marketing of a product D) novelty of a marketing communication E) type of medium used to deliver a marketing communication

A

97) IBM lets frontline employees spend up to $5,000 to solve a customer problem on the spot, which is an example of which of the following steps that a marketing CEO can take to create a market- and customer-focused company? A) Empower the employees. B) Hire strong marketing talent. C) Get outside help and guidance. D) Install a modern marketing planning system. E) Develop strong in-house marketing training programs.

A

99) What is the major difference between a conventional marketing channel and a vertical marketing system (VMS)? A) Elements in a conventional marketing channel act as separate businesses whereas the elements in a VMS act as a unified system. B) A VMS has many intermediaries whereas a conventional marketing channel has a limited number of intermediaries. C) A VMS is characterized by an independent producer whereas a conventional marketing channel is characterized by multiple producers. D) A conventional marketing channel has elements such as retailers and wholesalers whereas these elements are not present in a VMS. E) Producers have complete control over the other members in a conventional marketing channel whereas this control is minimal in a VMS.

A

10) Which of the following types of retailing generally entails the highest costs? A) full-service B) self-service C) limited-selection D) limited service E) self-selection

A

101) A(n) ________ vertical marketing system combines successive stages of production and distribution under single ownership. A) corporate B) administered C) contractual D) regulatory E) controlled

A

107) Qualities such as candor, humor, and naturalness make a spokesperson ________, which is an important component of spokesperson credibility. A) likable B) an expert C) trustworthy D) transformational E) informational

A

11) Companies should first think about the target market and then design the supply chain backward from that point. This strategy is called ________ planning. A) demand chain B) resource C) external channel D) materials E) strategic business

A

11) The role of marketing in the organization is changing. Traditionally marketers have played the roles of ________, charged with understanding customers' needs and transmitting their voice to various functional areas. A) middlemen B) opinion leaders C) clients D) influencers E) end consumers

A

111) Which of the following is a major advantage of adding more channels for selling? A) It helps the company increase its market coverage. B) It helps the company reduce its fixed costs. C) It reduces the likelihood of channel conflict. D) It is the best strategy for selling low-involvement consumer products. E) It results in economies of scale.

A

116) ________ is market share in comparison to the largest competitor. A) Relative market share B) Served market share C) Overall market share D) Market value E) Target market share

A

12) Bill and Josh are considering opening a retail store. They have identified their target market and location and are finalizing the details of the merchandise they will carry. Since the neighborhood is rundown and the customers in the area are very price-conscious, Bill and Josh want to offer goods from well-known brands, but at lower rates than the full retail prices of the products. They choose to stock excess production from manufacturers or goods that have remained unsold at other retailers. This is a description of a(n) ________ retailer. A) off-price B) specialty C) discount D) department E) catalog

A

130) Which of the following is an example of a brick-and-click company? A) IFB Industries, a company that sells products using various channels such as Internet, retailers, direct outlets, and franchisees. B) Opera, a Web browser and Internet suite developed by Opera Software, performs common tasks such as displaying Web sites and sending and receiving e-mail messages. C) Atrutron, a company that offers its customers access to the Internet using data transmission technology. D) eBay Inc., an American company, facilitates online auctions and shopping to people and businesses across the globe to buy and sell a broad variety of goods and services. E) Yahoo! Inc., an American corporation, provides services via the Internet such as directories, e-mail, news, advertising, online mapping, and so on.

A

134) M-commerce refers to ________. A) conducting business using mobile channels B) the use of mass media communications to attract customers C) providing mobile and on-site services to customers D) the use of a specific medium to communicate with prospects E) using the Internet as a medium for doing business

A

139) Beyond the optimal order quantity, total cost per unit increases because ________. A) inventory-carrying cost per unit increases B) inventory-carrying cost per unit decreases C) order-processing cost per unit increases D) order-processing cost per unit increases though inventory cost decreases E) inventory-processing cost per unit falls slowly

A

145) Which of the following is NOT one of the four important contributions an effectively trained company sales force can make to consumer marketing? A) remind end-consumers about the product B) increase stock position C) build enthusiasm D) conduct missionary selling E) manage key accounts

A

146) RX Corp. is a large manufacturer of electronic goods and sells its products through distributors and retailers. In order to keep pace with the growing use of the Internet, the company decides to start selling online. The company faces stiff opposition from its retailers as they believe that this will significantly reduce their profits. The company attempts to eliminate this resistance by offering its retailers commissions for processing and delivering orders received via the Web. This is an example of which of the following conflict resolution strategies? A) dual compensation B) joint membership C) arbitration D) co-optation E) strategic pricing

A

149) Which of the following channel conflict resolution techniques is used only if everything else proves ineffective? A) legal recourse B) mediation C) arbitration D) co-optation E) superordinate goals

A

15) Which of the following is an example of a public relations and publicity communication platform? A) lobbying B) company museums C) street activities D) company blogs E) incentive programs

A

153) Which of the following is NOT a predicted direction for marketing in the coming years? A) the demise of precision marketing and rise of mass marketing B) the demise of the marketing department and the rise of holistic marketing C) the demise of marketing intuition and the rise of marketing science D) the demise of manual marketing and the rise of both automated and creative marketing E) the demise of free-spending marketing and the rise of ROI marketing

A

17) A product-management organization makes sense if ________. A) the company's products are quite varied B) there are very few products in the company's portfolio C) the company is pursuing a low cost strategy D) the company's products satisfy similar customer needs E) the company is vertically integrated

A

18) A product- or brand-management organization is characterized as a(n) ________ system. A) hub-and-spoke B) waterfall C) internal marketing D) top-down E) lateral

A

2) A merchant is a(n) ________ in the marketing channel. A) wholesaler B) broker C) sales agent D) warehouse E) advertising agency

A

26) Which of the following entities is present in a zero-level marketing channel? A) consumers B) retailers C) brokers D) jobbers E) wholesalers

A

32) Which of the following is a benefit of franchising for franchisees? A) The franchisee finds it easier to borrow money from financial institutions. B) The franchisee receives ownership of the franchisor's trademark. C) The franchisee must change its operations to suit those of the franchisor. D) The franchisee collects royalty payments from the franchisor. E) The franchisee is paid by the franchisor for being part of the system.

A

33) According to researchers, which of the following is the correct order in which content of print advertisements matter? A) picture-headline-copy B) copy-picture-headline C) headline-copy-picture D) picture-copy-headline E) copy-headline-picture

A

33) Which of the following marketing communications tools is most effective at the later stages of the buying process? A) personal selling B) public relations C) advertising D) sales promotions E) direct marketing

A

45) Which of the following types of distribution involves severely limiting the number of channel intermediaries? A) exclusive B) selective C) intensive D) aggressive E) retail

A

56) Which of the following channels is associated with the lowest cost per transaction? A) Internet B) telemarketing C) retail stores D) distributor E) sales force

A

56) Which of the following is a disadvantage of using Yellow Pages as an advertising medium? A) high competition B) poor local market coverage C) low believability D) lack of adequate reach E) high total costs

A

58) Which of the following is true for the retail industry? A) Discount stores and catalog showrooms are competing for the same customers. B) Upscale retailers see a decline in sales as middle-market retailers thrive. C) Small, specialized retailers are crowding out larger, more diverse retailers. D) Store retailing sees no competition from nonstore retailing. E) Discount stores are not doing as well as middle-market retailers.

A

63) Which of the following media timing factors expresses the rate at which new customers enter the market? A) buyer turnover B) purchase frequency C) media reach D) weighted number of exposures E) advertising impressions

A

77) Marketing communication strategy can be decided by conducting an image analysis by profiling the target audience in terms of ________. A) brand knowledge B) purchase patterns C) demographic characteristics D) income levels E) psychographic characteristics

A

79) Which of the following is a prepurchase service offered by retailers? A) accepting orders over the telephone B) shipping the product C) delivery to the customer's doorstep D) general information E) interior decoration of the retail outlet

A

85) A producer must modify its channel design and arrangements if ________. A) consumer buying patterns change B) the competition in the market stabilizes C) the product is in the growth stage of its life cycle D) the market size remains unchanged for a particular period E) the firm's profits stabilize

A

94) ________ is the process that turns marketing plans into action assignments and ensures that such assignments are executed in a manner that accomplishes the plan's stated objectives. A) Marketing implementation B) Marketing research C) Marketing analysis D) Brand management E) Product management

A

95) ________ is the process by which firms assess the effects of their marketing activities and programs and make necessary changes and adjustments. A) Marketing control B) Marketing implementation C) Test marketing D) Market watch E) Market analysis

A

129) What is a customer-management organization? When should it be adopted?

A customer-management organization deals with individual customers rather than the mass market or even market segments. It is suitable when a close customer relationship is advantageous, such as when customers have diverse and complex requirements and buy an integrated bundle of products and services. Page Ref: 629 Objective: 2 Difficulty: Easy

and provide reviews on the cars' performance, their prices, and other factors. Which of the following personal communications channels is Top Gear closest to in description? A) expert channel B) formal channel C) social channel D) sponsored channel E) advocate channel

A

6) Most established companies focus on ________ innovation when they aim to enter new markets by tweaking existing products, or they want to stay one step ahead in the market by using variations on a core product. A) incremental B) continuous C) spontaneous D) radical E) competitive

A Diff: 2 LO: 15.1: How can new products be categorized? AACSB: Reflective thinking

16) Pager, a simple personal device for short messages, became famous in the 1990s. Troveron Communications launched a pager in the early twenty-first century. Due to the introduction of mobile phones and text messaging, the pager industry was on a decline. The company's innovations were not well received by the market and the product was a failure. Which of the following is the most likely reason for the product's failure in this case? A) poor launch timing of the product B) a small and fragmented target market C) high cost of development D) social and economic constraints E) hasty product development

A Diff: 2 LO: 15.2: What challenges does a company face in developing new products and services? AACSB: Analytical thinking; Application of knowledge

30) A team formed at Intercom Inc. to generate ideas for new products conducts frequent meetings and engages in activities such as mind mapping and brainstorming. Most of the meetings are conducted at informal locations away from office. These workplaces are called A) skunkworks B) idea funnels C) research centers D) stage-gate systems E) contextual bases

A Diff: 2 LO: 15.3: What organizational structures and processes do managers use to oversee new-product development? AACSB: Analytical thinking

35) A ________ process recognizes the value of returning to an earlier stage to make improvements before moving forward. A) spiral development B) reactive development C) market testing D) proactive development E) concept testing

A Diff: 2 LO: 15.3: What organizational structures and processes do managers use to oversee new-product development? AACSB: Reflective thinking

58) The ________ creativity technique begins with a thought that is written down, then proceeds to develop other thoughts that are linked to the first thought, and finally a new idea materializes by associating all of the thoughts. A) mind mapping B) contextual analysis C) attribute listing D) reverse assumption analysis E) morphological analysis

A Diff: 2 LO: 15.4: What are the main stages in developing new products and services?

63) The stage in the new product process that occurs first and has a pass ratio of 1:4 is the ________ stage. A) idea screening B) product development C) test marketing D) product soft launch E) concept testing

A Diff: 2 LO: 15.4: What are the main stages in developing new products and services? AACSB: Analytical thinking

56) Electro Locomotors (EL) Inc. engages in the manufacture, distribution, and marketing of custom vehicles in Brazil. Bono, a hybrid car brand that the company produces, is a market leader in the hybrid vehicles segment. With the increased global focus on nature-friendly vehicles and governmental subsidies for companies that manufacture hybrid vehicles, competition has increased for flex-fuel cars like Bono. The heads of the R&D team at EL are in a creative session to generate ideas for improvements to Bono. They consider each part of the car and discuss the possibilities of improvements in each of them. Identify the creativity technique used here. A) attribute listing B) forced relationship analysis C) morphological analysis D) reverse assumption analysis E) mind mapping

A Diff: 2 LO: 15.4: What are the main stages in developing new products and services? AACSB: Analytical thinking; Application of knowledge

61) Morboro Energy Inc. is a large chain of fuel stations in Europe. The company decided to expand its business by incorporating restaurants and automobile service stations as a part of its fuel outlets. This is an example of ________. A) lateral marketing B) brand extension C) market development D) internal marketing E) brand personification

A Diff: 2 LO: 15.4: What are the main stages in developing new products and services? AACSB: Analytical thinking; Application of knowledge

103) During ________ testing, the company's technical people observe how customers use the product, a practice that often exposes unanticipated problems of safety and servicing and alerts the company to customer training and servicing requirements. A) beta B) research C) sales-wave D) alpha E) simulated

A Diff: 2 LO: 15.6: What is the best way to manage concept and strategy development? AACSB: Application of knowledge

88) The trade-off approach may be easier to use when there ________. A) are many variables B) are only a few alternatives C) are observers that are biased D) are possible offers that are limited E) is only one highly desirable solution

A Diff: 2 LO: 15.6: What is the best way to manage concept and strategy development? AACSB: Reflective thinking

91) To estimate replacement sales, management must research the product's ________, the number of units that fail in year one, two, three, and so on. A) survival-age distribution B) life-cycle ratio C) obsolescence rate D) business turnover ratio E) product-performance usage

A Diff: 2 LO: 15.6: What is the best way to manage concept and strategy development? AACSB: Reflective thinking

92) ________ contribution lists the changes in income to other company products caused by the introduction of a new product. A) Supplementary B) Dragalong C) Gross D) Cumulative E) Net

A Diff: 2 LO: 15.6: What is the best way to manage concept and strategy development? AACSB: Reflective thinking

94) The job of translating target customer requirements into a working prototype is helped by a set of methods referred to as ________. A) quality function deployment B) quality control processes C) rapid prototyping D) marketing control E) control system formation

A Diff: 2 LO: 15.6: What is the best way to manage concept and strategy development? AACSB: Reflective thinking

136) Robert has heard about the latest cell phone from LG Electronics that features Wi-Fi hotspot technology. Wi-Fi hotspot is a technology that allows users to share mobile internet with other Wi-Fi enabled devices. Robert visits the company's Web site to read more about the phone. Robert is in the ________ stage of consumer-adoption process. A) interest B) awareness C) evaluation D) trial E) adoption

A Diff: 2 LO: 15.8: What factors affect the rate of diffusion and consumer adoption of newly launched products and services? AACSB: Analytical thinking; Application of knowledge

134) Adopters of new products move through five stages. These stages begin with ________ and follow with interest, evaluation, trial, and adoption. A) awareness B) investigation C) trial D) innovation E) diffusion

A Diff: 2 LO: 15.8: What factors affect the rate of diffusion and consumer adoption of newly launched products and services? AACSB: Reflective thinking

For new food products, marketing expenditures typically represent ________ of first-year sales.

57%

139) People who are technology enthusiasts, venturesome, and who enjoy tinkering with new products and mastering their intricacies are called ________. A) innovators B) early adopters C) early majority D) investigators E) experimenters

A Diff: 2 LO: 15.8: What factors affect the rate of diffusion and consumer adoption of newly launched products and services? AACSB: Reflective thinking

40) A manufacturer has invested $750,000 in a new product and wants to set a price to earn a 15 percent ROI. The cost per unit is $18 and the company expects to sell 50,000 units in the first year. Calculate the company's target-return price for this product. A) $20.25 B) $18.23 C) $18.10 D) $20.70 E) $25.50

A Page Ref: 397 Objective: 2 AACSB: Analytic skills Difficulty: Moderate

42) ________ pricing takes into account a host of inputs, such as the buyer's image of the product performance, the channel deliverables, the warranty quality, customer support, and attributes such as the supplier's reputation, trustworthiness, and esteem. A) Perceived-value B) Value C) Going-rate D) Auction-type E) Markup

A Page Ref: 398 Objective: 2 Difficulty: Easy

48) In ________, the firm bases its price largely on competitor's prices. A) going-rate pricing B) auction-type pricing C) markup pricing D) target-return pricing E) perceived-value pricing

A Page Ref: 401 Objective: 2 Difficulty: Easy

63) When Alan bought his car, the bank gave him 24 months to repay his car loan. But when Alan made a request to increase the time frame to 36 months, the bank granted the extension. The bank was willing to offer Alan a ________. A) longer payment term B) warranty contract C) service contract D) special customer price E) special event price

A Page Ref: 405 Objective: 3 AACSB: Analytic skills Difficulty: Moderate

30) Electro Locomotors (EL), Inc. engages in the manufacture, distribution, and marketing of custom vehicles in Brazil. Bono, a hybrid car brand that the company produces, is a market leader in the hybrid vehicles segment. With the increased global focus on nature friendly vehicles and governmental subsidies for companies that manufacture hybrid vehicles, competition has increased for flex-fuel cars like Bono. The heads of the R&D team at EL is in a creative session to generate ideas for improvements to Bono. They consider each part of the car and discuss the possibilities of improvements in each of them. Identify the creativity technique used here. A) attribute listing B) forced relationship analysis C) morphological analysis D) reverse assumption analysis E) mind mapping

A Page Ref: 579 Objective: 3 AACSB: Analytic skills Difficulty: Moderate

35) Morboro Energy, Inc. is a large chain of fuel stations in Europe. The company decided to expand its business by incorporating restaurants and automobile service stations as a part of its fuel outlets. This is an example of ________. A) lateral marketing B) brand extension C) market development D) internal marketing E) brand personification

A Page Ref: 580 Objective: 3 AACSB: Analytic skills Difficulty: Moderate

32) The ________ creativity technique begins in a thought that is written down, then proceeds to develop other thoughts that are linked to the first thought, and finally materializes a new idea by associating all the thoughts. A) mind mapping B) contextual analysis C) attribute listing D) reverse assumption analysis E) morphological analysis

A Page Ref: 580 Objective: 3 Difficulty: Easy

45) The trade-off approach may be easier to use when there are/is ________. A) many variables B) only a few alternatives C) observers that are biased D) possible offers that are limited E) only one highly desirable solution

A Page Ref: 584 Objective: 4 Difficulty: Easy

48) To estimate replacement sales, management must research the product's ________, the number of units that fail in year one, two, three, and so on. A) survival-age distribution B) lifecycle ratio C) obsolescence rate D) business turnover ratio E) product-performance usage

A Page Ref: 585 Objective: 4 Difficulty: Easy

50) ________ contribution lists the changes in income to other company products caused by the introduction of a new product. A) Supplementary B) Dragalong C) Gross D) Cumulative E) Net

A Page Ref: 586 Objective: 4 Difficulty: Easy

52) The job of translating target customer requirements into a working prototype is helped by a set of methods referred to as ________. A) quality function deployment B) quality control processes C) rapid prototyping D) marketing control E) control system formation

A Page Ref: 587 Objective: 4 Difficulty: Easy

66) Robert has heard about the latest cell phone from LG electronics that features Wi-Fi hotspot technology. Wi-Fi hotspot is a technology that allows users to share mobile internet with other Wi-Fi enabled devices. Robert visits the company's Web site to read more about the phone. Robert is in the ________ stage of consumer-adoption process. A) interest B) awareness C) evaluation D) trial E) adoption

A Page Ref: 591 Objective: 5 AACSB: Analytic skills Difficulty: Moderate

64) Adopters of new products move through five stages. These stages begin with ________ and follow with interest, evaluation, trial, and adoption. A) awareness B) investigation C) trial D) innovation E) diffusion

A Page Ref: 591 Objective: 5 Difficulty: Easy

69) People who are technology enthusiasts, venturesome, and who enjoy tinkering with new products and mastering their intricacies are called ________. A) innovators B) early adopters C) early majority D) investigators E) experimenters

A Page Ref: 591 Objective: 5 Difficulty: Easy

1) Appointing teams to manage customer-value-building processes and break down walls between departments is part of which of the following business practices? A) reengineering B) outsourcing C) benchmarking D) supplier partnering E) customer partnering

A Page Ref: 624 Objective: 1 Difficulty: Easy

5) Which of the following business practices involves reducing the number of organizational levels to get closer to the customer? A) flattening B) globalizing C) decentralization D) merging E) justifying

A Page Ref: 624 Objective: 1 Difficulty: Easy

11) ________ requires that everyone in the organization accept the concepts and goals of marketing and engage in choosing, providing, and communicating customer value. A) Internal marketing B) Corporate communications C) Integrated marketing communications D) Supply chain management E) Employee engagement

A Page Ref: 625 Objective: 2 AACSB: Analytic skills Difficulty: Easy

9) The role of marketing in the organization is changing. Traditionally marketers have played the roles of ________, charged with understanding customers' needs and transmitting their voice to various functional areas. A) middlemen B) opinion leaders C) clients D) influencers E) end consumers

A Page Ref: 625 Objective: 2 Difficulty: Easy

16) A product- or brand-management organization is characterized as a(n) ________ system. A) hub-and-spoke B) waterfall C) internal marketing D) top-down E) lateral

A Page Ref: 627 Objective: 2 Difficulty: Easy

15) A product-management organization makes sense if ________. A) the company's products are quite varied B) there are very few products in the company's portfolio C) the company is pursuing a low cost strategy D) the company's products satisfy similar customer needs E) the company is vertically integrated

A Page Ref: 627 Objective: 2 Difficulty: Moderate

19) There are three types of product-team structures. These are ________. A) vertical, triangular, and horizontal product teams B) vertical, horizontal, and circular product teams C) horizontal, vertical, and rectangular product teams D) horizontal, vertical, and flattened product teams E) vertical, rectangular, and circular product teams

A Page Ref: 628 Objective: 2 Difficulty: Easy

41) Which of the following represents the objective of a social marketing campaign aimed at changing people's behavior? A) Motivate people with obesity to eat healthy and exercise more often. B) Change public attitudes and stereotypes associated with people who are obese. C) Explain the different causes of obesity and how it can be prevented. D) Encourage people to participate in a walkathon aimed at promoting awareness about obesity. E) Attract people with obesity to sign up for a one-time free medical check up.

A Page Ref: 640 Objective: 3 AACSB: Analytic skills Difficulty: Moderate

43) The first step in the social marketing planning process is ________. A) determining the focus of the program B) selecting the target audience C) setting objectives and goals D) designing the market offering E) finding a source of funding

A Page Ref: 641 Objective: 3 AACSB: Ethical understanding and reasoning abilities Difficulty: Moderate

55) Your firm has experienced a decline in sales over the last three quarters. You have traced the problems to distribution inefficiencies. Which of the following should you track to ensure that the firm's distribution efficiency is maximized? A) average sales per point of sale B) sales from new products C) trial rate D) repurchase rate E) new customer gains

A Page Ref: 644 Objective: 5 AACSB: Analytic skills Difficulty: Moderate

58) To succeed in the future, marketing must ________. A) become more holistic B) build brands through promotion rather than performance C) be more departmental D) focus on free-spending E) rely more on mass marketing

A Page Ref: 645-648 Objective: 5 Difficulty: Moderate

Airfare (Scenario) The airfare for an economy class, one-way ticket from Los Angeles to New York is $500. Due to the recession, the airline manages to fill only 100 out of the 150 seats at $400 per seat. 64) How much of the sales performance gap is due to price decline? A) 28.5 percent B) 37 percent C) 2.5 percent D) 71.4 percent E) 0.4 percent

A Page Ref: 654 Objective: APP AACSB: Analytic skills Difficulty: Moderate

61) ________ looks at specific products, territories, and so forth that failed to produce expected sales. A) Microsales analysis B) Sales variance analysis C) Expense-to-sales analysis D) Full costing analysis E) Financial analysis

A Page Ref: 654 Objective: APP Difficulty: Easy

86) A(n) ________ appeal is a creative strategy that elaborates on a nonproduct-related benefit or image. A) logical B) transformational C) reasonable D) informational E) rational

B

Which of the following is an example of a retailer investment in technology that did not​ work? A. Electronic shelf labeling B. GPS on shopping carts C. Smart shopping carts D. Bidirectional infrared sensors E. ​In-store programming on plasma TVs

B

12) The most common form of marketing organization consists of ________ reporting to a marketing vice president who coordinates their activities. A) zonal managers B) functional specialists C) product managers D) area market specialists E) brand managers

B Page Ref: 625 Objective: 2 Difficulty: Easy

24) Which of the following is true regarding a matrix-management organization? A) It proves to be cost-effective in the long run. B) It often creates conflicts regarding authority and responsibility for marketing activities. C) It hampers the flow of information among marketing personnel. D) It fosters a strong sense of accountability for performance among product and market managers. E) It is best suited to companies that offer a small range of products to niche markets.

B Page Ref: 629 Objective: 2 Difficulty: Moderate

29) ________ gives products the appearance of being more environmentally friendly without living up to that promise. A) Ambush marketing B) Greenwashing C) Astroturfing D) Viral marketing E) Green politics

B Page Ref: 635 Objective: 3 AACSB: Ethical understanding and reasoning abilities Difficulty: Easy

30) ________ refers to the ability to meet humanity's needs without harming future generations. A) Greenwashing B) Sustainability C) Ecological footprinting D) Scalability E) Legal practice

B Page Ref: 635 Objective: 3 AACSB: Ethical understanding and reasoning abilities Difficulty: Easy

31) Cause-related marketing is part of ________. A) sustainability B) corporate societal marketing C) social media D) global marketing E) community marketing

B Page Ref: 636 Objective: 3 AACSB: Ethical understanding and reasoning abilities Difficulty: Easy

40) Which of the following represents the objective of a social marketing campaign aimed at changing people's values? A) Motivate people with obesity to eat healthy and exercise more often. B) Change public attitudes and stereotypes associated with people who are obese. C) Explain the different causes of obesity and how it can be prevented. D) Encourage people to participate in a walkathon aimed at promoting awareness about obesity. E) Help people with obesity implement to lifestyle changes.

B Page Ref: 640 Objective: 3 AACSB: Analytic skills Difficulty: Moderate

49) The purpose of profitability control is to ________. A) examine whether the planned results are being achieved B) examine where the company is making and losing money C) evaluate and improve the spending efficiency and impact of marketing expenditures D) examine whether the company is pursuing its best opportunities with respect to markets, products, and channels E) understand the efficiency of the sales force, advertising, sales promotion, and distribution

B Page Ref: 643 Objective: 5 Difficulty: Moderate

53) Which of the following is true regarding annual-plan control? A) This control process begins by identifying the causes of serious performance deviations. B) The tools used for this purpose are sales analysis, market share analysis, marketing expense-to-sales analysis, and financial analysis. C) The marketing controller has the primary responsibility for annual-plan control. D) Its purpose is to evaluate and improve the spending efficiency and impact of marketing expenditures. E) It measures profitability by product, territory, customer, segment, trade channel, and order size.

B Page Ref: 643-644 Objective: 5 Difficulty: Moderate

57) Which of the following is true regarding a marketing audit? A) It focuses primarily on a firm's macromarketing environment. B) It identifies the most-needed improvements and incorporates them into a corrective-action plan with short- and long-run steps. C) It is less effective at locating the real source of a problem than a functional audit. D) It analyzes only those marketing activities that have failed to produce adequate results. E) It relies solely on company managers for data and opinions.

B Page Ref: 645 Objective: 5 Difficulty: Moderate

62) Served market share ________. A) expresses the company's sales as a percentage of total market sales B) is sales as a percentage of the total sales to the market C) is market share in relationship to the largest competitor D) is always smaller than overall market share E) measures the relative contribution of different factors to a gap in sales performance

B Page Ref: 654 Objective: APP Difficulty: Moderate

71) The cost of the land where the plant which manufactures the common parts is set up falls under ________. A) traceable common costs B) non-traceable common costs C) variable costs D) manufacturing costs E) material costs

B Page Ref: 659 Objective: APP AACSB: Analytic skills Difficulty: Moderate

73) The cost of operating the common manufacturing facility is a(n) _____. A) opportunity cost B) traceable cost C) non-traceable cost D) sunk cost E) differential cost

B Page Ref: 659 Objective: APP AACSB: Analytic skills Difficulty: Moderate

Gabrielle is the chief marketing officer of Boyd Pharmaceuticals. She is meeting with Trent, the chief financial officer to decide on the company's marketing communications budget. They pore over the sales reports and other financial records and determine the amount of resources they can spare for marketing communications, after resources have been allocated to other functions, such as R&D, logistics, etc. What method did Gabrielle and Trent use to arrive at the marketing communications budget? A) objective-and-task method B) affordable method C) competitive-parity method D) activity-based method E) percentage-of-sales method

B) Affordable Method

________ is one of the sources of a spokesperson's credibility that refers to the specialized knowledge that he or she claims to possess. A) Trustworthiness B) Expertise C) Acquaintance D) Likability E) Professionalism

B) Expertise

A(n) ________ appeal is a creative strategy that elaborates on product or service attributes or benefits. A) aesthetic B) informational C) bandwagon D) emotional E) transformational

B) Informational

________ refers to a planning process designed to assure that all brand contacts received by a customer or prospect for a product, service, or organization are relevant to that person and consistent over time. A) Brand engagement B) Integrated marketing communications C) Market research D) Customerization E) Marketing research

B) Integrated marketing communications

________ refers to the means by which firms attempt to inform, persuade, and remind consumers — directly or indirectly — about the products and brands they sell. A) Human resource development B) Marketing communications C) Financial management D) Operations management E) Planning

B) Marketing Communications

Which of the following elements of the marketing communications mix includes a variety of programs directed internally to employees of the company or externally to consumers, other firms, the government, and media to promote or protect a company's image or its individual product communications? A) direct marketing B) public relations and publicity C) personal selling D) advertising E) sales promotion

B) Public relations and publicity

________ is a measure of communications effectiveness that describes the percentage of target market exposed to a communication. A) Frequency B) Reach C) Width D) Depth E) Range

B) Reach

________ includes all the activities in selling goods or services directly to final consumers for personal, non-business use. A) Wholesaling B) Retailing C) Procurement D) Promoting E) Warehousing

B) Retailing

One of the possible objectives of marketing communications is helping consumers evaluate a brand's perceived ability to meet a currently relevant need. Which of the following relevant brand needs is most likely emphasized by an advertisement for a luxury car? A) problem removal B) sensory gratification C) normal depletion D) intellectual stimulation E) problem avoidance

B) Sensory Gradification

Which of the following is an example of direct marketing? A) E&OE sells its herbal skincare products exclusively through its standalone stores. B) TCJ is a telemarketing firm that sells products from a number of different suppliers. C) Jayne's sells most of its products to customers through home sales parties. D) J3 is a storeless retailer that organizes the retail activity of the employees of four firms. E) Reynold's tries to minimize its staff costs by installing vending machines in its stores.

B) TCJ is a telemarketing firm that sells products from a number of different suppliers.

Stores are using ________ to measure a product's handling costs from the time it reaches the warehouse until a customer buys it in the retail store. A) electronic data interchange (EDI) B) direct product profitability (DPP) C) radio-frequency identification (RFID) D) global positioning systems (GPS) E) compounded annual grown rate (CAGR)

B) direct product profitability (DPP)

E&OE produces and markets its own brand of skincare products using herbal remedies and natural ingredients through standalone stores as well as an online portal. E&OE is a(n) ________ retailer. A) off-price B) specialty C) discount D) department E) extreme value

B) specialty

Which of the following is the strongest differentiator for brick-and-mortar stores who want to emphasize their superiority over online retailers? A) product quality B) the shopping experience C) product range D) pricing E) the retailer's reputation

B) the shopping experience

A stock order point of 10 means ordering the product ________. A) every 10 days B) when stock falls to 10 units C) every 10 units D) when stock falls to 9 units E) in batches of 10 items

B) when stock falls to 10 units

Which of the following levels of retailer service is associated with high staffing​ costs, higher proportions of specialty goods and​ slower-moving items, and many costly​ services? A. Limited service B. Full service C. ​Self-service D. ​Self-selection E. Pure services

B. Full Service

51) Differentiate between brokers and agents.

Brokers and agents both perform wholesaling functions. They facilitate buying and selling, on commission of 2 to 6 percent of the selling price and generally specialize by product line or customer type. Brokers bring buyers and sellers together and assist in negotiation; they are paid by the party hiring them — food brokers, real estate brokers, insurance brokers. Agents represent buyers or sellers on a more permanent basis. Most manufacturers' agents are small businesses with a few skilled salespeople: Selling agents have contractual authority to sell a manufacturer's entire output; purchasing agents make purchases for buyers and often receive, inspect, warehouse, and ship merchandise; commission merchants take physical possession of products and negotiate sales.

144) What is a marketing audit? Explain the four characteristics of a marketing audit.

A marketing audit is a comprehensive, systematic, independent, and periodic examination of a company's or business unit's marketing environment, objectives, strategies, and activities, with a view to determining problem areas and opportunities and recommending a plan of action to improve the company's marketing performance. A marketing audit should be: • comprehensive — it covers all the major marketing activities of a business • systematic — it is an orderly examination of the organization's macro- and micromarketing environments, marketing objectives and strategies, marketing systems, and specific activities • independent — outside consultants bring the necessary objectivity, broad experience in a number of industries, familiarity with the industry being audited, and undivided time and attention • periodic — firms typically initiate marketing audits only after failing to review their marketing operations during good times, with resulting problems. A periodic marketing audit can benefit companies in good health as well as those in trouble

106) Because shelf space is scarce, many supermarkets now charge a ________ for accepting a new brand, to cover the cost of listing and stocking it. A) retainer B) slotting fee C) residual fee D) contingent fee E) royalty

B

109) Which of the following is true regarding a marketing audit? A) It focuses primarily on a firm's macromarketing environment. B) It identifies the most-needed improvements and incorporates them into a corrective-action plan with short- and long-run steps. C) It is less effective at locating the real source of a problem than a functional audit. D) It analyzes only those marketing activities that have failed to produce adequate results. E) It relies solely on company managers for data and opinions.

B

144) Stacey wants to use a highly relevant, engaging, and implicit communications mix mode to use a "soft sell" approach for her new makeup line. The communications mix mode that has these characteristics is ________. A) sales promotions B) events and experiences C) advertising D) direct and interactive marketing E) personal selling

B

147) ________ refers to a planning process designed to assure that all brand contacts received by a customer or prospect for a product, service, or organization are relevant to that person and consistent over time. A) Brand engagement B) Integrated marketing communications C) Market research D) Customerization E) Marketing research

B

26) Which of the following is true regarding a matrix-management organization? A) It proves to be cost-effective in the long run. B) It often creates conflicts regarding authority and responsibility for marketing activities. C) It hampers the flow of information among marketing personnel. D) It fosters a strong sense of accountability for performance among product and market managers. E) It is best suited to companies that offer a small range of products to niche markets.

B

29) A ________ is a wholesaler-sponsored group of independent retailers engaged in bulk buying and common merchandising. A) retailer cooperative B) voluntary chain C) consumer cooperative D) merchandising conglomerate E) franchise organization

B

4) Total Beverages, a maker of fruit juices and health drinks, recently launched a new brand of packaged drinking water called AquaPure. In order to induce distributors to carry the product, Total offers all its intermediaries a free refrigerator to store bottles of AquaPure. This is an example of a ________. A) consumer promotion B) push strategy C) backward flow D) reverse flow E) pull strategy

B

84) Which of the following is the strongest differentiator for brick-and-mortar stores who want to emphasize their superiority over online retailers? A) product quality B) the shopping experience C) product range D) pricing E) the retailer's reputation

B

1) ________ refers to the means by which firms attempt to inform, persuade, and remind consumers — directly or indirectly — about the products and brands they sell. A) Human resource development B) Marketing communications C) Financial management D) Operations management E) Planning

B

102) The ________ is primarily responsible for strategic control. A) line and staff management B) marketing auditor C) marketing controller D) middle management E) BAMT

B

105) Which of the following is true regarding annual-plan control? A) This control process begins by identifying the causes of serious performance deviations. B) The tools used for this purpose are sales analysis, market share analysis, marketing expense-to-sales analysis, and financial analysis. C) The marketing controller has the primary responsibility for annual-plan control. D) Its purpose is to evaluate and improve the spending efficiency and impact of marketing expenditures. E) It measures profitability by product, territory, customer, segment, trade channel, and order size.

B

132) JBJ Sports is a leading sporting goods manufacturer from Atlanta. It has recently initiated a program in association with the Food4Kids Foundation, which raises funds to promote nutritional awareness of young children. The company plans to donate $1 for every item it sells in the North American market. Also, JBJ has roped in professional sports teams associated with it to wear the Food4Kids logo on their team gear. Which of the following best describes the motivation for JBJ's involvement in the program? A) entertaining key clients or rewarding key employees B) expressing commitment to the community or on social issues C) permitting merchandising or promotional opportunities D) stimulating quicker or greater purchase of particular brands E) identifying with a particular target market or lifestyle

B

22) The ________ marketing sales system works by recruiting independent businesspeople who act as distributors. A) catalog B) multilevel C) direct-response D) corporate E) direct

B

58) Which of the following is a disadvantage of using newsletters as an advertising medium? A) low audience selectivity B) high chances of runaway costs C) lack of adequate control D) relatively high costs E) lack of interactive possibilities

B

82) One of the possible objectives of marketing communications is helping consumers evaluate a brand's perceived ability to meet a currently relevant need. Which of the following is a positively oriented relevant brand need? A) problem removal B) social approval C) normal depletion D) problem avoidance E) incomplete satisfaction

B

82) Stores are using ________ to measure a product's handling costs from the time it reaches the warehouse until a customer buys it in the retail store. A) electronic data interchange (EDI) B) direct product profitability (DPP) C) radio-frequency identification (RFID) D) global positioning systems (GPS) E) compounded annual grown rate (CAGR)

B

83) A new firm typically starts as a local operation selling in a fairly circumscribed market by ________. A) finding and developing new intermediaries B) using a few existing intermediaries C) forming partnerships with the market leader D) creating a special channel E) forming partnerships with other firms

B

97) In consumer-goods market testing, the company seeks to estimate four variables. These four variables are: trial, first repeat, adoption, and ________. A) guaranteed response B) price sensitivity C) purchase frequency D) usage convenience E) preferential treatment

C Diff: 1 LO: 15.6: What is the best way to manage concept and strategy development? AACSB: Reflective thinking

7) In ________ retailing, salespeople are ready to assist in every phase of the "locate-compare-select" process. A) self-service B) self-selection C) full-service D) limited service E) limited-selection

C Diff: 1 LO: 18.1: What major types of marketing intermediaries occupy this sector? AACSB: Analytical thinking

114) ________ are independently owned businesses that take title to the merchandise they handle. They are full-service and limited-service jobbers, distributors, and mill supply houses. A) Brokers B) Agents C) Merchant wholesalers D) Specialized wholesalers E) Retailers' branches

C Diff: 1 LO: 18.5: What are some of the important issues in wholesaling? AACSB: Reflective thinking

17) After creating a product prototype, a company tests it within the firm to see how it performs in different applications. The company refines the prototype to correct the mistakes found in in-house testing. What should be the next step? A) commercializing the product B) performing concept testing C) conducting beta testing with customers D) creating a marketing strategy for the product E) performing business analysis

C Diff: 2 LO: 15.2: What challenges does a company face in developing new products and services? AACSB: Application of knowledge

57) ________ works by listing all the normal assumptions about an entity and then turning them around. A) Lateral marketing B) Attribute listing C) Reverse assumption analysis D) Forced relationships E) Morphological analysis

C Diff: 2 LO: 15.4: What are the main stages in developing new products and services? AACSB: Reflective thinking

89) ________ is a "hybrid" data collection technique that combines self-explicated importance ratings with pair-wise trade-off tasks. A) Sales wave analysis B) Simulative data testing C) Adaptive conjoint analysis D) Business analysis E) Exponential data collection

C Diff: 2 LO: 15.6: What is the best way to manage concept and strategy development? AACSB: Reflective thinking

93) The highest loss a project can create is called ________. A) rapid prototyping income B) payback income C) maximum investment exposure D) incremental yearly exposure E) cannibalized income

C Diff: 2 LO: 15.6: What is the best way to manage concept and strategy development? AACSB: Reflective thinking

121) Poga International, a multinational beverage corporation identifies that one of its competitors is launching an apple flavored drink. The company decides to launch an apple flavor brand along with its competitor. What timing strategy is used here? A) first entry B) blunt entry C) parallel entry D) late entry E) exchange entry

C Diff: 2 LO: 15.7: What is the best way to manage the commercialization of new products? AACSB: Analytical thinking; Application of knowledge

135) Identify the stage in the consumer adoption process in which the customer is stimulated to seek information about the innovation. A) awareness B) adoption C) interest D) evaluation E) trial

C Diff: 2 LO: 15.8: What factors affect the rate of diffusion and consumer adoption of newly launched products and services? AACSB: Reflective thinking

148) Which of the following terms refers to the degree to which the innovation can be tried on a limited basis? A) compatibility B) relative advantage C) divisibility D) communicability E) complexity

C Diff: 2 LO: 15.8: What factors affect the rate of diffusion and consumer adoption of newly launched products and services? AACSB: Reflective thinking

72) In the corporate headquarters of a supermarket chain, ________ are responsible for developing brand assortments and listening to salespersons' presentations. A) central buyers B) brokers C) specialist buyers D) agents E) specialized wholesalers

C Diff: 2 LO: 18.3: What marketing decisions do marketing intermediaries make? AACSB: Reflective thinking

104) ________ are unbranded, plainly packaged, less expensive versions of common products such as spaghetti, paper towels, and canned peaches. A) Common carriers B) Shills C) Generics D) Private labels E) Marques

C Diff: 2 LO: 18.4: What does the future hold for private label brands? AACSB: Analytical thinking

137) E&OE is trying to minimize its inventory costs, which are extremely high. The company has realized that it can achieve this by maintaining a near-zero inventory and producing more products only once it is ordered. Which of the following is true for E&OE? A) Inventory costs are lower than order-processing costs. B) Running costs are higher than inventory-carrying costs. C) Setup costs for the products are low. D) Order-processing costs are high. E) Order-processing costs are lower than setup costs.

C Diff: 2 LO: 18.6: What are some important issues in logistics? AACSB: Analytical thinking; Application of knowledge

20) Which of the following is an example of direct selling? A) E&OE sells its herbal skincare products exclusively through its standalone stores. B) TCJ is a telemarketing firm that sells products from a number of different suppliers. C) Jayne's sells most of its products to customers through home sales parties. D) J3 is an online shopping portal where customers can buy directly from manufacturers. E) Reynold's tries to minimize its staff costs by installing vending machines in its stores.

C Diff: 3 LO: 18.1: What major types of marketing intermediaries occupy this sector? AACSB: Analytical thinking; Application of knowledge

7) A company has developed the prototype of a mobile phone which it plans to launch in the next few months. The phone comes equipped with the most advanced technological features. As part of its test marketing efforts, it allows customers to examine and use the prototype and also gathers feedback regarding product features and price. The results of this test marketing effort show that customers are willing to pay at least $500, considering the phone's various features. As such, the company has found out about the customers' ________. A) last paid price B) expected future price C) lower-bound price D) upper-bound price E) typical price

C Page Ref: 387 Objective: 1 AACSB: Analytic skills Difficulty: Moderate

4) The reservation price or the maximum that most consumers would pay for a given product is known as the ________ price. A) expected future B) usual discounted C) upper-bound D) typical E) historical competitor

C Page Ref: 387 Objective: 1 Difficulty: Easy

12) A firm that is plagued with overcapacity, intense competition, or changing wants would do better if it pursues ________ as its major objective. A) market skimming B) product-quality leadership C) survival D) profit maximization E) market penetration

C Page Ref: 389 Objective: 2 Difficulty: Easy

34) The decline in the average cost of production with accumulated production experience is called the ________. A) demand curve B) supply chain C) learning curve D) value chain E) indifference curve

C Page Ref: 393 Objective: 2 Difficulty: Easy

38) Which of the following is the most elementary pricing method? A) value pricing B) going-rate pricing C) markup pricing D) target-return pricing E) perceived-value pricing

C Page Ref: 396 Objective: 2 Difficulty: Easy

46) Matt's retail store offers all its products at $2 lesser than its competitors throughout the year. The store never runs any promotional campaigns or offers any additional special discounts. Matt's retail store is following a(n) ________. A) auction-type pricing policy B) target-plus pricing policy C) everyday low pricing policy D) high-low pricing policy E) going-rate pricing policy

C Page Ref: 400 Objective: 2 AACSB: Analytic skills Difficulty: Moderate

53) In which of the following forms of countertrade do buyers and sellers directly exchange goods, with no money and no third party is involved? A) buyback arrangements B) offsets C) barter D) sealed bids E) compensation deals

C Page Ref: 404 Objective: 3 Difficulty: Easy

57) ________ are offered by a manufacturer to trade-channel members if they will perform certain functions, such as selling, storing, and record keeping. A) Consumer promotions B) Quantity discounts C) Functional discounts D) Seasonal discounts E) Trade-in allowances

C Page Ref: 404 Objective: 3 Difficulty: Easy

61) ________ reward dealers for participating in advertising and sales support programs. A) Functional discounts B) Trade discounts C) Promotional allowances D) Rebates E) Quantity discounts

C Page Ref: 404 Objective: 3 Difficulty: Easy

10) After creating a product prototype, a company tests it within the firm to see how it performs in different applications. The company refines the prototype to correct the mistakes found in in-house testing. What should be the next step? A) commercializing the product B) performing concept testing C) conducting beta testing with customers D) creating a marketing strategy for the product E) performing business analysis

C Page Ref: 574 Objective: 1 Difficulty: Easy

11) Identify a shortcoming of giving the responsibility of new product development to the product managers of a company. A) They would not be familiar with the industry standards. B) Product managers would not have an operational focus. C) Product managers are often busy managing existing lines. D) They will find it difficult to gain support from employees. E) Product managers are less likely to use participative management.

C Page Ref: 574 Objective: 2 Difficulty: Easy

15) The ________ used by companies divides the innovation process into stages, with a checkpoint at the end of each stage. A) reverse assumption analysis technique B) skunkworks system C) stage-gate system D) spiral development process E) new-product department technique

C Page Ref: 574 Objective: 2 Difficulty: Easy

18) Sweet Treats is a startup confectionery manufacturer that sells chocolates, toffees, marshmallows, jelly candies, and so on. Should the firm adopt a push or a pull strategy? Explain your answer.

Considering that Sweet Treats is a startup, its products are likely to have low brand loyalty. Moreover, its products are likely to be impulse items. Thus, a push strategy would be more suitable.

100) Who is primarily responsible for efficiency control? A) BAMT B) marketing auditor C) middle management D) line and staff management E) top management

D

105) ________ is a consumer promotion tool that involves inviting prospective purchasers to try the product without cost in the hope that they will buy. A) Sampling B) Premium C) Cross-promotion D) Free trial E) Rebate

D

106) Which of the following is an example of a distribution metric used for measuring the performance of marketing plans? A) effective reach B) customer acquisition costs C) market share D) stocks cover in days E) response rate

D

150) What is the trade-off in return for the benefits associated with an integrated multichannel marketing system? A) increased market coverage B) customers who buy in one channel are more profitable than customers who buy across different channels C) lower channel costs D) two or more channels may end up competing for the same customer E) customized selling

D

4) Discount stores that try to keep prices as low as possible are more likely to function using ________ operations. A) limited service B) self-selection C) full-service D) self-service E) limited-selection

D

104) Marketing effectiveness rating instruments and marketing audits are approaches to ________ control. A) annual-plan B) profitability C) efficiency D) strategic E) statistical

D

148) The easiest measure of marketing public relations effectiveness is the ________. A) resultant effect on the company's sales figures B) effect it has on its market capitalization C) number of exposures carried by the media D) changes observed in consumers' brand knowledge E) impact it has on the company's market share

C

20) Which of the following is an example of direct selling? A) E&OE sells its herbal skincare products exclusively through its standalone stores. B) TCJ is a telemarketing firm that sells products from a number of different suppliers. C) Jayne's sells most of its products to customers through home sales parties. D) J3 is an online shopping portal where customers can buy directly from manufacturers. E) Reynold's tries to minimize its staff costs by installing vending machines in its stores.

C

41) The weighted number of exposures (WE) of an advertising message over a given medium is given by ________. A) WE = reach × frequency B) WE = (reach × frequency) / impact C) WE = reach × frequency × impact D) WE = (reach + frequency) / impact E) WE = frequency / reach

C

5) Comparative advertising works best when ________. A) it elicits cognitive and behavioral motivations simultaneously B) the firm is trying to minimize brand dilution C) consumers are processing advertising in a detailed, analytical mode D) it elicits affective motivation, followed by cognitive motivation E) the advertising message uses negative fear appeals

C

5) Reynold's is a grocery chain that has always catered to mid-market customers. However, the owner, Mal, has noticed that an influx of new residents are buying mostly the lower-cost and discounted products. To attract customers, Mal decides to make a gradual switch to the discount store format, but to do this, he will have to cut costs wherever possible. Which of the following types of services should Mal avoid in order to lower costs? A) limited service B) self-selection C) full-service D) self-service E) limited-selection

C

5) Which of the following is an example of a trade promotion? A) free samples B) discount coupons C) display allowances D) contests for sales reps E) premiums

C

65) Which of the following sequences accurately represents the hierarchy-of-effects model of marketing communications? A) attention-interest-desire-action B) awareness-interest-evaluation-trial-adoption C) awareness-knowledge-liking-preference-conviction-purchase D) exposure-reception-cognitive response-attitude-intention-behavior E) knowledge-persuasion-decision-implementation-confirmation

C

65) Which of the following should be done to increase the likelihood that social marketing programs will be successful? A) Choose target markets that are the least ready to respond. B) Promote multiple, doable behaviors in clear, simple terms. C) Make it easy to adopt the behavior. D) Adopt a soft sell approach, as opposed to attention grabbing messages. E) Explain the benefits in an exaggerated manner.

C

72) In the corporate headquarters of a supermarket chain, ________ are responsible for developing brand assortments and listening to salespersons' presentations. A) central buyers B) brokers C) specialist buyers D) agents E) specialized wholesalers

C

72) ________ is continuous advertising at low levels, reinforced periodically by waves of heavier activity. A) Flighting B) Frequency capping C) Pulsing D) Concentration E) Session capping

C

74) Share of ________ represents the proportion of company advertising of a product to all advertising of that product. A) wallet B) mind C) voice D) market E) cost

C

8) ________ advertising aims to convince current purchasers that they made the right choice. A) Persuasive B) Informational C) Reinforcement D) Reminder E) Comparative

C

80) Which of the following is a postpurchase service offered by retailers? A) accepting orders over the telephone B) advertising and window displays C) delivery to the customer's doorstep D) general information E) interior decoration of the retail outlet

C

93) Which of the following is an example of a trade promotion? A) free samples B) discount coupons C) display allowances D) contests for sales reps E) premiums

C

94) Which of the following personal communications channels consist of family members, neighbors, friends, and associates talking to target buyers? A) expert channels B) advocate channels C) social channels D) formal channels E) sponsored channels

C

11) Which of the following is an example of a limited-service retailer? A) Customers at TAB pick out the products they want and pay at the checkout counter. B) Salespeople at Eli are always on hand to offer shoppers advice and assistance in choosing products. C) Mina's allows customers to checkout their own purchases in order to reduce the staff count. D) As CLO's offers a number of merchandise-return privileges, customers need information and assistance while shopping. E) Alison's stocks a number of specialty products and the store's salespeople generally help shoppers with all their needs.

D

111) Which of the following is likely to be an important trend in marketing in the future? A) marketing intuition B) free-spending marketing C) manual marketing D) marketing science E) mass marketing

D

115) ________ serve bulk industries such as coal, lumber, and heavy equipment, assuming title and risk from the time an order is accepted to its delivery. A) Producers' cooperatives B) Cash and carry wholesalers C) Truck wholesalers D) Drop shippers E) Rack jobbers

D

121) A(n) ________ market share of exactly 100 percent means that a company is tied for the market lead. A rise in relative market share means the company is gaining on its leading competitor. A) overall B) served C) potential D) relative E) actual

D

148) Winstar is a large-scale manufacturer with more than a hundred partners across the globe. When making decisions concerning distribution and channel optimization, the company invites members from its channel partners to be part of its advisory committee. This helps the company maintain harmony with its partners. Which of the following conflict resolution techniques is Winstar using? A) diplomatic counselling B) mediation C) arbitration D) co-optation E) joint membership

D

23) In ________, a salesperson goes to the home of a host who has invited friends, demonstrates the products, and takes orders. A) catalog marketing B) franchising C) direct-response selling D) network marketing E) direct marketing

D

58) ________ marketing by nonprofit or government organizations furthers a cause. A) Corporate societal B) Brand C) Causal D) Social E) Place

D

62) Which of the following represents the objective of a social marketing campaign aimed at changing people's actions? A) Motivate people with obesity to eat healthy and exercise more often. B) Change public attitudes and stereotypes associated with people who are obese. C) Explain the different causes of obesity and how it can be prevented. D) Encourage people to participate in a walkathon aimed at promoting awareness about obesity. E) Help people with obesity to implement lifestyle changes.

D

79) When Apple introduced the iPod in October, 2001, it was the first-of-its-kind product that offered sizable storage capacity for songs and a portable device that was not seen before in the market. Which of the following is most likely to have been the marketing communications objective for the iPod at the time of its introduction? A) developing brand awareness B) building customer traffic C) enhancing purchase actions D) establishing product category E) enhancing firm image

D

9) The predominant response function for advertising is often concave, but when it is S-shaped, ________. A) sales are flat, and advertising does not generate any sales impact B) any increase in advertising spending results in a proportionally positive increase in sales C) any increase in advertising spending results in a proportionally negative decrease in sales D) some positive amount of advertising is necessary to generate any sales impact, but sales increases eventually flatten out E) advertising is not necessary to generate any sales impact

D

90) ________ is a source of a spokesperson's credibility that describes how objective and honest the spokesperson is perceived to be. A) Likability B) Expertise C) Experience D) Trustworthiness E) Compassion

D

96) According to consumer-packaged goods companies, which of the following effects is attributed to the heavy use of sales promotion? A) increased brand loyalty B) focus on long-run marketing planning C) improved brand-quality image D) increased price sensitivity E) greater coupon redemption rates

D

A) impact on market share of sponsor B) amount of time a brand is clearly visible on a television screen C) amount of relevant newsprint mentioning the sponsor D) influence on consumers' brand knowledge of the sponsor E) net impact on the sponsor's bottom line

D

Independent Grocers Alliance​ (IGA) is an example of a​ ________, or a​ wholesaler-sponsored group of independent retailers engaged in bulk buying and common merchandising. A. consumer cooperative B. retailer cooperative C. franchise organization D. voluntary chain E. merchandising conglomerate

D

Which of the following is least effective for manufacturers when competing against or collaborating with private​ labels? A. Create winning value propositions. B. Partner effectively. C. Innovate brilliantly. D. Match prices. E. Fight selectively.

D

department store chain to target the upcoming Thanksgiving weekend. Which of the following advertising timing patterns is best suited for running these ads? A) continuity B) flighting C) pulsing D) concentration E) frequency capping

D

likely objective of Mountain Dew's sponsorship of these events? A) to enhance corporate image B) to express commitment to the community or on social issues C) to entertain key clients or reward key employees D) to create perceptions of key brand image associations E) to become part of a personally relevant moment in consumers' lives

D

2) New-to-the-world products are ________. A) low-cost products designed to obtain an edge in highly competitive markets B) new product enhancements that supplement established products C) new versions of an existing product that has been less successful D) new products that create an entirely new market E) existing products that are targeted to new geographical markets

D Diff: 1 LO: 15.1: How can new products be categorized? AACSB: Reflective thinking

4) Discount stores that try to keep prices as low as possible are more likely to function using ________ operations. A) limited service B) self-selection C) full-service D) self-service E) limited-selection

D Diff: 1 LO: 18.1: What major types of marketing intermediaries occupy this sector? AACSB: Reflective thinking

60) Staples is a giant retailer that concentrates on selling office supplies. Staples is an example of a(n) ________. A) ambush marketer B) supercenter C) megamarketer D) category killer E) guerilla marketer

D Diff: 1 LO: 18.2: What major changes are occurring in the modern retail marketing environment with respect to competitive market structure and technology? AACSB: Application of knowledge

3) Which of the following firms is most likely to seek radical innovation? A) a traditional publishing company B) a small scale FMCG company C) a firm that procures and markets wheat D) a high-tech firm in telecommunications E) a firm that sells mineral water

D Diff: 2 LO: 15.1: How can new products be categorized? AACSB: Reflective thinking

84) Brown & Smith Inc. engages in the design, development, making, and retail selling of designer jewelry in North America. Before approving a new design, the company draws it on a computer and then produces models to show potential consumers and get their reactions. This allows the company to analyze the possible customer reaction. Identify the concept testing method used here. A) conjoint analysis B) perceptual mapping C) virtual reality testing D) rapid prototyping E) digital fabrication

D Diff: 2 LO: 15.6: What is the best way to manage concept and strategy development? AACSB: Analytical thinking; Application of knowledge

100) Which of the following is usually referred to as a full-blown test market? A) an internal focus group B) a discussion group C) a country D) a city or a few cities E) a laboratory

D Diff: 2 LO: 15.6: What is the best way to manage concept and strategy development? AACSB: Reflective thinking

85) Consumer preferences for alternative product concepts can be measured through ________, a method for deriving the utility values that consumers attach to varying levels of a product's attributes. A) concept testing B) perceptual mapping C) gap level analysis D) conjoint analysis E) morphological analysis

D Diff: 2 LO: 15.6: What is the best way to manage concept and strategy development? AACSB: Reflective thinking

96) ________ testing is a customer test that involves placing the product within the firm to see how it performs in different applications. A) Beta B) Research C) Sales-wave D) Alpha E) Simulated

D Diff: 2 LO: 15.6: What is the best way to manage concept and strategy development? AACSB: Reflective thinking

99) Which of the following is an advantage of sales-wave research? A) It gives surprisingly accurate results on advertising effectiveness. B) It allows the brand to gain favorable shelf position. C) It guarantees immediate brand recognition and high sales volume. D) It can be implemented quickly with a fair amount of security. E) It easily creates a buzz in the minds of customers.

D Diff: 2 LO: 15.6: What is the best way to manage concept and strategy development? AACSB: Reflective thinking

138) Daryl convinced his prospective client that Car A was the best for him. But, the client insisted that the car cost him a good $10,000 more than Car B, the one which he was thinking of buying. Daryl told him that the amount he would have to spend on the fuel, insurance, repairs, and maintenance for Car B would be 5 times more than what he would have to spend on Car A. Finally convinced, the client consented to buy Car A. What technique did Daryl use to convince his customer?

Daryl convinced his customer that Car A offers him the lowest total cost of ownership. Page Ref: 391 Objective: 2 AACSB: Analytic skills Difficulty: Moderate

11) After determining its pricing objectives, what is the next logical step a firm should take in setting its pricing policy? A) It should analyze its competitors' costs, prices, and offers. B) It should select its pricing method. C) It should select its final price. D) It should determine the demand for its product. E) It should estimate the cost of its product.

D Page Ref: 389 Objective: 2 Difficulty: Easy

19) Market skimming pricing makes sense under all the following conditions, EXCEPT ________. A) if a sufficient number of buyers have a high current demand B) if the unit costs of producing a small volume are high enough to cancel the advantage of charging what the traffic will bear C) if the high initial price does not attract more competitors to the market D) if consumers are likely to delay buying the product until its price drops E) if the high price communicates the image of a superior product

D Page Ref: 390 Objective: 2 Difficulty: Easy

41) An umbrella manufacturing company's fixed costs are $275,000. The variable cost per unit is $5 and each umbrella is sold at $10. How many units should the firm sell in order to break even? A) 18000 B) 5500 C) 27500 D) 55000 E) 1819

D Page Ref: 397 Objective: 2 AACSB: Analytic skills Difficulty: Moderate

58) When hotels, motels, and airlines offer discounts in slow selling periods, they are said to be offering ________. A) trade discounts B) quantity discounts C) functional discounts D) seasonal discounts E) trade-in allowances

D Page Ref: 404 Objective: 3 Difficulty: Easy

69) When Coca-Cola carries a different price depending on whether the consumer purchases it in a fine restaurant, a fast-food restaurant, or a vending machine, then this form of price discrimination is known as ________. A) product-form pricing B) loss-leader pricing C) special event pricing D) channel pricing E) location pricing

D Page Ref: 406 Objective: 3 Difficulty: Easy

74) For price discrimination to work ________. A) the market must be segmentable and the segments must show similar intensities of demand B) members in the lower-price segment must be able to resell the product to the higher-price segment C) competitors must be able to undersell the firm in the higher-price segment D) the practice must not breed customer resentment and ill will E) the extra revenue derived from price discrimination must not exceed the cost of segmenting and policing the market

D Page Ref: 407 Objective: 3 Difficulty: Moderate

2) New-to-the-world products are ________. A) low-cost products designed to obtain an edge in highly competitive markets B) new product enhancements that supplement established products C) new versions of an existing product that has been less successful D) new products that create an entirely new market E) existing products that are targeted to new geographical markets

D Page Ref: 570 Objective: 1 Difficulty: Easy

3) Which of the following firms is most likely to seek radical innovation? A) a traditional publishing company B) a small scale FMCG company C) a firm that procures and markets wheat D) a high-tech firm in telecommunications E) a firm that sells mineral water

D Page Ref: 570 Objective: 1 Difficulty: Easy

4) Most new-product activities are devoted to ________. A) changing the target markets B) developing new-to-the-world products C) introducing backward integration D) improving existing products E) changing the existing market dynamics

D Page Ref: 570 Objective: 1 Difficulty: Easy

17) Which of the following questions is answered during the business analysis of ideas? A) Will this product meet our profit goals? B) Have we got a technically and commercially sound product? C) Can we find a cost-effective, affordable marketing strategy? D) Can this product meet sales expectations? E) Can we find a good concept consumers say they would try?

D Page Ref: 575 Objective: 2 Difficulty: Easy

18) During which of the following stages in new product development decision making do managers analyze if they can find a good thought consumers say they would try? A) idea generation B) idea screening C) business analysis D) concept development and testing E) product development and testing

D Page Ref: 575 Objective: 2 Difficulty: Easy

36) A ________ error occurs when the company dismisses a good idea. A) probability B) performance C) double counting D) DROP E) GO

D Page Ref: 580 Objective: 3 Difficulty: Easy

37) A(n) ________ is a possible product the company might offer to the market. A) test brand B) alpha product C) beta version product D) product idea E) product concept

D Page Ref: 581 Objective: 4 Difficulty: Easy

41) Brown & Smith, Inc. engages in the design, development, making, and retail of designer jewelry in North America. Before approving a new design of jewelry the company draws it on a computer and then produces models to show potential consumers for their reactions. This allows the company to analyze the possible customer reaction. Identify the concept testing method used here. A) conjoint analysis B) perceptual mapping C) virtual reality testing D) rapid prototyping E) digital fabrication

D Page Ref: 582 Objective: 4 AACSB: Analytic skills Difficulty: Moderate

42) Consumer preferences for alternative product concepts can be measured through ________, a method for deriving the utility values that consumers attach to varying levels of a product's attributes. A) concept testing B) perceptual mapping C) gap level analysis D) conjoint analysis E) morphological analysis

D Page Ref: 583 Objective: 4 Difficulty: Easy

54) ________ is a customer test that involves placing the product within the firm to see how it performs in different applications. A) Beta testing B) Research testing C) Sales-wave testing D) Alpha testing E) Simulated testing

D Page Ref: 587 Objective: 4 Difficulty: Easy

62) Some firms might delay the launch of their products until after the competitor has borne the cost of educating the market. Such an entry is called ________ entry. A) strategic pay off B) parallel C) balancing D) late E) compensating

D Page Ref: 590 Objective: 4 Difficulty: Easy

68) Which of the following is the mental step in consumer-adoption process where the customer starts considering whether to try the innovation? A) adoption B) trial C) awareness D) evaluation E) interest

D Page Ref: 591 Objective: 5 Difficulty: Easy

75) Relative advantage of an innovation refers to the degree to which ________. A) it matches the values and experiences of the individuals B) it is difficult to understand or use C) it can be tried on a limited basis D) it appears superior to existing products E) the benefits of use are observable or describable to others

D Page Ref: 592 Objective: 5 AACSB: Analytic skills Difficulty: Moderate

3) ________ involves studying "best practice companies" to improve performance. A) Empowering B) Globalizing C) Flattening D) Benchmarking E) Focusing

D Page Ref: 624 Objective: 1 Difficulty: Easy

6) Which of the following business practices focuses specifically on designing the organization and setting up processes to respond quickly to changes in the environment? A) benchmarking B) outsourcing C) focusing D) accelerating E) globalizing

D Page Ref: 624 Objective: 1 Difficulty: Easy

23) Companies that produce many products for many markets may adopt a ________ marketing organization. A) flat B) brand C) product D) matrix E) top-down

D Page Ref: 629 Objective: 2 Difficulty: Easy

30) Which of the following benefits is offered by sales promotion tools? A) Sales promotion tools are more authentic and credible to buyers than others such as advertising, public relations, and personal selling. B) Sales promotion tools can reach prospects who prefer to avoid mass media and targeted promotions. C) Sales promotion tools are typically an indirect form of "soft-sell" and hence, better received by customers. D) Sales promotion tools incorporate some concession, inducement, or contribution that gives value to the consumer. E) Sales promotion tools allow buyers personal choices and encourage them to respond directly.

D

34) Which of the following is the main advantage of radio as an advertising medium? A) low competition B) more attention than television C) longer duration of ad exposure D) flexibility E) standardized rate structures

D

36) Which of the following marketing communications tools is most effective at influencing customers at the conviction stage of buyer readiness? A) advertising B) publicity C) sales promotion D) personal selling E) events and experiences

D

37) The number of different persons or households exposed to a particular media schedule at least once during a specified time period is known as ________. A) range B) impact C) intensity D) reach E) frequency

D

57) Which of the following is true regarding cause-related marketing? A) The positive impact of cause-related marketing can be increased through sporadic involvement with numerous causes. B) Many companies focus on multiple causes to simplify execution and maximize impact. C) Limiting support to a single cause increases the pool of stakeholders who can transfer positive feelings from the cause to the firm. D) Most firms choose causes that fit their corporate or brand image and matter to their employees and shareholders. E) In order to avoid public backlash, firms are advised to adopt a hard-sell approach to their cause efforts.

D

139) Name a few products for which personal influence plays a significant role in marketing communications.

Personal influence carries especially great weight when products are expensive, risky, or purchased infrequently, and when products suggest something about the user's status or taste. On that count, life insurance, real estate, health services, automobiles, luxury items, consultation services, etc. can be expected to be significantly influenced by word-of-mouth and endorsements by credible sources.

142) Marco is working on promoting his company's Glazer brand of electronic razors. The company estimates 50 million potential users and sets a target of attracting 8 percent of the market. He hopes to reach 80 percent of the potential customers with an advertising message. He would be pleased if 25 percent of the prospects that were aware, tried the Glazer. According to further estimates, 40 percent of all triers will become loyal users. If the cost of exposing 1 percent of the target population to one impression is $4500 on an average, determine the necessary advertising budget, according to the objective-and-task method.

Using the objective-and-task method, Marco can arrive at the marketing communications budget by following these steps: 1. The company estimates 50 million potential users and sets a target of attracting 8 percent of the market — that is, 4 million users. 2. The percentage of the market that should be reached by advertising is determined. Marco hopes to reach 80 percent (40 million prospects) with his advertising message. 3. The percentage of aware prospects that should be persuaded to try the brand is then calculated. Marco would be pleased if 25 percent of aware prospects (10 million) tried Glazers. He estimates that 40 percent of all triers, or 4 million people, will become loyal users. This is the market goal. 4. Next, the number of advertising impressions per 1 percent trial rate is estimated. Marco estimates that 40 advertising impressions (exposures) for every 1 percent of the population will bring about a 25 percent trial rate. 5. The number of gross rating points that would have to be purchased is now calculated. A gross rating point is one exposure to 1 percent of the target population. Because Marco wants to achieve 40 exposures to 80 percent of the population, he will want to buy 3,200 gross rating points. 6. Finally, the necessary advertising budget on the basis of the average cost of buying a gross rating point is estimated. To expose 1 percent of the target population to one impression costs an average of $4,500. Therefore, 3,200 gross rating points will cost $14,400,000 (= $4,500 × 3,200) in the introductory year.

In response to a short-term oversupply of wine in the marketplace, the makers of Kendall-Jackson developed two new brands by using _______ (designing products on a computer and producing rough models to show potential consumers for their reactions) to quickly bring its ideas to life.

Rapid Prototyping

101) Gabrielle is the chief marketing officer of Boyd Pharmaceuticals. She is meeting with Trent, the chief financial officer to decide on the company's marketing communications budget. After extensive discussions, they decide that the size of the budget will be calculated as a fraction of the overall turnover. What method did Gabrielle and Trent use to arrive at the marketing communications budget? A) affordable method B) objective-and-task method C) competitive-parity method D) activity-based method E) percentage-of-sales method

E

19) A disadvantage of the product-management organization is that ________. A) it marginalizes a company's smaller brands B) it reduces organizational responsiveness to new products in the marketplace C) product managers generally exercise authority outside their areas of responsibility D) it prevents product managers from gaining sufficient expertise in their product areas E) it fails to build long-term strengths as brand managers normally manage brands for only short periods

E

4) Merging involves ________. A) reducing the number of organizational levels to get closer to the customer B) removing barriers that separate organizational departments C) partnering with fewer but better value-adding suppliers D) working more closely with customers to add value to their operations E) acquiring firms in the same or complementary industries

E

59) LCH is a leading electronics company that produces and markets its own brand of desktop and laptop computers, for both individual consumers and businesses. During the planning process, LCH conducts a ________, which profiles all interactions customers in the target market may have with LCH, its computers, and its services. A) sales promotion B) brand association C) brand contact D) response hierarchy model E) communication audit

E

73) In supermarkets and other retail outlets, RFID is used to ________. A) change prices instantaneously B) check for spoilage or damage to goods C) advertise special offers and discounts D) run continual promotional messages E) monitor inventory and track goods

E

78) Which of the following is the marketing communications objective for a new-to-the-world product, such as electric cars? A) enhancing brand awareness B) developing brand attitude C) increasing brand purchase intention D) encouraging repeat purchases E) establishing category need

E

98) Which of the following is a sales promotion tool that is consumer franchise building in nature? A) price-off packs B) contests and sweepstakes C) consumer refund offers D) trade allowances E) frequency awards

E

100) When Fred sends in the box top from his cereal to receive a free DVD of a cartoon featuring one of his favorite characters, he is responding to a ________. A) self-liquidating premium B) with-pack premium C) reduced-price pack D) banded pack E) free in-the-mail premium

E

101) A brand developed by a retailer and/or wholesaler that is available only in selected retail outlets is called a ________ brand. A) generic B) franchisee C) marque D) national E) private-label

E

104) An administered VMS coordinates successive stages of production and distribution through ________. A) an automated central control unit B) single ownership C) the combined efforts of all its members D) the establishment of contractual obligations E) the size and power of one of the members

E

108) Which of the following terms describes the practice of retailers purchasing a greater quantity during a sales promotion period than they can immediately sell? A) diverting B) panic buying C) straight rebuy D) buyout E) forward buying

E

112) ________ time begins with the promotional launch and ends when approximately 95 percent of the deal merchandise is in the hands of consumers. A) Lead B) Hold C) Setup D) Link E) Sell-in

E

113) Rolex calls itself the "Official Timekeeper" of the Wimbledon and Australian Open lawn tennis championships, by virtue of its sponsorships of the marquee events. What is the most likely objective for Rolex's sponsorship deal with these events? A) to permit merchandising or promotional opportunities B) to express commitment to the community or on social issues C) to create experiences and evoke feelings D) to identify with a particular target market or lifestyle E) to increase salience of company or product name

E

12) Marketers often cut the cost of advertising dramatically by using consumers as their creative team. This strategy is known as ________. A) disintermediation B) public relations C) vertical integration D) reintermediation E) crowdsourcing

E

135) When Japanese teenagers carry DOCOMO phones from NTT and use them to order goods, they are engaged in ________. A) B2B e-commerce B) brick-and-click commerce C) infomediation D) dilution E) m-commerce

E

142) Which of the following is an example of a private carrier? A) MET is a transporter that operates only in the Chicago area and charges fixed prices. B) BCL is a family firm that owns only three trucks, but takes small orders for transport. C) VTV owns a fleet of trucks and transports goods for any client for a fee. D) COM is a shipping firm that transports goods by road and rail across the United States. E) BEL Inc. manufactures parts for automobiles and transports its products to customers itself.

E

143) As Ben manages communications for his company's watch brand, which has reached the decline stage in the product life cycle, which of the following marketing communications mix tools is he most likely to continue? A) interactive marketing B) advertising C) personal selling D) direct marketing E) sales promotion

E

22) Which of the following is true about a brand-asset management team (BAMT)? A) A BAMT is part of the triangular and vertical product team structures. B) BAMTs often report directly to the organization's chief branding officer. C) BAMTs are a traditional, tried and tested means of managing brands. D) Companies with a product-management organization often have only one BAMT. E) The BAMT consists of key representatives from functions affecting the brand's performance.

E

40) Which of the following marketing communications tools has the highest cost-effectiveness in the introduction stage of the product life cycle? A) personal selling B) sales promotion C) interactive marketing D) direct marketing E) events and experiences

E

45) What is the weighted number of exposures of a media schedule that reaches 80 percent of the target audience, with an exposure frequency of 4 and impact value of 2? A) 10 B) 160 C) 240 D) 320 E) 640

E

46) Which of the following is an advantage of using newspapers as an advertising medium? A) long lifespan B) high reproduction quality C) huge "pass-along" audience D) high level of targeting E) good local market coverage

E

50) In order to promote ethical cultures, companies should do all of the following EXCEPT ________. A) disseminate a written code of ethics B) build a company tradition of ethical behavior C) hold people responsible for observing ethical and legal guidelines D) ensure every employee knows and observes relevant laws E) encourage business practices that are not clearly ethical or unethical

E

7) ________ refer to the marketing communications element that involves company-sponsored activities and programs designed to create daily or special brand-related interactions with consumers. A) Publicity campaigns B) Trade promotions C) Advertisements D) Public relations E) Events and experiences

E

76) A store selling expensive artwork and luxury goods typically falls into the ________ group with respect to margins and volume. A) mixed markup, high-volume B) low-volume, mixed markup C) low-volume, low-markup D) high-volume, low-markup E) high-markup, low-volume

E

99) ________ software provides a set of Web-based applications that automate and integrate project management, campaign management, budget management, asset management, brand management, customer relationship management, and knowledge management. A) Annual plan control B) Profitability control C) Efficiency control D) Strategic control E) Marketing resource management (MRM)

E

Aldi, Lidl, Dollar​ General, and Family Dollar are examples of​ ________, because they carry a more restricted merchandise​ mix, at even lower​ prices, than discount stores. A. convenience stores B. specialty stores C. superstores D. ​off-price retailers E. ​extreme-value stores

E

24) Telemarketing is a type of ________. A) direct selling B) network marketing C) multilevel selling D) close-range marketing E) direct marketing

E Diff: 1 LO: 18.1: What major types of marketing intermediaries occupy this sector? AACSB: Analytical thinking

5) Which of the following is the best example of a new-to-the-world product? A) Walmart, the retail giant, opens new stores in an underdeveloped African country. B) Pestorica, a publishing company, decides to launch a new sports magazine. C) Tata Motors, an Indian automobile company, acquires Jaguar to extend its business. D) An Asian company licenses a US apparel brand name though the brand is not familiar in Asia. E) Kids-Med, a company that produces childcare products, launches a non-contact thermometer.

E Diff: 2 LO: 15.1: How can new products be categorized? AACSB: Analytical thinking; Application of knowledge

14) It has been observed that most new products have shorter product life cycles. What is the reason for this? A) Most new products do not use technology. B) Most new products are not backed by a marketable idea. C) New products do not get adequate management support. D) Social and governmental constraints lead to this failure. E) Rivals quickly copy products that are successful.

E Diff: 2 LO: 15.2: What challenges does a company face in developing new products and services? AACSB: Reflective thinking

60) A group of college graduates decides to start a business. Though they are knowledgeable in various business domains, they are unable to arrive at a valuable business idea. They decide to search for ideas in a structured manner. They meet and start discussing everyone's ideas. Each idea is recorded and then the thoughts that come up in relation to the ideas are written down and discussed. This process helps them to finalize a business plan. What technique is used here? A) morphological analysis B) forced relationship analysis C) reverse assumption analysis D) attribute listing E) mind mapping

E Diff: 2 LO: 15.4: What are the main stages in developing new products and services? AACSB: Analytical thinking; Application of knowledge

50) Companies such as Edison Nation and the Big Idea Group have sprung up to tap into ________ possibilities, often combining its own design, branding, engineering, and sales teams with online participants, forming a community for devising new products. A) stage-gating B) cocreation C) microstocking D) buzzing E) crowdsourcing

E Diff: 2 LO: 15.4: What are the main stages in developing new products and services? AACSB: Application of knowledge

54) The ________ technique used for stimulating creativity identifies a problem and then considers the dimension, the medium, and the power source. A) attribute listing B) reverse assumption analysis C) mind mapping D) lateral marketing E) morphological analysis

E Diff: 2 LO: 15.4: What are the main stages in developing new products and services? AACSB: Reflective thinking

77) A(n) ________ is an elaborated version of a product idea expressed in consumer terms. A) test brand B) alpha product C) beta product D) business schedule E) product concept

E Diff: 2 LO: 15.5: What is the best way to manage the generation of new ideas? AACSB: Analytical thinking

7) It has been observed that most new products have shorter product life cycles. What is the reason for this? A) Most new products do not use technology. B) Most new products are not backed by a marketable idea. C) New products do not get adequate management support. D) Social and governmental constraints lead to this failure. E) Rivals quickly copy products that are successful.

E Page Ref: 572 Objective: 1 Difficulty: Easy

28) The ________ technique used for stimulating creativity identifies a problem and then considers the dimension, the medium, and the power source. A) attribute listing B) reverse assumption analysis C) mind mapping D) lateral marketing E) morphological analysis

E Page Ref: 579 Objective: 3 Difficulty: Easy

39) ________ means presenting the product concept to target consumers, symbolically of physically, and getting their reactions. A) Perceptual mapping B) Brand-positioning mapping C) Brand attribute mapping D) Concept development E) Concept testing

E Page Ref: 582 Objective: 4 Difficulty: Easy

47) ________ exhibit replacement cycles dictated by physical wear or obsolescence associated with changing style, features, and performance. A) Frequently purchased products B) High-moving goods C) Inexpensive products D) Commodity products E) Infrequently purchased products

E Page Ref: 585 Objective: 4 Difficulty: Easy

49) For a frequently purchased new product, the seller estimates repeat sales as well as first-time sales. A high rate of repeat purchasing means customers ________. A) value price more than differentiation B) do not support innovation on brands C) value differentiation more than price D) prefer personalized products rather than standard ones E) are satisfied with the product

E Page Ref: 585 Objective: 4 Difficulty: Easy

56) A large FMCG company decides to test market Kora, a new brand of face cleanser, to be launched soon. The company initially distributes a few free samples to some prospective consumers. Later it offers the product to the customers at a discounted price and observes that not only more than seventy percent of the customers are purchasing it but the same number are also satisfied using it. The company keeps using this process 3-4 times to obtain a correct count of the number of people purchasing the product repeatedly. Which of the following testing methods is being used here? A) simulated testing B) controlled testing C) full test marketing D) parallel testing E) sales-wave research

E Page Ref: 588 Objective: 4 AACSB: Analytic skills Difficulty: Moderate

Which of the following is an example of a private carrier? A) MET is a transporter that operates only in the Chicago area and charges fixed prices. B) BCL is a family firm that owns only three trucks, but takes small orders for transport. C) VTV owns a fleet of trucks and transports goods for any client for a fee. D) COM is a shipping firm that transports goods by road and rail across the United States. E) BEL Inc. manufactures parts for automobiles and transports its products to customers itself.

E) BEL Inc. manufactures parts for automobiles and transports its products to customers itself.

LCH is a leading electronics company that produces and markets its own brand of desktop and laptop computers, for both individual consumers and businesses. During the planning process, LCH conducts a ________, which profiles all interactions customers in the target market may have with LCH, its computers, and its services. A) sales promotion B) brand association C) brand contact D) response hierarchy model E) communication audit

E) Communication Audit

Which of the following elements of the marketing communications mix involves face-to-face interaction with one or more prospective purchasers for the purpose of making presentations, answering questions, and procuring orders? A) advertising B) sales promotion C) word-of-mouth marketing D) public relations E) personal selling

E) Personal Selling

Aisha is a marketing manager with Injoos, a company that manufactures packaged fruit juices. Knowing that several other companies exist in the market that offer similar products, Aisha decides to build a customer base from among those who prefer to avoid mass media and other targeted promotions. Which of the following marketing communications tools would be her best option to build a favorable impression among the prospective customers? A) advertising B) personal selling C) sales promotions D) direct marketing E) public relations

E) Public Relations

As Ben manages communications for his company's watch brand, which has reached the decline stage in the product life cycle, which of the following marketing communications mix tools is he most likely to continue? A) interactive marketing B) advertising C) personal selling D) direct marketing E) sales promotion

E) Sales Promotion

hich of the following is true for retailing? A) Manufacturers are not considered to be retailers as they are engaged in producing the product. B) Vending machines are considered to be retailing only if they are located within stores. C) Retailing deals only with goods; it does not include services. D) Selling from a consumer's home is direct selling, but not retailing. E) Wholesalers are only considered to be retailers if they are selling to final consumers.

E) Wholesalers are only considered to be retailers if they are selling to final consumers.

Electronic shelf labeling allows retailers to ________. A) check inventory levels instantaneously B) order electronically from suppliers C) run continual promotional messages D) advertise sales and special offers E) change price levels instantaneously

E) change price levels instantaneously

147) How can a firm periodically reassess its strategic approach to the marketplace?

Each company should periodically reassess its strategic approach to the marketplace with a good marketing audit. Companies can also perform marketing excellence reviews and ethical/social responsibility reviews.

140) How can a firm periodically reassess its strategic approach to the marketplace?

Each company should periodically reassess its strategic approach to the marketplace with a good marketing audit. Companies can also perform marketing excellence reviews and ethical/social responsibility reviews. Page Ref: 645 Objective: 5 Difficulty: Easy

The step in the consumer-adoption process where a consumer considers whether to try the innovation is _______.

Evaluation

Dollar stores base their strategy around extreme ________, cutting the costs associated with constant sales and promotions and increasing consumer confidence in everyday shelf prices.

Everyday Low Pricing

157) Identify when exclusive distribution deals are legal and when they break the law.

Exclusive arrangements are legal as long as they do not substantially lessen competition or tend to create a monopoly and as long as both parties enter into them voluntarily. Exclusive dealing often includes exclusive territorial agreements. The producer may agree not to sell to other dealers in a given area, or the buyer may agree to sell only in its own territory. The first practice increases dealer enthusiasm and commitment. It is also perfectly legal — a seller has no legal obligation to sell through more outlets than it wishes. The second practice, whereby the producer tries to keep a dealer from selling outside its territory, has become a major legal issue. Producers of a strong brand sometimes sell it to dealers only if they will take some or all of the rest of the line. This practice is called full-line forcing. Such tying agreements are not necessarily illegal, but they do violate US law if they tend to lessen competition substantially.

40) Many companies now focus on departments as opposed to processes, because processes can be a barrier to smooth performance.

FALSE

41) A superstore is a storeless retailer serving a specific clientele who are entitled to buy from a list of retailers that have agreed to give discounts in return for membership.

FALSE

41) Transforming into a true market-driven company involves organizing around products.

FALSE

68) Electronic shelf labeling allows retailers to check inventory levels instantaneously.

FALSE

78) Social marketing is a new trend in marketing.

FALSE

88) The gross margin on a product bears a direct relation to the direct product profit.

FALSE

89) When a manufacturer requests a behavior that is warranted under the contract, the manufacturer is using coercive power.

FALSE

90) A manufacturer obtains referent power when it has special knowledge that intermediaries value.

FALSE

83) Today, consumers are price takers and accept prices at face value or as given.

FALSE Page Ref: 386 Objective: 1 Difficulty: Easy

89) Companies strive to maximize their current profits if they are plagued with overcapacity, intense competition, or changing consumer wants.

FALSE Page Ref: 389 Objective: 2 Difficulty: Easy

90) In reality, it is very easy for firms to estimate their demand and cost functions.

FALSE Page Ref: 389 Objective: 2 Difficulty: Easy

91) If firms wish to maximize their market share, they should opt for market-skimming pricing.

FALSE Page Ref: 389 Objective: 2 Difficulty: Easy

105) A quantity discount is a price reduction given to those who pay their bills promptly.

FALSE Page Ref: 404 Objective: 3 Difficulty: Easy

106) Trade-in allowances reward dealers for participating in advertising and sales support programs.

FALSE Page Ref: 404 Objective: 3 Difficulty: Easy

110) When firms charge different prices to different customer groups for the same product or service, it is a case of second-degree price discrimination.

FALSE Page Ref: 406 Objective: 3 Difficulty: Easy

90) The demand landscape uses conceptual lens and structured innovative-thinking tools to achieve market perspectives from different angles.

FALSE Page Ref: 576 Objective: 3 Difficulty: Easy

92) Crowdsourcing means inviting the Internet community to help create content or software, often with prize money or a moment of glory as an incentive.

FALSE Page Ref: 577 Objective: 3 Difficulty: Easy

99) A category concept defines the product's attributes and features.

FALSE Page Ref: 581 Objective: 4 Difficulty: Easy

105) The trade-off approach may be easier to use when there are only a few variables and alternatives.

FALSE Page Ref: 584 Objective: 4 Difficulty: Easy

108) Survival-age distribution refers to the number of customers that the product has in year one, two, three, and so on.

FALSE Page Ref: 585 Objective: 4 Difficulty: Easy

101) Social marketing programs take little time to develop and are generally easy to implement.

FALSE Page Ref: 642 Objective: 3 AACSB: Ethical understanding and reasoning abilities Difficulty: Easy

107) Repurchase rate is a sales metric that is used to evaluate the performance of marketing plans.

FALSE Page Ref: 644 Objective: 5 Difficulty: Easy

110) Self-audits tend to provide more objective information than audits conducted by external consultants.

FALSE Page Ref: 645 Objective: 5 Difficulty: Easy

111) A marketing audit only benefits a company that is in trouble; companies in good health do not need to conduct them.

FALSE Page Ref: 645 Objective: 5 Difficulty: Easy

114) In the context of overall market share, customer loyalty refers to the percentage of all customers who buy from the company.

FALSE Page Ref: 655 Objective: APP Difficulty: Easy

115) The first step in conducting a marketing profitability analysis involves assigning functional expenses to marketing entities.

FALSE Page Ref: 657 Objective: APP Difficulty: Easy

92) In competitive markets with low entry barriers, the optimal channel structure will inevitably change over time.

TRUE

108) The lower price of generics is made possible by lower-cost labeling and packaging and minimal advertising, and sometimes lower-quality ingredients.

TRUE

108) Though the target audience can be profiled in terms of demographic, psychographic, or behavioral segments, it is often useful to do so in terms of usage and loyalty.

TRUE

116) In a one-sided message, presenting the strongest argument first arouses attention and interest, important in media where the audience often does not attend to the whole message.

TRUE

116) Sales promotions in markets of high brand similarity can produce a high sales response in the short run but little permanent gain in brand preference over the longer term.

TRUE

117) Incessant price reductions, coupons, deals, and premiums can devalue a product in buyers' minds.

TRUE

123) Manufacturers handle forward buying and diverting by producing and delivering less than the full order in an effort to smooth production.

TRUE

124) Additional costs beyond the cost of specific promotions include the risk that promotions might decrease long-run brand loyalty.

TRUE

125) Personal communications channels derive their effectiveness from individualized presentation and feedback and include direct and interactive marketing, word-of-mouth marketing, and personal selling.

TRUE

127) Although personal communication is often more effective than mass communication, mass media might be the major means of stimulating personal communication.

TRUE

129) Desktop marketing gives marketers information and decision structures on computer dashboards.

TRUE

129) Two-step communication suggests that mass communicators should direct messages specifically to opinion leaders and let them carry the message to others.

TRUE

132) Price pressure will increase for undifferentiated products than differentiated products.

TRUE

134) The ratio of promoters to detractors is a customer metric that is used to evaluate the performance of marketing plans.

TRUE

135) A marketing audit is an orderly examination of the organization's macro- and micromarketing environments, marketing objectives and strategies, marketing systems, and specific activities.

TRUE

135) An ideal event is unique but not encumbered with many sponsors, lends itself to ancillary marketing activities, and reflects or enhances the sponsor's brand or corporate image.

TRUE

136) Consumers are more engaged and attentive with their smart phones than when they are online.

TRUE

137) Although supply-side exposure methods provide quantifiable measures, equating media coverage with advertising exposure ignores the content of the respective communications.

TRUE

138) To succeed in the future, marketing must be more holistic and less departmental.

TRUE

139) In the future, there will be greater emphasis on precision marketing as opposed to mass marketing.

TRUE

139) The supply-side measurement method focuses on potential exposure to a brand by assessing the extent of media coverage, and the demand-side method focuses on exposure reported by consumers.

TRUE

142) Operating management is most effective in controlling direct costs and traceable common costs.

TRUE

147) Inventory cost increases at an accelerating rate as the customer-service level approaches 100 percent.

TRUE

149) By using full-service marketing communications agencies, integrated and more effective marketing communications at a much lower total communications cost can be achieved.

TRUE

149) Order-processing costs must be compared with inventory-carrying costs because the larger the average stock carried, the higher the inventory-carrying costs.

TRUE

152) Creative public relations can affect public awareness at a fraction of the cost of advertising.

TRUE

153) MPR can hold down promotion cost because it costs less than direct-mail and media advertising.

TRUE

155) Dual compensation pays existing channels for sales made through new channels.

TRUE

156) In the future, marketers should use the Internet more, improve the sales automation system, apply market automation to routine decisions, and develop formal marketing decision models and marketing dashboards.

TRUE

16) When Staples introduced a new $69.99 paper-shredding device called the MailMate in 2006 by striking a two-episode deal with NBC's popular television program, The Office, it was using product placement.

TRUE

21) Marketing communications allow companies to link their brands to other people, places, events, brands, experiences, feelings, and things.

TRUE

24) Marketing communications in almost every medium and form have been on the rise, and some consumers feel they are increasingly invasive.

TRUE

27) In building brand equity, marketers should be "media neutral" and evaluate all communication options on effectiveness and efficiency.

TRUE

29) Internal marketing requires that everyone in the organization buy into the concepts and goals of marketing and engage in choosing, providing, and communicating customer value.

TRUE

36) Retailing includes all the activities involved in selling goods or services directly to final consumers.

TRUE

38) Market managers are staff people, with duties like those of a product manager.

TRUE

39) The network marketing sales system works by recruiting independent businesspeople who act as distributors.

TRUE

40) Electronic shopping is a form of direct marketing.

TRUE

45) Advertising allows the buyer to receive and compare the messages of various competitors.

TRUE

45) The franchisor owns a trade or service mark and licenses it to franchisees in return for royalty payments.

TRUE

47) In general, personal selling is used more with complex, expensive, and risky goods and in markets with fewer and larger sellers.

TRUE

61) Products such as heating and cooling systems are usually sold and maintained by the company or by franchised dealers.

TRUE

66) An intensive distribution strategy serves well for products such as snack foods and soft drinks.

TRUE

67) As discount retailers improve their quality and image, consumers have been willing to trade down.

TRUE

67) Firms that are viewed as being socially responsible have the added benefit of being able to attract employees.

TRUE

68) A "do-feel-learn" response sequence is considered appropriate when the audience has high involvement but perceives little or no differentiation within the product category.

TRUE

74) Corporate philanthropy as a whole is on the rise.

TRUE

75) Established brands usually are supported with lower advertising budgets, measured as a ratio to sales.

TRUE

81) The actual success of a social marketing program should be evaluated in terms of the program objectives.

TRUE

82) The rationale behind place advertising is that marketers are better off reaching people where they work, play, and shop.

TRUE

85) The forgetting rate is the rate at which the buyer forgets the brand; the higher the forgetting rate, the more continuous the advertising should be.

TRUE

86) Featuring exclusive national brands that are not available at competing retailers is a way of generating consumer interest in a retailer.

TRUE

93) Early buyers might be willing to pay for high-value-added channels, but later buyers will switch to lower-cost channels.

TRUE

94) Postpurchase services include shipping and delivery, gift wrapping, adjustments, and returns.

TRUE

85) Customers usually have a lower price threshold below which prices signal inferior or unacceptable quality, as well as an upper price threshold above which prices are prohibitive and the product appears not worth the money.

TRUE Page Ref: 386 Objective: 1 Difficulty: Easy

87) When examining products, consumers compare an observed price to an internal reference price they remember or an external frame of reference.

TRUE Page Ref: 387 Objective: 1 Difficulty: Easy

88) Many consumers use price as an indicator of quality and value.

TRUE Page Ref: 388 Objective: 1 Difficulty: Easy

95) Price elasticity depends upon the magnitude and direction of the contemplated price change.

TRUE Page Ref: 392 Objective: 2 Difficulty: Easy

97) Total costs consist of the sum of the fixed and variable costs for any given level of production.

TRUE Page Ref: 393 Objective: 2 Difficulty: Easy

100) Value pricing requires a company to reengineer its operations to become a low-cost producer.

TRUE Page Ref: 400 Objective: 2 Difficulty: Easy

104) Offset is a form of countertrade where sellers receive full payment in cash and agree to spend a substantial amount of the money in the country where they are trading within a stated time period.

TRUE Page Ref: 404 Objective: 3 Difficulty: Easy

108) Loss leader pricing dilutes a company's brand image.

TRUE Page Ref: 405 Objective: 3 Difficulty: Easy

111) The airline industries implement yield pricing by offering discounted but limited early purchases, higher-priced late purchases, and the lowest rates on unsold inventory just before it expires.

TRUE Page Ref: 406 Objective: 3 Difficulty: Easy

115) Cost inflation provokes price increases.

TRUE Page Ref: 408 Objective: 4 Difficulty: Easy

117) Escalator clauses are found in contracts for major industrial projects, such as aircraft construction and bridge building.

TRUE Page Ref: 408 Objective: 4 Difficulty: Easy

118) Generally, consumers prefer small price increases on a regular basis to sudden, sharp increases.

TRUE Page Ref: 409 Objective: 4 Difficulty: Easy

119) Shrinking the amount of product instead of raising the price is a good way to counteract consumer resistances to price increases.

TRUE Page Ref: 409 Objective: 4 Difficulty: Easy

120) A company must consider the product's stage in the life cycle and its importance in the company's portfolio before responding to a competitor's price cut.

TRUE Page Ref: 409 Objective: 5 Difficulty: Easy

79) Companies typically must create a strong R&D and marketing partnership to pull off a radical innovation.

TRUE Page Ref: 570 Objective: 1 Difficulty: Easy

81) Companies that fail to develop new products leave their existing offerings vulnerable to increased domestic and foreign competition.

TRUE Page Ref: 571 Objective: 1 Difficulty: Easy

87) Venture teams are cross-functional groups charged with developing a specific product or business.

TRUE Page Ref: 574 Objective: 2 Difficulty: Easy

94) Employees can be a source of ideas for improving production, products, and services.

TRUE Page Ref: 578 Objective: 3 Difficulty: Easy

96) Reverse assumption analysis lists all the normal assumptions about an entity and then turns them around.

TRUE Page Ref: 579 Objective: 3 Difficulty: Easy

101) Rapid prototyping refers to designing products on a computer and then producing rough models to show potential consumers for their reactions.

TRUE Page Ref: 582 Objective: 4 Difficulty: Easy

103) Consumer preferences for alternative product concepts can be measured with conjoint analysis.

TRUE Page Ref: 583 Objective: 4 Difficulty: Easy

107) A business analysis is typically performed after the management has developed the product concept and marketing strategy.

TRUE Page Ref: 585 Objective: 4 Difficulty: Easy

109) Dragalong income refers to additional income whereas cannibalized income refers to reduced income.

TRUE Page Ref: 586 Objective: 4 Difficulty: Easy

112) Sales-wave research can be implemented quickly and carried out without final packaging and advertising.

TRUE Page Ref: 588 Objective: 4 Difficulty: Easy

113) Expensive industrial goods and new technologies will normally undergo alpha and beta testing.

TRUE Page Ref: 589 Objective: 4 Difficulty: Easy

78) Companies producing a variety of products and brands often establish a product- (or brand-) management organization.

TRUE Page Ref: 627 Objective: 2 Difficulty: Easy

109) A marketing audit is an orderly examination of the organization's macro- and micromarketing environments, marketing objectives and strategies, marketing systems, and specific activities.

TRUE Page Ref: 645 Objective: 5 Difficulty: Easy

113) In the future, there will be greater emphasis on precision marketing as opposed to mass marketing.

TRUE Page Ref: 648 Objective: 4 Difficulty: Easy

28) Briefly describe the current marketing communications environment.

Technology and other factors have profoundly changed the way consumers process communications, and even whether they choose to process them at all. The rapid diffusion of multipurpose smart phones, broadband and wireless Internet connections, and ad-skipping digital video recorders (DVRs) have eroded the effectiveness of the mass media. In 1960, a company could reach 80 percent of US women with one 30-second commercial aired simultaneously on three TV networks: ABC, CBS, and NBC. Today, the same ad would have to run on 100 channels or more to achieve this marketing feat. Consumers not only have more choices of media, they can also decide whether and how they want to receive commercial content.

44) What is a market-management organization?

Many companies sell their products to differing markets. When customers fall into different user groups with distinct buying preferences and practices, a market-management organization is desirable. Market managers supervise several market-development managers, market specialists, or industry specialists and draw on functional services as needed. Market managers of important markets might even have functional specialists reporting to them. Market managers are staff (not line) people, with duties like those of product managers. They develop long-range and annual plans for their markets and are judged by their market's growth and profitability. Because this system organizes marketing activity to meet the needs of distinct customer groups, it shares many advantages and disadvantages of product-management systems.

54) Karl Lipton is the marketing communications coordinator for a major electronics manufacturer. He is assigned with charting out a communications strategy for a new range of mobile phones developed by his company. How will Karl's communications strategy look over the course of the mobile phones' life cycle?

Mobile phones have relatively short life cycles. In the introduction stage of the phones' life cycle, advertising, events and experiences, and publicity have the highest cost-effectiveness. In-store personal selling helps improve distribution coverage, while sales promotions such as discounted accessories, freebies etc., help induce trial. In the growth stage, demand has its own momentum through word of mouth and interactive marketing. Web sites are effective tools during the growth stage. As the phones reach the maturity stage, advertising, promotional events and experiences, and personal selling again become more important. As the phones enter the decline stage of their life cycle, sales promotion takes over as the most important tool in the marketing communications mix, even as other communication tools are reduced, and salespeople give the phones only minimal attention.

129) ShoeZone is a shoe retailer with outlets across the country. The company is trying to reduce its inventory and warehousing costs, but needs to keep delivery speeds as short as possible. What can ShoeZone do to achieve this?

More stocking locations mean goods can be delivered to customers more quickly, but warehousing and inventory costs are higher. To reduce these costs, ShoeZone might centralize its inventory in one place and use fast transportation to fill orders.

24) Describe how advertising objectives are set to reflect the product class.

The advertising objective should emerge from a thorough analysis of the current marketing situation. If the product class is mature, the company is the market leader, and brand usage is low, the objective is to stimulate more usage. If the product class is new, the company is not the market leader, but the brand is superior to the leader, then the objective is to convince the market of the brand's superiority.

23) Explain the classification of advertising objectives.

The classification of advertising objectives includes: • Informative advertising aims to create brand awareness and knowledge of new products or new features of existing products. • Persuasive advertising aims to create liking, preference, conviction, and purchase of a product or service. Some persuasive advertising uses comparative advertising, which makes an explicit comparison of the attributes of two or more brands. Comparative advertising works best when it elicits cognitive and affective motivations simultaneously, and when consumers are processing advertising in a detailed, analytical mode. • Reminder advertising aims to stimulate repeat purchase of products and services. • Reinforcement advertising aims to convince current purchasers that they made the right choice.

157) List some of the marketing trends that are likely to emerge in the near future.

The coming years will see: • The demise of the marketing department and the rise of holistic marketing • The demise of free-spending marketing and the rise of ROI marketing • The demise of marketing intuition and the rise of marketing science • The demise of manual marketing and the rise of both automated and creative marketing • The demise of mass marketing and the rise of precision marketing

20) Garolds Stores operates as a low price retailer. It offers home fashion products, such as wall decor, frames, candles, bath and bedding products, furniture, home accents, and kitchen products. The firm has showrooms and retail stores across United States. Customers can buy products in three different ways: (1) they can place orders online and have the products shipped to their homes; (2) they can buy directly from the showrooms and retail outlets; or (3) they can place orders online and pick them up from the nearest showroom. What are the benefits that Garolds Stores could achieve by using various channels to sell to customers?

The company can increase its sales by targeting different segments of customers through various channels. Multichannel marketing allows the company to deliver the right products in the right places in the right way at the least cost.

141) Caribou Coffee Company, Inc. operates coffeehouses primarily in the United States. The company wants to launch a new brand of specialty coffee and it has created a few hypothetical combinations. The company wants to select two of the combinations after obtaining a ranking by customers based on their preference. What concept testing method is suited here?

The company can use conjoint analysis, a method for deriving the utility values that consumers attach to varying levels of a product's attributes. Page Ref: 583 Objective: 4 AACSB: Analytic skills Difficulty: Moderate

74) National Beverage Corp. produces and distributes a wide range of beverages. It offers a selection of flavored soft drinks, juices, sparkling waters, energy drinks, nutritionally enhanced waters, and other specialty beverages. What distribution strategy will be most suitable for the company's products?

The company has to use intensive distribution, which places the goods or services in as many outlets as possible. This strategy serves well for products consumers buy frequently or in a variety of locations.

119) Altrudex Inc. is involved in the manufacture, distribution, and sale of consumer electronics. The company sources over half the products it sells from companies it partly or wholly owns. It also owns a large retail chain and sells its products through them. What marketing system is Altrudex using?

The company is using a corporate VMS. A corporate VMS combines successive stages of production and distribution under single ownership.

19) The Norticon Group provides and manages computers and network systems for businesses and communities. Norticon sells its products through various means. It uses its sales force to sell to large customers and telemarketing to sell to smaller customers. The company also sells its products via the Internet. Briefly explain the marketing approach being used by the company.

The company is using hybrid channels or multichannel marketing which involves the use of two or more marketing channels to reach customer segments. In multichannel marketing, each channel targets a different segment of buyers, or different need states for one buyer, and delivers the right products in the right places in the right way at the least cost.

126) What is the consumer-adoption process? What are the various steps in the process?

The consumer-adoption process is the mental steps through which an individual passes from first hearing about an innovation to final adoption. The steps are the following: (1) Awareness: The consumer becomes aware of the innovation but lacks information about it. (2) Interest: The consumer is stimulated to seek information about the innovation. (3) Evaluation: The consumer considers whether to try the innovation. (4)Trial: The consumer tries the innovation to improve his or her estimate of its value. (5)Adoption: The consumer decides to make full and regular use of the innovation. Page Ref: 591 Objective: 5 Difficulty: Easy

132) NV Inc. has launched a touch sensitive handset in the Indian market and priced the same at INR 9500. Although many people are checking it out and showing interest about purchasing it, majority of them are holding themselves back because they feel that it is not worth INR 9500. They compare the handsets' feature with that of its other competitors offering the same features and come to a conclusion that it is worth INR 8500 and nothing more than that. What kind of a reference price are the consumers using?

The consumers are using the upper-bound price. Upper-bound price refers to the reservation price or the maximum that most consumers would pay. Page Ref: 387 Objective: 1 AACSB: Analytic skills Difficulty: Moderate

98) A group of entrepreneurs is planning to start a food processing company. Comment on the likely evolution of the company's marketing channels.

The firm is likely to start as a local operation selling in a fairly circumscribed market, using a few existing intermediaries. At this stage, it is often difficult to convince the available intermediaries to handle the firm's line. If the firm is successful, it might branch into new markets with different channels. In smaller markets, the firm might sell directly to retailers; in larger markets, through distributors.

22) In developing an advertising program, marketing managers can make the five major decisions known as the five Ms. List and explain each of these Ms.

The five major decisions are: 1. Mission — sales goals and advertising objectives 2. Money — factors to consider: stage in PLC, market share and consumer base, competition and clutter, advertising frequency, and product substitutability 3. Message — message generation, message evaluation and selection, message execution, and social-responsibility review 4. Media — reach, frequency, impact, major media types, specific media vehicles, media timing, and geographical media allocation 5. Measurement — communication impact and sales impact.

25) Give a brief description of the various factors that affect advertising budget decisions.

The various factors that affect advertising budget decisions include: • Stage in the product life cycle — New products typically merit large advertising budgets to build awareness and to gain consumer trial. Established brands usually are supported with lower advertising budgets, measured as a ratio to sales. • Market share and consumer base — High-market-share brands usually require less advertising expenditure as a percentage of sales to maintain share. To build share by increasing market size requires larger expenditures. • Competition and clutter — In a market with a large number of competitors and high advertising spending, a brand must advertise more heavily to be heard. Even simple clutter from advertisements not directly competitive to the brand creates a need for heavier advertising. • Advertising frequency — The number of repetitions needed to put the brand's message across to consumers has an obvious impact on the advertising budget. • Product substitutability — Brands in less-differentiated or commodity-like product classes (beer, soft drinks, banks, and airlines) require heavy advertising to establish a unique image.

109) Which of the following retailer practices involves buying more units than needed of a product under a sales promotion in a region where the manufacturer offers a promotion deal and shipping the surplus to their stores in non-deal regions? A) diverting B) panic buying C) hoarding D) stockpiling E) forward buying

A

23) Identify the channel function that constitutes both backward and forward flow. A) obtaining funds for financing B) storage of physical goods C) persuasive communication D) movement of physical goods E) overseeing actual transfer of ownership

A

50) Exclusive dealing arrangements are mainly used by companies looking for an edge in markets increasingly driven by ________. A) price B) efficiency C) product variety D) add-on services E) spatial convenience

A

32) Cause marketing has a particularly interested audience in civic-minded ________. A) Millennial consumers B) baby boomers C) Gen Xers D) Gen Yers E) shadow boomers

A Page Ref: 637 Objective: 3 AACSB: Ethical understanding and reasoning abilities Difficulty: Easy

A) advertising allowance B) free samples C) contests for sales reps D) display allowance E) discount coupons

C

136) Suicide is one of the leading causes of death worldwide. Develop the possible objectives of a social marketing campaign which aims to change people's cognitions, values, behaviors, and actions related to suicide.

Cognitive—To promote awareness about the risk factors associated with suicide and how it can be prevented. Action—To motivate people to volunteer for a crisis hotline service aimed at providing support to people with suicidal tendencies. Behavioral—To reduce the incidence of suicide by discouraging substance abuse. Value - To eliminate the stigma associated with suicidal people. Page Ref: 640 Objective: 3 AACSB: Analytic skills Difficulty: Moderate

137) Compare and contrast the use of negative and positive appeals in advertising messages.

Communicators use negative appeals such as fear, guilt, and shame to get people to do things (brush their teeth, have an annual health checkup) or stop doing things (smoking, abusing alcohol, overeating). Fear appeals work best when they are not too strong, when source credibility is high, and when the communication promises, in a believable and efficient way, to relieve the fear it arouses. Messages are most persuasive when moderately discrepant with audience beliefs. Stating only what the audience already believes at best just reinforces beliefs, and if the messages are too discrepant, audiences will counterargue and disbelieve them. Communicators also use positive emotional appeals such as humor, love, pride, and joy. Motivational or "borrowed interest" devices — such as the presence of cute babies, frisky puppies, popular music, or provocative sex appeals — are often employed to attract attention and raise involvement with an ad. These techniques are thought necessary in the tough new media environment characterized by low-involvement consumer processing and competing ad and programming clutter. Attention-getting tactics are often too effective. They may also detract from comprehension, wear out their welcome fast, and overshadow the product.

50) What are the major characteristics of sales promotions as an element of the marketing communications mix?

Companies use sales promotion tools — coupons, contests, premiums, and the like — to draw a stronger and quicker buyer response, including short-run effects such as highlighting product offers and boosting sagging sales. Sales promotion tools offer three distinctive benefits: 1. Ability to be attention-getting — They draw attention and may lead the consumer to the product. 2. Incentive — They incorporate some concession, inducement, or contribution that gives value to the consumer. 3. Invitation — They include a distinct invitation to engage in the transaction now.

125) What are the different forms of countertrade?

Countertrade can take the following forms: • Barter - The buyer and seller directly exchange goods, with no money and no third party involved. • Compensation deal - The seller receives some percentage of the payment in cash and the rest in products. • Buyback arrangement - The seller sells a plant, equipment, or technology to another country and agrees to accept as partial payment products manufactured with the supplied equipment. • Offset - The seller receives full payment in cash but agrees to spend a substantial amount of the money in that country within a stated time period. Page Ref: 404 Objective: 3 Difficulty: Moderate

6) A large staff, along with a higher proportion of specialty goods and slower-moving items and many services, are usually features of ________ retailing. A) self-service B) self-selection C) limited service D) full-service E) limited-selection

D

111) ________ time is the time necessary to prepare a promotional program prior to launching it. A) Sell-in B) Link C) Setup D) Lead E) Hold

D

127) JSE Securities Exchange is the largest stock exchange in Africa. The JSE provides a market where securities can be traded freely under a regulated procedure. The company acts as an intermediary between the traders. JSE is an example of a(n) ________. A) internal broker B) infomediary C) customer community D) market maker E) third party arbitrator

D

132) ________ costs for a manufacturer consist of setup costs and running costs. A) Inventory-carrying B) Containerization C) Wholesaling D) Order-processing E) Transportation

D

138) E&OE is trying to minimize its inventory costs, which are extremely high. The company has realized that it can achieve this by maintaining a near-zero inventory and producing more products only once it is ordered. Which of the following will be true for E&OE? A) Short production runs will be more expensive than longer ones. B) Setup and order-processing costs will be high. C) The order point will be high. D) Order-processing costs will be lower than the inventory-carrying costs. E) E&OE can reduce the average cost per unit by producing a long run.

D

30) Which of the following is true for franchisors? A) The franchisor has to pay the franchisee to be part of the franchise system. B) The franchisor licenses the trademark from the franchisee. C) The franchisor must change its operations to suit those of the franchisee. D) The franchisor collects royalty payments from the franchisee. E) The franchisor pays startup costs for the franchisee.

D

144) What are direct, traceable common, and nontraceable common costs? Give an example of each.

Direct costs are costs that we can assign directly to the proper marketing entities, such as materials costs and sales-force salaries. Traceable common costs are incurred indirectly but can be attributed on a plausible basis to various marketing entities, such as rent expenses. Nontraceable common costs are costs whose assignments are highly arbitrary, such as "corporate image" expenditures. Page Ref: 659 Objective: APP Difficulty: Moderate

120) Explain how stage-gate systems are used by companies to manage innovation and new product launches.

Many top companies use the stage-gate system to divide the innovation process into stages, with a gate or checkpoint at the end of each. The project leader, working with a cross-functional team, must bring a set of known deliverables to each gate before the project can pass to the next stage. To move from the business plan stage into product development requires a convincing market research study of consumer needs and interest, a competitive analysis, and a technical appraisal. Senior managers review the criteria at each gate to make one of four decisions: go, kill, hold, or recycle. Stage-gate systems make the innovation process visible to all and clarify the project leaders and team's responsibilities at each stage. The gates or controls should not be so rigid, however, that they inhibit learning and the development of novel products. Many firms have parallel sets of projects working through the process, each at a different stage. Some firms use a spiral development process that recognizes the value of returning to an earlier stage to make improvements before moving forward. Page Ref: 574-575 Objective: 2 Difficulty: Easy

75) Provide a general description of the four classic response hierarchy models.

Micromodels of marketing communications concentrate on consumers' specific responses to communications. All the response hierarchy models assume the buyer passes through cognitive, affective, and behavioral stages, in that order. This "learn-feel-do" sequence is appropriate when the audience has high involvement with a product category perceived to have high differentiation, such as an automobile or house. An alternative sequence, "do-feel-learn," is relevant when the audience has high involvement but perceives little or no differentiation within the product category, such as an airline ticket or personal computer. A third sequence, "learn-do-feel," is relevant when the audience has low involvement and perceives little differentiation, such as with salt or batteries. By choosing the right sequence, the marketer can do a better job of planning communications.

The Internet enables sellers to discriminate between buyers, and buyers to discriminate between sellers. One of the ways a seller can use the Internet to discriminate between buyers is by ______.

Monitoring customer behavior to tailor offers to individuals

117) What are the types of new products that a firm can introduce? What are the problems associated with introducing a truly innovative product? What are the necessary conditions to create a radically innovative product?

New products range from new-to-the-world products that create an entirely new market to minor improvements or revisions of existing products. Most new-product activity is devoted to improving existing products. Truly innovative and new to the world products incur the greatest cost and risk. Although radical innovations can hurt the company's bottom line in the short run, if they succeed they can create a greater sustainable competitive advantage than ordinary products and produce significant financial rewards as a result. Companies typically must create a strong R&D and marketing partnership to pull off a radical innovation. The right corporate culture is another crucial determinant; the firm must prepare to cannibalize existing products, tolerate risk, and maintain a future market orientation. Few reliable techniques exist for estimating demand for radical innovations. Page Ref: 570 Objective: 1 Difficulty: Easy

130) Explain the concept of overdemand.

One of the factors that prompts a firm to increase its prices is overdemand. When a company cannot supply all its customers, it can raise its prices, ration supplies, or both. It can increase its price in the following ways: • Delayed quotation pricing - The company does not set a final price until the product is finished or delivered. This pricing is prevalent in industries with long production lead times, such as industrial construction and heavy equipment. • Escalator clauses - The company requires the customer to pay today's price and all or part of any inflation increase that takes place before delivery. Escalator clauses are found in contracts for major industrial projects, such as aircraft construction and bridge building. • Unbundling - The company maintains its price but removes or prices separately one or more elements that were part of the former offer, such as free delivery or installation. Car companies sometimes add higher-end audio entertainment systems or GPS navigation systems as extras to their vehicles. • Reduction of discounts - The company instructs its sales force not to offer its normal cash and quantity discounts. Page Ref: 408 Objective: 4 Difficulty: Moderate

Tata Steel adopted a ________ process, which divided the innovation process into stages with checkpoint at the end of each stage, to generate 50 to 100 ideas for everyone that makes it to implementation.

Stage-Gate

26) Briefly describe the role of television as an advertising medium.

Television is generally acknowledged as the most powerful advertising medium and reaches a broad spectrum of consumers at low cost per exposure. TV advertising has two particularly important strengths. First, it can vividly demonstrate product attributes and persuasively explain their corresponding consumer benefits. Second, it can dramatically portray user and usage imagery, brand personality, and other intangibles. Because of the fleeting nature of the ad, however, and the distracting creative elements often found in it, product-related messages and the brand itself can be overlooked. Moreover, the high volume of nonprogramming material on television creates clutter that makes it easy for consumers to ignore or forget ads.

121) How does the Internet help sellers discriminate between buyers and vice-versa?

The Internet helps buyers to: • Get instant price comparisons from thousands of vendors • Name their price and have it met • Get products free The Internet helps sellers to: • Monitor customer behavior and tailor offers to individuals • Give certain customer access to special prices The Internet helps both buyers and sellers to negotiate prices in online auctions and exchanges or even in person. Page Ref: 384-386 Objective: 1 Difficulty: Moderate

141) List some of the marketing trends that are likely to emerge in the near future.

The coming years will see: • The demise of the marketing department and the rise of holistic marketing • The demise of free-spending marketing and the rise of ROI marketing • The demise of marketing intuition and the rise of marketing science • The demise of manual marketing and the rise of both automated and creative marketing • The demise of mass marketing and the rise of precision marketing Page Ref: 648 Objective: 4 Difficulty: Moderate

146) What is the purpose of profitability control?

The purpose of profitability control is to examine where the company is making and losing money. Companies should measure the profitability of their products, territories, customer groups, segments, trade channels, and order sizes to help determine whether to expand, reduce, or eliminate any products or marketing activities.

139) What is the purpose of profitability control?

The purpose of profitability control is to examine where the company is making and losing money. Companies should measure the profitability of their products, territories, customer groups, segments, trade channels, and order sizes to help determine whether to expand, reduce, or eliminate any products or marketing activities. Page Ref: 644 Objective: 5 Difficulty: Moderate

70) List some recent trends in retailing.

The recent trends in retail include the following: • New retail forms and combinations • Growth of giant retailers and nonstore-based retailing • Decline of middle-market retailers • Growing investment in technology • Global profile of major retailers • Growth of shopper marketing

38) Members of a marketing channel perform three types of functions. Provide examples of these three functions in the context of a publishing company that publishes books and magazines.

The storage and movement of books from the company to the customer constitutes a forward flow. Ordering books and paying for them constitute a backward flow from customers to the company. Sharing information and risk taking are activities that occur in both directions.

124) What are the different price-setting methods? Briefly describe each of them.

The six major price-setting methods are: markup pricing, target-return pricing, perceived-value pricing, value pricing, going-rate pricing, and auction-type pricing. • Markup pricing - This is the most elementary pricing method wherein a standard markup is added to the product's cost. • Target-return pricing - Here, the firm determines the price that yields its target rate of return on investment. • Perceived-value pricing - Perceived value is made up of a host of inputs, such as the buyer's image of the product performance, the channel deliverables, the warranty quality, customer support, and softer attributes such as the supplier's reputation, trustworthiness, and esteem. Companies must deliver the value promised by their value proposition, and the customer must perceive this value. • Value pricing - Companies win loyal customers by charging a fairly low price for a highly-quality offering. Value price is just not a matter of lowering the prices but also it is a matter of reengineering the company's operations to become a low-cost producer without sacrificing quality, to attract a large number of value conscious customers. • Going-rate pricing - Here, the firm bases its price largely on competitors' prices. • Auction-type pricing - There are three major types of auctions and their separate pricing procedures: i. English auctions - These have one seller and many buyers. The highest bidder gets the item. ii. Dutch auctions - This features one seller and many buyers, or one buyer and many sellers. In the first kind, an auctioneer announces a high price for a product and then slowly decreases the price until a bidder accepts. In the other, the buyer announces something he wants to buy, and potential sellers compete to offer the lowest price. iii. Sealed-bid auction - This lets would-be suppliers submit only one bid. The suppliers have no knowledge about the other bids. Page Ref: 396-402 Objective: 2 Difficulty: Moderate

145) A British aircraft manufacturer sold planes to Brazil for 70 percent cash and the rest in coffee. This is an example of what kind of a countertrade?

This is an example of a compensation deal. In such deals, the seller receives some percentage of the payment in cash and the rest in products. Page Ref: 404 Objective: 3 Difficulty: Moderate

121) Flash Designs is an apparel manufacturing company and has adopted a franchising model to distribute and sells its garments. The company recently received complaints from a particular franchisee that another competing franchisee was infringing on its territory. What type of conflict is this?

This is an example of horizontal channel conflict, between channel members at the same level.

21) Cortron Consultants provides business consulting services for startups. It helps customers design their supply chains by first evaluating the target market and then proceeds backward from that point. What is this strategy of reverse design called? Briefly explain.

This strategy is called demand chain planning. This helps the companies have a clear focus on the target market.

120) True Value is a retailer-owned cooperative with over 4,000 independent retail locations worldwide. Members of True Value own their individual stores and operate independently. True Value helps its members procure materials at a low cost. What kind of marketing system is True Value? Explain briefly.

True Value is a type of contractual VMS. It is a retail cooperative wherein retailers take the initiative and organize a new business entity to carry on wholesaling and possibly some production.

Which of the following is one of the last decisions management faces when designing a test market?

What action to take?

14) Identify the channel function that constitutes both backward and forward flow. A) obtaining funds for financing B) storage of physical goods C) persuasive communication D) movement of physical goods E) overseeing actual transfer of ownership

a

19) A jobber in a three-level marketing channel is a(n) ________. A) small-scale wholesaler B) external broker C) advertising agent D) independent evaluator E) communication channel

a

78) Which of the following is a major threat facing the brick-and-mortar manufacturers when they add an e-commerce channel? A) It creates the possibility of a backlash from the existing intermediaries. B) It increases the likelihood of product cannibalization. C) Successful implementation leads to a significant increase in operational costs. D) It significantly increases the resource requirements of the organization. E) E-commerce channels often have low potential for attracting customers.

a

80) M-commerce refers to ________. A) conducting business using mobile channels B) the use of mass media communications to attract customers C) providing mobile and on-site services to customers D) the use of a specific medium to communicate with prospects E) using the Internet as a medium for doing business

a

9) A firm uses its sales force to sell to large accounts and outbound telemarketing to sell to medium-sized accounts. The firm is using ________ marketing. A) hybrid B) pull C) personalized D) vertical E) internal

a

35) Electrobar, a European manufacturer of industrial kitchenware, sells to industrial canteens, restaurants, hotels, and so forth. The company provides a one-year warranty on all products and also allows customers to pay in installments-they pay fifty percent on delivery and the rest as equal installments. This refers to which element in the "trade-relations mix"? A) price policy B) conditions of sale C) distributors' territorial rights D) exclusive dealings E) mutual services and responsibilities

b

47) A new firm typically starts as a local operation selling in a fairly circumscribed market by ________. A) finding and developing new intermediaries B) using a few existing intermediaries C) forming partnerships with the market leader D) creating a special channel E) forming partnerships with other firms

b

5) Total Beverages, a maker of fruit juices and health drinks, recently launched a new brand of packaged drinking water called AquaPure. In order to induce distributors to carry the product, Total offers all its intermediaries a free refrigerator to store bottles of AquaPure. This is an example of a ________. A) consumer promotion B) push strategy C) backward flow D) reverse flow E) pull strategy

b

51) A(n) ________ includes the producer, wholesaler(s), and retailer(s) acting as a unified system. A) parallel marketing channel B) vertical marketing system C) extensive marketing channel D) internal marketing system E) conventional marketing channel

b

18) A direct marketing channel is a ________. A) one-level channel B) two-level channel C) zero-level channel D) three-level channel E) reverse-flow channel

c

144) On sites such as eBay and Amazon.com, the seller puts up an item and bidders raise the offer price until the top price is reached. What kind of auctions are these?

eBay and Amazon.com are perfect examples of English auctions. Such auctions have one seller and many buyers and the highest bidder gets the item. Page Ref: 402 Objective: 2 Difficulty: Moderate

12) Companies should first think of the target market and then design the supply chain backward from that point. This strategy is called ________. A) demand chain planning B) resource planning C) external channel planning D) materials planning E) strategic business planning

a

23) Which of the following terms refers to the add-on services, such as credit, delivery, installation, and repairs, provided by a marketing channel? A) service backup B) product accessories C) external products D) product variety E) service extensions

a

43) A manufacturer offers its intermediaries an extra benefit for performing a promotional activity. This is an example of the use of ________ power. A) reward B) coercive C) functional D) expert E) referent

a

63) ________ channel conflict occurs between channel members at the same level. A) Horizontal B) Vertical C) Multichannel D) Administrative E) Contractual

a

4) A manufacturer uses the company's sales force and trade promotions to carry, promote, and sell products to end users. Which of the following strategies is this manufacturer using? A) personalization strategy B) tailoring strategy C) push strategy D) pull strategy E) consumer promotion strategy

c

22) As a service output produced by marketing channels, product variety refers to the ________. A) units the channel permits a customer to purchase at once B) assortment provided by the marketing channel C) add-on services provided by the channel D) ability of a product to provide incremental value E) degree to which the channel makes it easy for customers to purchase a product

b

6) Spike Inc. is a sportswear manufacturer that recently launched its new line of customizable running shoes. The shoes come with a digital component that allows them to adapt to the runner's biomechanics. To promote this new product, Spike launches an advertising campaign and also ropes in a famous athlete to endorse the product. This is an example of a ________. A) trade promotion B) reverse flow C) push strategy D) pull strategy E) backward flow

d

15) Producers often shift some functions to intermediaries. Which of the following is the most significant benefit of doing this? A) It increases customer loyalty. B) It provides the producer with greater control over operations. C) It reduces the amount of direct customer interaction. D) It lowers the producer's costs and prices. E) It ensures greater information security.

d

34) ________ call(s) for the producer to establish a schedule of discounts and allowances that intermediaries see as equitable and sufficient. A) Exclusive dealings B) Mutual services C) Territorial rights D) Price policy E) Tying agreements

d

123) Briefly describe the different types of pricing objectives.

When a company is preparing to sets its price, first of all it has to select its pricing objectives. The five major objectives available to a company are: survival, maximum current profit, maximum market share, maximum market skimming, and product-quality leadership. • Survival - Companies pursue survival as their major objective if they are plagued with overcapacity, intense competition, or changing consumer wants. As long as prices cover variable costs and some fixed costs, the company stays in business. • Maximum current profit - Companies who try to maximize their current profit, estimate the demand and costs associated with alternative prices and choose the price that produces maximum current profit, cash flow, or rate of return on investment. This strategy assumes the firm knows its demand and cost functions, but in reality, these are difficult to estimate. • Maximum market share - Companies that want to maximize their market share believe that a higher sales volume will lead to lower unit costs and higher long-run profit. They set the lowest price, assuming the market is price sensitive. This is a market-penetration pricing strategy. • Maximum market skimming - Companies unveiling a new technology favor setting high prices to maximize market skimming. Companies that use this, introduce their products at a high price and slowly drop the price over time. • Product-quality leadership - Companies that aim to be product quality leaders strive to be affordable luxuries, i.e., they want their products and services to be characterized by high levels of perceived quality, taste, and status with a price just high enough not to be out of the consumer's reach. Page Ref: 389-390 Objective: 2 Difficulty: Moderate

151) What is the full-cost approach of evaluating a marketing entity's performance?

When evaluating a marketing entity's performance, the major controversy is about whether to allocate the nontraceable common costs to the marketing entity. Such allocation is called the full-cost approach, and its advocates argue that all costs must ultimately be imputed in order to determine true profitability. However, this argument confuses the use of accounting for financial reporting with its use for managerial decision making.

145) What is the full-cost approach of evaluating a marketing entity's performance?

When evaluating a marketing entity's performance, the major controversy is about whether to allocate the nontraceable common costs to the marketing entity. Such allocation is called the full-cost approach, and its advocates argue that all costs must ultimately be imputed in order to determine true profitability. However, this argument confuses the use of accounting for financial reporting with its use for managerial decision making. Page Ref: 659 Objective: APP Difficulty: Moderate

Where pricing is key competitive factor (aerospace, railroads, and oil companies), companies often establish a pricing department to set or assist others in setting appropriate prices. According to research, in B-to-B settings, pricing performance improves under what circumstances?

When pricing functions are spread horizontally across the sales, marketing, and finance units

16) Which of the following is an example of a zero-level channel? A) A company takes online orders from customers and ships the products to them. B) An organization uses a combination of direct salespeople and sales agencies to increase sales. C) A company sells its products through wholesalers and retailers. D) A company sells its products through chains of supermarkets and other large sellers. E) A large company forms alliances with smaller companies to increase sales coverage.

a

25) Which of the following types of distribution involves severely limiting the number of channel intermediaries? A) exclusive B) selective C) intensive D) aggressive E) retail

a

28) Which of the following is a major disadvantage of using the Internet as a marketing channel? A) It is less effective for complex products. B) It lacks convenience and practicality. C) It cannot be used to reach a wide audience. D) It is considered expensive. E) It causes the company to lose direct contact with customers.

a

29) Armon Apparels designs, manufactures, and distributes athletic apparel and accessories for men and women. The company has only nine distributors across the United States. These distributors control a nationwide network of 600 retailers. The company does not sell its products through other channels. This is an example of ________ distribution. A) selective B) intensive C) exclusive D) internal E) passive

a

30) Exclusive dealing arrangements are mainly used by companies looking for an edge in markets increasingly driven by ________. A) price B) efficiency C) product variety D) add-on services E) spatial convenience

a

42) ________ power can be effective, but its exercise produces resentment and can lead the intermediaries to organize countervailing power. A) Coercive B) Reward C) Legitimate D) Expert E) Referent

a

48) Who will most likely be willing to pay for high-value-added channels? A) early buyers of a product B) internal customers of a company C) small and matured buyers of an industry D) consumers of low involvement products E) repeat customers of a product

a

50) What is the major difference between a conventional marketing channel and a vertical marketing system (VMS)? A) Elements in a conventional marketing channel act as separate businesses whereas the elements in a VMS act as a unified system. B) A VMS has many intermediaries whereas a conventional marketing channel has a limited number of intermediaries. C) A VMS is characterized by an independent producer whereas a conventional marketing channel is characterized by multiple producers. D) A conventional marketing channel has elements such as retailers and wholesalers whereas these elements are not present in a VMS. E) Producers have complete control over the other members in a conventional marketing channel whereas this control is minimal in a VMS.

a

57) A group of small sellers take the initiative and organize a new business entity to carry on wholesaling and possibly some production. This initiative is called a(n) ________. A) retailer cooperative B) franchise organization C) area-based cartel D) sponsored voluntary chain E) alternate selling channel

a

59) A group of small grocery shops forms a new business entity to buy products directly from manufacturers. The group buys products in bulk which are then distributed among members. This helps the shops obtain better profit margins. Which of the following types of vertical marketing system can be observed here? A) contractual B) corporate C) administered D) controlled E) regulatory

a

62) Which of the following is a major advantage of adding more channels for selling? A) It helps the company increase its market coverage. B) It helps the company reduce its fixed costs. C) It reduces the likelihood of channel conflict. D) It is the best strategy for selling low-involvement consumer products. E) It results in economies of scale.

a

68) RX Corp. is a large manufacturer of electronic goods and sells its products through distributors and retailers. In order to keep pace with the growing use of the Internet, the company decides to start selling online. The company faces stiff opposition from its retailers as they believe that this will significantly reduce their profits. The company attempts to eliminate this resistance by offering its retailers commissions for processing and delivering orders received via the Web. This is an example of which of the following conflict resolution strategies? A) dual compensation B) joint membership C) arbitration D) co-option E) strategic pricing

a

72) Which of the following is the most complete and accurate description of a pure-click company? A) These are companies that launch a Web site without any previous existence as a firm. B) These firms' business models are based on advertising revenue. C) These are established companies that have an online site for selling products. D) These companies get paid every time a user visits their Web site. E) These companies sell online space to other Web sites and are paid for the service.

a

76) Exxon Consulting, works as an agent on behalf of business consumers to collect information on various industrial products. When faced with buying decisions, businesses can approach Exxon to obtain detailed information on the various options available to them. The company earns revenue by selling such information to various customers. Marten Consulting can be called a(n) ________. A) infomediary B) market maker C) customer community D) third party arbitrator E) informant

a

77) Consumer surveys suggest that one of the most significant inhibitors of online shopping is the absence of ________. A) pleasurable experiences B) competitive prices C) adequate technical information D) after-sales service E) facilities to compare offerings

a

79) Which of the following is an example of a brick-and-click company? A) IFB Industries, a company that sells products using various channels such as Internet, retailers, direct outlets, and franchisees. B) Opera, a Web browser and Internet suite developed by Opera Software, performs common tasks such as displaying Web sites and sending and receiving e-mail messages. C) Atrutron, a company that offers its customers access to the Internet using data transmission technology. D) eBay Inc., an American company, facilitates online auctions and shopping to people and businesses across the globe to buy and sell a broad variety of goods and services. E) Yahoo! Inc., an American corporation, provides services via the Internet such as directories, e-mail, news, advertising, online mapping, and so on.

a

53) The most advanced supply-distributor arrangements for ________ vertical marketing systems rely on distribution programming. A) corporate B) administered C) contractual D) regulatory E) controlled

b

56) A(n) ________ VMS consists of independent firms at different levels of production and distribution, integrating their programs on a contractual basis to obtain more economies or sales impact than they could achieve alone. A) administered B) contractual C) corporate D) regulated E) referent

b

58) In a retailer cooperative, ________. A) profits are equally divided among members B) members plan their advertising jointly C) nonmembers cannot buy through the co-op D) members rely on distribution programming E) members standardize their selling practices

b

7) When is a pull strategy appropriate? A) when there is low brand loyalty B) when consumers are able to perceive differences between brands C) when brand choice is made in the store D) when it is a low involvement purchase E) when the product is an impulse item

b

8) Using the push strategy is most appropriate when ________. A) consumers are able to perceive differences between brands B) the product being sold is an impulse item C) there is high brand loyalty for the product D) the product is a high involvement purchase E) consumers choose the brand before they go to the store

b

71) Explain the three distribution strategies based on the number of intermediaries.

• Exclusive distribution: This strategy focuses on severely limiting the number of intermediaries. It's appropriate when the producer wants to maintain control over the service level and outputs offered by the resellers, and it often includes exclusive dealing arrangements. By granting exclusive distribution, the producer hopes to obtain more dedicated and knowledgeable selling. It requires a closer partnership between seller and reseller and is used in the distribution of new automobiles, some major appliances, and some women's apparel brands. • Selective distribution: This distribution strategy relies on only some of the intermediaries willing to carry a particular product. Whether established or new, the company does not need to worry about having too many outlets; it can gain adequate market coverage with more control and less cost than intensive distribution. • Intensive distribution: This strategy places the goods or services in as many outlets as possible. This strategy serves well for snack foods, soft drinks, newspapers, candies, and gum — products consumers buy frequently or in a variety of locations.

1) ________ includes all the activities in selling goods or services directly to final consumers for personal, non-business use. A) Wholesaling B) Retailing C) Procurement D) Promoting E) Warehousing

B

Which of the following is a true statement about price elasticity?

Price elasticity depends on the magnitude and direction of the contemplated price change

156) MPR's contribution to a company's bottom line is the easiest to measure among all the available promotion tools.

FALSE

17) If the product class is mature, then the advertising objective is to convince the market of the brand's superiority.

FALSE

18) Puffery refers to simple exaggerations in advertisements that are not meant to be believed and are considered illegal.

FALSE

19) Advertising elasticities were estimated to be higher for established (0.3) than for new (0.1) products.

FALSE

31) A functional organization allows for adequate planning as the number of products and markets of a firm increases.

FALSE

134) What are the steps involved in developing effective marketing communications?

1. Identify target audience 2. Determine objectives 3. Design communications 4. Select channels 5. Establish budget 6. Decide on media mix 7. Measure results 8. Manage integrated marketing communications

117) List and define some of the important shifts that have taken place in business and marketing practices.

1. Reengineering is the appointment of teams to manage customer-value-building processes and break down walls between departments. 2. Outsourcing involves buying more goods and services from outside domestic or foreign vendors. 3. Benchmarking is the study of "best practice companies" to improve performance. 4. Supplier partnering focuses on partnering with fewer but better value-adding suppliers. 5. Customer partnering entails working more closely with customers to add value to their operations. 6. Merging involves acquiring or merging with firms in the same or complementary industries to gain economies of scale and scope. 7. Globalizing focuses on increasing the effort to "think global" and "act local." 8. Flattening refers to reducing the number of organizational levels to get closer to the customer. 9. Focusing involves determining the most profitable businesses and customers and focusing on them. 10. Accelerating involves designing the organization and setting up processes to respond more quickly to changes in the environment. 11. Empowering means encouraging and empowering personnel to produce more ideas and take more initiative. 12. Justifying means becoming more accountable by measuring, analyzing, and documenting the effects of marketing actions. 13. Broadening involves factoring the interests of customers, employees, shareholders, and other stakeholders into the activities of the enterprise. 14. Monitoring involves tracking what is said online and elsewhere and studying customers, competitors, and others to improve business practices. Page Ref: 624 Objective: 1 Difficulty: Moderate

Gap and Pottery Barn are examples of​ ________, where two or more retail outlets are owned and controlled by a corporation that employs central buying and merchandising and sells similar lines of merchandise. A. corporate chain stores B. franchise organizations C. consumer cooperatives D. retailer cooperatives E. merchandising conglomerates

A

32) Marketing channel functions such as storage and movement and communications constitute a backward flow of activities.

FALSE

12) Bill and Josh are considering opening a retail store. They have identified their target market and location and are finalizing the details of the merchandise they will carry. Since the neighborhood is rundown and the customers in the area are very price-conscious, Bill and Josh want to offer goods from well-known brands, but at lower rates than the full retail prices of the products. They choose to stock excess production from manufacturers or goods that have remained unsold at other retailers. This is a description of a(n) ________ retailer. A) off-price B) specialty C) discount D) department E) catalog

A Diff: 2 LO: 18.1: What major types of marketing intermediaries occupy this sector? AACSB: Analytical thinking; Application of knowledge

32) Which of the following is a benefit of franchising for franchisees? A) The franchisee finds it easier to borrow money from financial institutions. B) The franchisee receives ownership of the franchisor's trademark. C) The franchisee must change its operations to suit those of the franchisor. D) The franchisee collects royalty payments from the franchisor. E) The franchisee is paid by the franchisor for being part of the system.

A Diff: 2 LO: 18.1: What major types of marketing intermediaries occupy this sector? AACSB: Reflective thinking

58) Which of the following is true for the retail industry? A) Discount stores and catalog showrooms are competing for the same customers. B) Upscale retailers see a decline in sales as middle-market retailers thrive. C) Small, specialized retailers are crowding out larger, more diverse retailers. D) Store retailing sees no competition from nonstore retailing. E) Discount stores are not doing as well as middle-market retailers.

A Diff: 2 LO: 18.2: What major changes are occurring in the modern retail marketing environment with respect to competitive market structure and technology? AACSB: Reflective thinking

83) In your neighborhood there is a small men's store that has a limited selection of clothing, but the selection that is carried is of very high quality and price. Services include free alterations and tailoring, personalized recordkeeping, and free dry cleaning. Inside the store are deep leather chairs and couches and thick pile carpet. Upon entering the store, a customer feels "special and rich." In terms of differentiation, what is this retailer trying to communicate by its decorations and service level? A) differentiation based on services mix and atmosphere B) differentiation based on prepurchase services C) differentiation based on postpurchase services and atmosphere D) differentiation based on ancillary services and atmosphere E) differentiation based on prepurchase and postpurchase services

A Diff: 2 LO: 18.3: What marketing decisions do marketing intermediaries make? AACSB: Application of knowledge

79) Which of the following is a prepurchase service offered by retailers? A) accepting orders over the telephone B) shipping the product C) delivery to the customer's doorstep D) general information E) interior decoration of the retail outlet

A Diff: 2 LO: 18.3: What marketing decisions do marketing intermediaries make? AACSB: Reflective thinking

120) ________ refers to buying large carload lots and dividing them into smaller units before shipping them out to consumers. A) Bulk breaking B) Containerization C) Wholesaling D) Warehousing E) Broking

A Diff: 2 LO: 18.5: What are some of the important issues in wholesaling? AACSB: Analytical thinking

130) ________ encompass sales forecasting, production planning, and inbound materials transportation. A) Market logistics B) Containerization C) Transportation D) Nonstore retailing E) Wholesaling

A Diff: 2 LO: 18.6: What are some important issues in logistics? AACSB: Analytical thinking

139) Beyond the optimal order quantity, total cost per unit increases because ________. A) inventory-carrying cost per unit increases B) inventory-carrying cost per unit decreases C) order-processing cost per unit increases D) order-processing cost per unit increases though inventory cost decreases E) inventory-processing cost per unit falls slowly

A Diff: 2 LO: 18.6: What are some important issues in logistics? AACSB: Analytical thinking

141) ________ consists of putting goods in boxes or trailers that are easy to transfer between two transportation modes. A) Containerization B) Haulage C) Inventory carrying D) Order processing E) Warehousing

A Diff: 2 LO: 18.6: What are some important issues in logistics? AACSB: Analytical thinking

15) Which of the following strategies for new-product development incorporates buyers' preferences in the final design of the product? A) quality function deployment B) market leadership C) cost leadership D) incremental innovation E) disruptive technology

A Diff: 3 LO: 15.2: What challenges does a company face in developing new products and services? AACSB: Reflective thinking

77) Most retailers will put low prices on some items in order to increase traffic to the store. These low-priced products are known as ________. A) loss leaders B) price ceilings C) price skimmers D) price floors E) cold calls

A Diff: 3 LO: 18.3: What marketing decisions do marketing intermediaries make? AACSB: Analytical thinking

14) Companies who believe that a higher sales volume leads to lower unit costs and higher long-run profits are attempting to ________. A) maximize their market share B) skim the market C) become a product-quality leader D) merely survive in the market E) maximize their current profits

A Page Ref: 389 Objective: 2 Difficulty: Easy

24) Consumers are less price sensitive when ________. A) price is only a small part of the total cost spent on the product over its lifetime B) they perceive the higher prices to be unjustified C) they change their buying habits regularly D) there are many substitutes and competitors in the market E) they are buying high-cost items

A Page Ref: 391 Objective: 2 Difficulty: Moderate

27) If consumers were largely indifferent to a $0.5 increase in the price of a gallon of milk, the price rise is said to fall within customers' ________. A) price indifference band B) experience curve C) arm's-length price D) learning curve E) net price index

A Page Ref: 392 Objective: 2 AACSB: Analytic skills Difficulty: Moderate

35) Experience-curve pricing ________. A) assumes competitors are weak followers B) allows products to project a high quality image C) is applicable only to manufacturing costs D) focuses on reducing fixed costs E) is generally risk-free

A Page Ref: 394 Objective: 2 Difficulty: Moderate

41) Which of the following marketing communications tools is most influential at the maturity stage of a product's life cycle? A) sales promotions B) direct marketing C) advertising D) publicity E) interactive marketing

C

87) What are some of the forces that are driving companies to practice corporate social responsibility?

A number of forces are driving companies to practice a higher level of corporate social responsibility, such as rising customer expectations, evolving employee goals and ambitions, tighter government legislation and pressure, investor interest in social criteria, media scrutiny, and changing business procurement practices.

130) What are some of the forces that are driving companies to practice corporate social responsibility?

A number of forces are driving companies to practice a higher level of corporate social responsibility, such as rising customer expectations, evolving employee goals and ambitions, tighter government legislation and pressure, investor interest in social criteria, media scrutiny, and changing business procurement practices. Page Ref: 631 Objective: 3 AACSB: Ethical understanding and reasoning abilities Difficulty: Easy

109) The growth of "house brands" has skyrocketed in recent years. What benefits do intermediaries receive from sponsoring their own brands?

A private label brand (also called a reseller, store, house, or distributor brand) is a brand that retailers and wholesalers develop. These brands can be more profitable. Intermediaries search for manufacturers with excess capacity that will produce private label goods at low cost. Other costs, such as research and development, advertising, sales promotion, and physical distribution, are also much lower, so private labels can generate a higher profit margin. Retailers also develop exclusive store brands to differentiate themselves from competitors. Many price-sensitive consumers prefer store brands in certain categories. These preferences give retailers increased bargaining power with marketers of national brands.

117) What is a vertical marketing system (VMS)? What are the various types of VMSs?

A vertical marketing system includes the producer, wholesaler(s), and retailer(s) acting as a unified system. One channel member, the channel captain, owns or franchises the others or has so much power that they all cooperate. There are three types of VMSs: corporate, administered, and contractual. • A corporate VMS combines successive stages of production and distribution under single ownership. • An administered VMS coordinates successive stages of production and distribution through the size and power of one of the members. Manufacturers of dominant brands can secure strong trade cooperation and support from resellers. • A contractual VMS consists of independent firms at different levels of production and distribution, integrating their programs on a contractual basis to obtain more economies or sales impact than they could achieve alone.

37) Briefly explain the various levels of marketing channels.

A zero-level channel, also called a direct marketing channel, consists of a manufacturer selling directly to the final customer. The major examples are door-to-door sales, home parties, mail order, telemarketing, TV selling, Internet selling, and manufacturer-owned stores. A one-level channel contains one selling intermediary, such as a retailer. A two-level channel contains two intermediaries. In consumer markets, these are typically a wholesaler and a retailer. A three-level channel contains three intermediaries. In the meatpacking industry, wholesalers sell to jobbers, essentially small-scale wholesalers, who sell to small retailers.

Which of the following elements of the marketing communications mix refers to any paid form of nonpersonal presentation and promotion of ideas, goods, or services by an identified sponsor via print, broadcast, network, electronic, and display media? A) advertising B) personal selling C) sales promotion D) direct marketing E) public relations

A) Advertising

Marketing communication strategy can be decided by conducting an image analysis by profiling the target audience in terms of ________. A) brand knowledge B) purchase patterns C) demographic characteristics D) income levels E) psychographic characteristics

A) Brand Knowledge

Top Gear is an award-winning British television series about motor vehicles, mainly cars. It is presented by hosts who test drive new cars and provide reviews on the cars' performance, their prices, and other factors. Which of the following personal communications channels is Top Gear closest to in description? A) expert channel B) formal channel C) social channel D) sponsored channel E) advocate channel

A) Expert Channel

Qualities such as candor, humor, and naturalness make a spokesperson ________, which is an important component of spokesperson credibility. A) likable B) an expert C) trustworthy D) transformational E) informational

A) Likable

Which of the following is an example of a public relations and publicity communication platform? A) lobbying B) company museums C) street activities D) company blogs E) incentive programs

A) Lobbying

________ encompass sales forecasting, production planning, and inbound materials transportation. A) Market logistics B) Containerization C) Transportation D) Nonstore retailing E) Wholesaling

A) Market logistics

Which of the following statements correctly reflects a characteristic of public relations as a marketing communications tool? A) Public relations can reach prospects who prefer to avoid mass media and targeted promotions. B) They incorporate some concession, inducement, or contribution that gives value to the consumer. C) Given their live, real-time quality, public relations tools are more actively engaging for consumers. D) Public relations communications can be prepared to appeal to the addressed individual. E) Public relations tools create an immediate and interactive episode between two or more persons.

A) Public relations can reach prospects who prefer to avoid mass media and targeted promotions.

Which of the following is an example of a personal selling communication platform? A) sales presentations B) company blogs C) telemarketing D) TV shopping E) press kits

A) Sales Presentations

Which of the following is a benefit of franchising for franchisees? A) The franchisee finds it easier to borrow money from financial institutions. B) The franchisee receives ownership of the franchisor's trademark. C) The franchisee must change its operations to suit those of the franchisor. D) The franchisee collects royalty payments from the franchisor. E) The franchisee is paid by the franchisor for being part of the system.

A) The franchisee finds it easier to borrow money from financial institutions.

Which of the following ads depict an informational appeal? A) Thompson Water Seal can withstand intense rain, snow, and heat. B) The California Milk Processor Board ran the successful "Got Milk?" ad to boost declining sales. C) VW advertised to active, youthful people with its famed "Drivers Wanted" campaign. D) Pringles advertised "Once You Pop, the Fun Don't Stop" for years. E) KFC has marketed its fast-food products under the slogan, "Finger Lickin' Good."

A) Thompson Water Seal can withstand intense rain, snow, and heat.

Which of the following is the correct order of stages that a buyer is assumed to pass through, by the four classic response hierarchy models? A) cognitive stage — affective stage — behavioral stage B) affective stage — cognitive stage — behavioral stage C) behavioral stage — affective stage — cognitive stage D) cognitive stage — behavioral stage — affective stage E) affective stage — behavioral stage — cognitive stage

A) cognitive stage — affective stage — behavioral stage

37) When customers fall into different user groups with distinct buying preferences and practices, a product team structure is desirable.

FALSE

In your neighborhood there is a small men's store that has a limited selection of clothing, but the selection that is carried is of very high quality and price. Services include free alterations and tailoring, personalized recordkeeping, and free dry cleaning. Inside the store are deep leather chairs and couches and thick pile carpet. Upon entering the store, a customer feels "special and rich." In terms of differentiation, what is this retailer trying to communicate by its decorations and service level? A) differentiation based on services mix and atmosphere B) differentiation based on prepurchase services C) differentiation based on postpurchase services and atmosphere D) differentiation based on ancillary services and atmosphere E) differentiation based on prepurchase and postpurchase services

A) differentiation based on services mix and atmosphere

Most retailers will put low prices on some items in order to increase traffic to the store. These low-priced products are known as ________. A) loss leaders B) price ceilings C) price skimmers D) price floors E) cold calls

A) loss leaders

A factory outlet is an example of a(n) ________ retailer. A) off-price B) specialty C) discount D) department E) catalog

A) off-price

Which of the following possible consumer reference prices is generally unavailable to a consumer?

Actual Future Price

74) Alan is an executive with an ad agency that has been entrusted with accounts for a used-car showroom, a home appliances maker, and a soap company. With reference to response hierarchy models, how does Alan plan communication strategies for the three accounts effectively?

All of the four classic response hierarchy models assume that the buyer passes through cognitive (learn), affective (feel), and behavioral stages (do), in that order. Since buying a car, used or new, represents a significant investment for the buyer, it can be said that the buyer has involvement in the purchase decision and perceives high differentiation with the product category. Hence a "learn-feel-do" sequence is considered appropriate for the used-car showroom account. Similarly, a buyer intending to purchase home appliances, such as a dishwasher or a refrigerator, has high involvement in the purchase decision, even though he perceives little differentiation within the product category. Hence, a "do-feel-learn" approach may be appropriate when planning communications for the home appliances account. Finally, a buyer has low involvement in purchasing soap that has very little differentiation within its category. For the soap company account, Alan should consider a "learn-do-feel" sequence for planning communications.

122) What are the different possible consumer reference prices?

Although consumers have fairly good knowledge of price ranges, surprisingly few can accurately recall specific prices. When examining prices, consumers often employ reference prices, comparing an observed price to an internal reference price they remember or an external frame of reference such as a posted "regular retail price." These reference prices include: • Fair price - what consumers feels the product should cost • Typical price • Last price paid • Upper-bound price - reservation price or the maximum most consumers would pay • Lower-bound price - lower threshold price or the minimum most consumers would pay • Historical competitor price • Expected future price • Usual discounted price Page Ref: 387 Objective: 1 Difficulty: Moderate

51) What is the role of advertising as a communications tool in marketing for business markets?

Although marketers rely more on sales calls in business markets, advertising still plays a significant role: • Advertising can provide an introduction to the company and its products. • If the product has new features, advertising can explain them. • Reminder advertising is more economical than sales calls. • Advertisements offering brochures and carrying the company's phone number or Web address are an effective way to generate leads for sales representatives. • Sales representatives can use copies of the company's ads to legitimize their company and products. • Advertising can remind customers how to use the product and reassure them about their purchase.

48) List the four categories of nonstore retailing and briefly explain each.

Although the overwhelming bulk of goods and services are sold through stores, nonstore retailing has been growing much faster than store retailing. Nonstore retailing falls into four major categories: 1. Direct selling, also called multilevel selling and network marketing, is a multibillion-dollar industry, with hundreds of companies selling door-to-door or at home sales parties. A salesperson goes to the home of a host who has invited friends; the salesperson demonstrates the products and takes orders. Pioneered by Amway, the multilevel (network) marketing sales system works by recruiting independent businesspeople who act as distributors. The distributor's compensation includes a percentage of sales made by those he or she recruits, as well as earnings on direct sales to customers. These direct-selling firms, now finding fewer consumers at home, are developing multidistribution strategies. 2. Direct marketing has roots in direct-mail and catalog marketing. It includes telemarketing, television direct-response marketing, and electronic shopping. As people become more accustomed to shopping on the Internet, they are ordering a greater variety of goods and services from a wider range of Web sites. 3. Automatic vending offers a variety of merchandise, including impulse goods such as soft drinks, coffee, candy, newspapers, magazines, and other products such as hosiery, cosmetics, hot food, and paperbacks. Vending machines are found in factories, offices, large retail stores, gasoline stations, hotels, restaurants, and many other places. They offer 24-hour selling, self-service, and merchandise that is stocked to be fresh. 4. Buying service is a storeless retailer serving a specific clientele, usually employees of large organizations, who are entitled to buy from a list of retailers that have agreed to give discounts in return for membership.

To protect a company from inflation on contracts for major industrial projects, such as aircraft construction or bridge building, _______ requires the customer to pay today's price and all or part of any inflation increase that takes place before delivery.

An Escalator Clause

38) Discount stores usually stock leftover goods, overruns, and irregular merchandise, sold at less than retail.

FALSE

125) Briefly explain the concept of annual-plan control.

Annual-plan control ensures the company achieves the sales, profits, and other goals established in its annual plan. At its heart is management by objectives. First, management sets monthly or quarterly goals. Second, it monitors performance in the marketplace. Third, management determines the causes of serious performance deviations. Fourth, it takes corrective action to close gaps between goals and performance. This control model applies to all levels of the organization. Top management sets annual sales and profit goals; each product manager, regional district manager, sales manager, and sales rep is committed to attaining specified levels of sales and costs. Each period, top management reviews and interprets the results. Marketers today have better marketing metrics for measuring the performance of marketing plans. Four tools for the purpose are sales analysis, market share analysis, marketing expense-to-sales analysis, and financial analysis. Page Ref: 643-644 Objective: 5 Difficulty: Moderate

100) E&OE is looking to reduce its inventory costs for all its products. The company realizes that its inventory depends on the setup costs of its various products. How do setup costs affect E&OE's inventory costs?

As inventory draws down, management must know at what stock level to place a new order. The company needs to balance order-processing costs and inventory-carrying costs. Order-processing costs for E&OE consist of setup costs and running costs (operating costs when production is running) for the item. If setup costs are low, E&OE can produce the item often, and the average cost per item is stable and equal to the running costs. If setup costs are high, E&OE can reduce the average cost per unit by producing a long run and carrying more inventory. The larger the average stock carried, the higher the inventory-carrying costs.

89) ________ is one of the sources of a spokesperson's credibility that refers to the specialized knowledge that he or she claims to possess. A) Trustworthiness B) Expertise C) Acquaintance D) Likability E) Professionalism

B

29) Intercom Inc., together with its subsidiaries, primarily engages in the generation, transmission, and distribution of electric power in the United States. The company observes that its growth has stagnated over a period of two years. In an attempt to promote growth, it considers adding new features to the existing products and introducing a few new products. The company forms a committee consisting of three top executives, one of the production mangers, a few operational managers, and a representative of the HR department to generate ideas. This team is called a(n) ________ team. A) virtual B) venture C) fundamental D) elemental E) transitory

B Diff: 2 LO: 15.3: What organizational structures and processes do managers use to oversee new-product development? AACSB: Interpersonal relations and teamwork

87) Universal Services Inc. provides communication services to residential and business customers in rural and small urban communities primarily in northern England. The company offers services such as local and long distance voice, data, and Internet and broadband product offerings. The company, in an attempt to increase the attractiveness of its offerings, decides to provide special voice and data packages to its customers. The company designs eight different packs that offer varying voice and data benefits to customers. The company then asks a few of its customers to rank the packs in order to choose two best packs. Which of the following testing methods is being used in this scenario? A) virtual reality testing B) conjoint analysis C) perceptual mapping D) product fabrication E) rapid prototyping

B Diff: 2 LO: 15.6: What is the best way to manage concept and strategy development? AACSB: Analytical thinking; Application of knowledge

8) In the ________ type of retailing, customers usually find their own goods, although they can ask salespeople for assistance. A) self-service B) self-selection C) full-service D) limited service E) limited-selection

B Diff: 2 LO: 18.1: What major types of marketing intermediaries occupy this sector? AACSB: Analytical thinking

16) Aldi, Lidl, Dollar General, and Family Dollar are examples of ________, as they carry a more restricted merchandise mix than discount stores at even lower prices. A) off-price retailers B) extreme value stores C) superstores D) convenience stores E) specialty stores

B Diff: 2 LO: 18.1: What major types of marketing intermediaries occupy this sector? AACSB: Application of knowledge

84) Which of the following is the strongest differentiator for brick-and-mortar stores who want to emphasize their superiority over online retailers? A) product quality B) the shopping experience C) product range D) pricing E) the retailer's reputation

B Diff: 2 LO: 18.3: What marketing decisions do marketing intermediaries make? AACSB: Analytical thinking

135) A stock order point of 10 means ordering the product ________. A) every 10 days B) when stock falls to 10 units C) every 10 units D) when stock falls to 9 units E) in batches of 10 items

B Diff: 2 LO: 18.6: What are some important issues in logistics? AACSB: Analytical thinking

82) Stores are using ________ to measure a product's handling costs from the time it reaches the warehouse until a customer buys it in the retail store. A) electronic data interchange (EDI) B) direct product profitability (DPP) C) radio-frequency identification (RFID) D) global positioning systems (GPS) E) compounded annual grown rate (CAGR)

B Diff: 3 LO: 18.3: What marketing decisions do marketing intermediaries make? AACSB: Analytical thinking

3) While shopping at the mall, Jane was asked by one of the sales representatives at the cosmetics counter to try out a new lipstick that her company was test marketing. The company representative asks her how much she would be willing to pay for the lipstick. After trying it out, Jane is of the opinion that $5 is just the right price for it. What type of a reference price is Jane using? A) usual discounted price B) fair price C) maximum retail price D) last price paid E) historical competitor price

B Page Ref: 387 Objective: 1 AACSB: Analytic skills Difficulty: Moderate

66) In ________, the seller charges different amounts to different classes of buyers. A) perceived value pricing B) third-degree price discrimination C) first-degree price discrimination D) second-degree price discrimination E) psychological discounting

B Page Ref: 406 Objective: 3 Difficulty: Easy

72) The airline and hospitality industries use ________, by which they offer discounted but limited early purchases, higher-priced late purchases, and the lowest rates on unsold inventory just before it expires. A) special-customer pricing B) yield pricing C) cash rebates D) location pricing E) customer-segment pricing

B Page Ref: 406 Objective: 3 Difficulty: Easy

16) ________ are informal workplaces, sometimes garages, where intrapreneurial teams attempt to develop new products. A) Stage-gate systems B) Skunkworks C) Funnels D) Opportunity spaces E) Research centers

B Page Ref: 574 Objective: 2 Difficulty: Easy

44) Universal Services, Inc. provides communication services to residential and business customers in rural and small urban communities primarily in northern England. The company offers services such as local and long distance voice, data, and Internet and broadband product offerings. The company, in an attempt to increase the attractiveness of its offerings, decides to provide special voice and data packages to its customers. The company designs eight different packs that offer varying voice and data benefits to customers. The company then asks a few of its customers to rank the packs in order to choose two best packs. Which of the following testing methods is being used in the above scenario? A) virtual reality testing B) conjoint analysis C) perceptual mapping D) product fabrication E) rapid prototyping

B Page Ref: 583 Objective: 4 AACSB: Analytic skills Difficulty: Moderate

63) A(n) ________ is any good, service, or idea that is perceived as new, no matter how long its history. A) commodity B) innovation C) adoption D) invention E) novel product

B Page Ref: 591 Objective: 5 Difficulty: Easy

7) As a business practice, broadening involves _________. A) acquiring or merging with firms in the same or complementary industries to gain economies of scale and scope B) factoring the interests of customers, employees, shareholders, and other stakeholders into the activities of the enterprise C) buying more goods and services from outside domestic or foreign vendors D) appointing teams to manage customer-value-building processes and break down walls between departments E) becoming more accountable by measuring, analyzing, and documenting the effects of marketing actions

B Page Ref: 624 Objective: 1 Difficulty: Moderate

105) Gabrielle is the chief marketing officer of Boyd Pharmaceuticals. She is meeting with Trent, the chief financial officer to decide on the company's marketing communications budget. They decide to trust in the prevailing collective wisdom of the industry as a whole, and not wanting to instigate a communications war, settle on spending only as much as their nearest market rival does on marketing communications. What method did Gabrielle and Trent use to arrive at the marketing communications budget? A) objective-and-task method B) affordable method C) competitive-parity method D) activity-based method E) percentage-of-sales method

C

11) ________ is an element of the marketing communications mix that involves people-to-people oral, written, or electronic communications that relate to the merits or experiences of purchasing or using products or services. A) Personal selling B) Sales promotion C) Word-of-mouth marketing D) Public relations E) Advertising

C

12) Which of the following observations is true? A) In today's business environment, marketers are mainly middlemen. B) Marketing has the sole ownership of customer interactions. C) Only when all employees realize their job is to create, serve, and satisfy customers does the company become an effective marketer. D) Marketers must clearly differentiate all customer-facing processes, such that every customer receives a personalized marketing message. E) Marketers must avoid collaborating with other organizational departments, as it blurs functional responsibilities.

C

133) Which of the following factors forms the basis of assessing sponsorship activities through supply-side methods? A) consumers' brand knowledge B) impact on sponsor's bottom line C) extent of media coverage D) brand exposure reported by consumers E) sales pattern of sponsored products

C

15) Which of the following is the main advantage of a functional marketing organization? A) easy coordination B) lower staff requirements C) administrative simplicity D) reduced competition between functional groups E) smooth working relationships

C

23) One of the options in a product-management organization is to eliminate product manager positions for minor products and assign two or more products to each remaining manager. Under what conditions is this alternative feasible? A) when the product mix is highly diverse B) when there are very few products in the company's portfolio C) when two or more products serve a similar set of needs D) when customers fall into different user groups E) when the company produces many products for many markets

C

26) A(n) ________ is a storeless retailer serving a specific clientele — usually employees of large organizations — who are authorized to buy from a list of retailers that have agreed to give discounts in return for inclusion on the list. A) direct-selling vendor B) direct marketing vendor C) buying service D) automatic vendor E) corporate retailer

C

27) A direct marketing channel is a ________ channel. A) one-level B) two-level C) zero-level D) three-level E) reverse-flow

C

32) TV advertising is considered to be particularly advantageous because ________. A) the low volume of nonprogramming material on television makes it difficult for consumers to ignore or forget ads B) it provides detailed product information and effectively communicates user and usage imagery C) it can vividly demonstrate product attributes and persuasively explain their corresponding consumer benefits D) TV channels are very targeted, ads are relatively inexpensive to produce and place, and short closings allow for quick response E) it lets companies achieve a balance between broad and localized market coverage

C

4) ________ advertising aims to create liking, preference, conviction, and purchase of a product or service. A) Corporate B) Reminder C) Persuasive D) Reinforcement E) Informational

C

56) Cadbury's "Sports for Schools" promotion offered sports and fitness equipment for schools in exchange for vouchers. The problem was that the public and media saw a perverse incentive for children to eat more chocolate, a product associated with obesity. Which of the following best summarizes Cadbury's problem? A) Customers felt that the cause was not in sync with the company's brand image. B) Consumers did not value the cause Cadbury was promoting. C) Customers questioned the link between the product and the cause and saw the firm as self-serving and exploitive. D) Consumers resented being sold an inferior product on the back of a cause-marketing program. E) Consumers felt that the campaign did not make a sufficient attempt to change the target audience's behavior.

C

59) Pepe Homes manufactures a range of bathroom accessories and fittings that include bathtubs, shower stalls, etc. While deciding on an effective advertising media vehicle for its planned advertising campaign, the marketing department decides that its advertising objectives would be best achieved if they used a medium that would portray their brand as prestigious and a symbol of luxury. To this effect, the medium should offer marketers a high degree of audience selectivity and high-quality reproduction. Which of the following advertising media would best serve the advertising purposes of Pepe Homes? A) outdoor media B) radio C) magazines D) television E) newspapers

C

59) Social marketing programs designed to discourage cigarette smoking or excessive consumption of alcohol are examples of ________ campaigns. A) cognitive B) active C) behavioral D) value E) normative

C

7) In ________ retailing, salespeople are ready to assist in every phase of the "locate-compare-select" process. A) self-service B) self-selection C) full-service D) limited service E) limited-selection

C

100) Which of the following is NOT true about the two-step process by which mass communications affect personal attitudes? A) The influence of mass media is mediated by opinion leaders, people whose opinions others seek or who carry their opinions to others. B) The influence of mass media on public opinion is not as direct, powerful, and automatic as marketers have supposed. C) The two-step flow supports the notion that consumption styles are primarily influenced by a trickle-up or trickle-down effect from mass media. D) People interact primarily within their own social groups and acquire ideas from opinion leaders in their groups. E) Two-step communication suggests that mass communicators should direct messages specifically to opinion leaders and let them carry the message to others.

C

108) A ________ is a comprehensive, systematic, independent, and periodic examination of a company's or business unit's marketing environment, objectives, strategies, and activities, with a view to determining problem areas and opportunities and recommending a plan of action to improve the company's marketing performance. A) marketing plan B) test market C) marketing audit D) market-based scorecard analysis E) marketing metric

C

109) A franchise organization is an example of a(n) ________ vertical marketing system. A) corporate B) administered C) contractual D) regulatory E) controlled

C

110) A ________ is a sales force promotion tool that aims at inducing the sales force or dealers to increase their sales results over a stated period, with prizes (money, trips, gifts, or points) going to those who succeed. A) trade show B) frequency program C) sales contest D) sweepstake E) patronage award

C

110) In a ________ marketing system, two or more unrelated companies put together resources or programs to exploit an emerging marketing opportunity. A) reverse flow B) vertical C) horizontal D) lateral E) forward flow

C

113) Which of the four characteristics of a marketing audit is described by the fact that outside consultants can bring necessary objectivity to the process? A) comprehensive B) systematic C) independent D) periodic E) opinion-based

C

116) ________ sell and deliver a limited line of semiperishable goods to supermarkets, grocery stores, hospitals, restaurants, and hotels. A) Producers' cooperatives B) Cash and carry wholesalers C) Truck wholesalers D) Drop shippers E) Rack jobbers

C

119) Which of the following is true of brokers? A) Brokers represent buyers or sellers on a semipermanent basis. B) Most brokers are small businesses with a few skilled salespeople. C) Brokers bring buyers and sellers together and assist in negotiation. D) Selling brokers have contractual authority to sell a manufacturer's entire output. E) Purchasing brokers make purchases for buyers and often receive, inspect, warehouse, and ship merchandise.

C

131) Audi models featured prominently in the 2010 blockbuster Iron Man 2, including main character Tony Stark's personal R8 Spyder. Which of the following is the most rational explanation for Audi's decision to associate itself with the movie? A) to express commitment to the community or on social issues B) to identify with a particular target market or lifestyle C) to create experiences and evoke feelings D) to become part of a personally relevant moment in consumers' lives E) to entertain key clients or reward key employees

C

136) Optimal order quantities exist when the curves for the order-processing cost per unit and inventory-carrying cost per unit ________. A) are collinear B) are diagonal to each other C) intersect D) are parallel to each other E) equal zero

C

137) E&OE is trying to minimize its inventory costs, which are extremely high. The company has realized that it can achieve this by maintaining a near-zero inventory and producing more products only once it is ordered. Which of the following is true for E&OE? A) Inventory costs are lower than order-processing costs. B) Running costs are higher than inventory-carrying costs. C) Setup costs for the products are low. D) Order-processing costs are high. E) Order-processing costs are lower than setup costs.

C

95) Sales promotion expenditures increased as a percentage of budget expenditure for a number of years. Growth has, however, recently slowed. Which of the following is a factor that has contributed to the growth of sales promotion expenditures? A) Many brands have come to be seen as dissimilar. B) The efficiency of advertising as a promotion has improved. C) The trade demands more deals from manufacturers. D) Consumers have become less price-oriented. E) The number of brands in the market has decreased.

C

97) Which of the following is a form of mass communications channel? A) interactive marketing B) personal selling C) public relations D) word-of-mouth marketing E) sales presentations

C

22) Gordon Apparel, Inc. engages in the design, manufacture, and retail of fashion apparel for women, men, children, and pets. The company is known for its timely launch of apparels and stylish designer clothing. The company believes a new product has to create needs and not just satisfy the existing needs. To achieve this goal it engages in controlled idea generation techniques such as brainstorming while creating new designs. Which of the following parts of the DIG framework is given prominence here? A) business analysis B) strategic blueprint C) opportunity space D) internal design analysis E) demand landscape

C Page Ref: 576 Objective: 3 AACSB: Analytic skills Difficulty: Moderate

23) Identify the element in the DIG network that considers how a new product can fit into customers lives and how it can be distinguished from the competitors' products. A) business analysis B) internal design analysis C) strategic blueprint D) demand landscape E) opportunity space

C Page Ref: 576 Objective: 3 Difficulty: Easy

29) In an attempt to improve the product, a company that manufactures screwdrivers discusses the modification of each attribute, such as replacing the wooden handle with plastic, providing torque power, adding different screw heads, and so on. This creativity technique is called ________. A) attribute listing B) mind mapping C) morphological analysis D) lateral analysis E) reverse analysis

C Page Ref: 579 Objective: 3 Difficulty: Easy

31) ________ works by listing all the normal assumptions about an entity and then turning them around. A) Lateral marketing B) Attribute listing C) Reverse assumption analysis D) Forced relationships E) Morphological analysis

C Page Ref: 579 Objective: 3 Difficulty: Easy

51) The highest loss a project can create is called ________. A) rapid prototyping income B) payback income C) maximum investment exposure D) incremental yearly exposure E) cannibalized income

C Page Ref: 586 Objective: 4 Difficulty: Easy

55) In consumer-goods market testing, the company seeks to estimate four variables. These four variables are: trial, first repeat, adoption, and ________. A) guaranteed response B) price sensitivity C) purchase frequency D) usage convenience E) preferential treatment

C Page Ref: 588 Objective: 4 Difficulty: Easy

61) Poga International, a multinational beverage corporation identifies that one of its competitors is launching an apple flavored drink. The company decides to launch an apple flavor brand along with its competitor. What timing strategy is used here? A) first entry B) blunt entry C) parallel entry D) late entry E) exchange entry

C Page Ref: 590 Objective: 4 AACSB: Analytic skills Difficulty: Moderate

67) Armordo is a famous vacuum cleaner brand in Africa. Clara has heard of Armordo and she knows that the product is a success. She has started considering whether or not to buy the vacuum cleaner. What stage of the consumer-adoption process is Clara in? A) awareness B) adoption C) evaluation D) trial E) interest

C Page Ref: 591 Objective: 5 AACSB: Analytic skills Difficulty: Moderate

65) Identify the stage in the consumer adoption process in which the customer is stimulated to seek information about the innovation. A) awareness B) adoption C) interest D) evaluation E) trial

C Page Ref: 591 Objective: 5 Difficulty: Easy

8) As a marketing practice, monitoring involves ________. A) designing the organization and setting up processes to respond quickly to changes in the environment B) becoming more accountable by measuring, analyzing, and documenting the effects of marketing actions C) tracking what is said online and elsewhere and studying customers, competitors, and others to improve business practices D) determining the most profitable businesses and customers and expending greater organizational resources to capitalize on them E) factoring the interests of customers, employees, shareholders, and other stakeholders into the activities of the enterprise

C Page Ref: 624 Objective: 1 Difficulty: Moderate

10) Which of the following observations is true? A) In today's business environment, marketers are mainly middlemen. B) Marketing has the sole ownership of customer interactions. C) Only when all employees realize their job is to create, serve, and satisfy customers does the company become an effective marketer. D) Marketers must clearly differentiate all customer-facing processes, such that every customer receives a personalized marketing message. E) Marketers must avoid collaborating with other organizational departments, as it blurs functional responsibilities.

C Page Ref: 625 Objective: 2 Difficulty: Moderate

13) Which of the following is the main advantage of a functional marketing organization? A) easy coordination B) lower staff requirements C) administrative simplicity D) reduced competition between functional groups E) smooth working relationships

C Page Ref: 627 Objective: 2 Difficulty: Moderate

18) A product-management organization ________. A) often proves to be cost-effective B) simplifies the process of developing a national strategy C) focuses on building market share rather than customer relationships D) reduces an organization's staffing requirements E) allows product managers to achieve functional expertise

C Page Ref: 628 Objective: 2 Difficulty: Moderate

38) Which of the following represents the objective of a cognitive social marketing campaign? A) Motivate people with obesity to eat healthy and exercise more often. B) Change public attitudes and stereotypes associated with people who are obese. C) Explain the different causes of obesity and how it can be prevented. D) Encourage people to participate in a walkathon aimed at promoting awareness about obesity. E) Help people with obesity to implement lifestyle changes.

C Page Ref: 640 Objective: 3 AACSB: Analytic skills Difficulty: Moderate

56) A ________ is a comprehensive, systematic, independent, and periodic examination of a company's or business unit's marketing environment, objectives, strategies, and activities, with a view to determining problem areas and opportunities and recommending a plan of action to improve the company's marketing performance. A) marketing plan B) test market C) marketing audit D) market-based scorecard analysis E) marketing metric

C Page Ref: 645 Objective: 5 Difficulty: Easy

74) The manufacturer launches a brand building advertising campaign. The campaign does not promote any one specific car but is aimed at promoting the company as a whole. Which type of cost does this fall under? A) direct costs B) material costs C) non-traceable costs D) traceable costs E) labor costs

C Page Ref: 659 Objective: APP AACSB: Analytic skills Difficulty: Moderate

Gabrielle is the chief marketing officer of Boyd Pharmaceuticals. She is meeting with Trent, the chief financial officer to decide on the company's marketing communications budget. They decide to trust in the prevailing collective wisdom of the industry as a whole, and not wanting to instigate a communications war, settle on spending only as much as their nearest market rival does on marketing communications. What method did Gabrielle and Trent use to arrive at the marketing communications budget? A) objective-and-task method B) affordable method C) competitive-parity method D) activity-based method E) percentage-of-sales method

C) Competitive-parity method

Which of the following elements of the marketing communication mix involves use of mail, telephone, fax, e-mail, or Internet to communicate with or solicit response or dialogue from specific customers and prospects? A) advertising B) personal selling C) direct marketing D) public relations E) sales promotion

C) Direct Marketing

________ was originally pioneered by Japanese firms such as Toyota to produce goods with minimal waste of time, materials, and money. A) Electronic funds transfer (EFT) B) Market logistics C) Electronic data interchange (EDI) D) Lean manufacturing E) Supply chain management

C) Electronic data interchange (EDI)

According to the hierarchy-of-effects model, which of the following corresponds to the behavioral stage that a buyer passes through? A) awareness B) knowledge C) purchase D) preference E) conviction

C) Purchase

All response hierarchy models of the communication process assume the buyer passes through cognitive, affective, and behavioral stages, in that order. Which of the following product categories lends itself most appropriately to such a "learn-feel-do" sequence? A) clothes B) dishwashers C) real estate D) personal computer E) air tickets

C) Real Estate

Which of the following personal communications channels consist of family members, neighbors, friends, and associates talking to target buyers? A) expert channels B) advocate channels C) social channels D) formal channels E) sponsored channels

C) Social Channels

________ sell and deliver a limited line of semiperishable goods to supermarkets, grocery stores, hospitals, restaurants, and hotels. A) Producers' cooperatives B) Cash and carry wholesalers C) Truck wholesalers D) Drop shippers E) Rack jobbers

C) Truck wholesalers

Which of the following sequences accurately represents the hierarchy-of-effects model of marketing communications? A) attention-interest-desire-action B) awareness-interest-evaluation-trial-adoption C) awareness-knowledge-liking-preference-conviction-purchase D) exposure-reception-cognitive response-attitude-intention-behavior E) knowledge-persuasion-decision-implementation-confirmation

C) awareness-knowledge-liking-preference-conviction-purchase

A(n) ________ is a storeless retailer serving a specific clientele — usually employees of large organizations — who are authorized to buy from a list of retailers that have agreed to give discounts in return for inclusion on the list. A) direct-selling vendor B) direct marketing vendor C) buying service D) automatic vendor E) corporate retailer

C) buying service

A ________ is a retail firm owned by its customers. Members contribute money to open their own store, vote on its policies, elect a group to manage it, and receive dividends. A) retailer cooperative B) voluntary chain C) consumer cooperative D) merchandising conglomerate E) franchise organization

C) consumer cooperative

Which of the following is a postpurchase service offered by retailers? A) accepting orders over the telephone B) advertising and window displays C) delivery to the customer's doorstep D) general information E) interior decoration of the retail outlet

C) delivery to the customer's doorstep

Avon, Tupperware, and Southwestern Company of Nashville are among companies in the multibillion-dollar ________ industry, which involves selling door-to-door or at home sales parties. A) direct marketing B) catalog showroom C) direct selling D) automatic vending E) buying services

C) direct selling

LCH is a leading electronics company that produces and markets its own brand of desktop and laptop computers for both individual consumers and businesses. Which of the following sequences of consumer responses is relevant as a marketing communications model for LCH's products? A) learn-do-feel B) feel-learn-do C) do-feel-learn D) feel-do-learn E) do-learn-feel

C) do-feel-learn

H&A is a retail chain that specializes in selling goods at very low prices. To achieve this, it stocks a very narrow assortment of basic necessities and offers customers a "no-frills" shopping experience. H&A is an example of a(n) ________ store. A) off-price B) specialty C) hard-discount D) superstore E) convenience

C) hard-discount

Which of the following circumstances are best suited for personal selling? A) when the products used are simple and easy-to-use B) when there is minimal risk involved in buying or using the products C) when the market has fewer and larger sellers D) when the products being marketed are inexpensive and easily available E) when prospective customers are spread across a wide geographic area

C) when the market has fewer and larger sellers

When planning communications for a detergent brand, which of the following sequences of buyer responses should the marketer choose on which to base the communications model? A) feel-do-learn B) do-feel-learn C) feel-learn-do D) learn-do-feel E) learn-feel-do

D) learn-do-feel

50) List the major types of limited-service wholesalers and explain how each type functions.

Cash and carry wholesalers sell a limited line of fast-moving goods to small retailers for cash. Truck wholesalers sell and deliver a limited line of semiperishable goods to supermarkets, grocery stores, hospitals, restaurants, and hotels. Drop shippers serve bulk industries such as coal, lumber, and heavy equipment. They assume title and risk from the time an order is accepted to its delivery. Rack jobbers serve grocery retailers in nonfood items. Delivery people set up displays, price goods, and keep inventory records; they retain title to goods and bill retailers only for goods sold to the end of the year. Producers' cooperatives assemble farm produce to sell in local markets. Mail-order wholesalers send catalogs to retail, industrial, and institutional customers; orders are filled and sent by mail, rail, plane, or truck.

A company gets the idea of producing a powder to add to milk to increase its nutritional value and taste and forms several concepts: an instant drink for adults who want a quick nutritious breakfast without preparation; a tasty snack for children to drink as a midday refreshment; and a health supplement for older adults to drink in the late evening before they go to bed. Each of these represents a ________.

Category Concept

133) When Cathy went shopping, she paid a lot to buy a jacket that had a well-known designer's tag attached to it. After a few days, she came across a jacket which was undistinguishable from the one she had bought but was priced 5 times lesser than the earlier one. She didn't give this a second thought because she was convinced that the designer label she had bought was worth it. What can be deduced from this?

Cathy was using the price as an indicator of quality. She was using image pricing. This kind of pricing is especially effective with ego-sensitive products such as perfumes, expensive cars, and designer clothing. Page Ref: 388 Objective: 1 AACSB: Analytic skills Difficulty: Moderate

91) Define cause-related marketing. What is the difference between cause-related marketing and social marketing?

Cause-related marketing links the firm's contributions to a designated cause to customers' engaging directly or indirectly in revenue-producing transactions with the firm. Cause-related marketing supports a cause whereas social marketing by nonprofit or government organizations furthers a cause.

94) What is channel power? Explain the various types of channel power.

Channel power is the ability to alter channel members' behavior so they take actions they would not have taken otherwise. • Coercive power: A manufacturer threatens to withdraw a resource or terminate a relationship if intermediaries fail to cooperate. This power can be effective, but its exercise produces resentment and can lead the intermediaries to organize countervailing power. • Reward power: The manufacturer offers intermediaries an extra benefit for performing specific acts or functions. Reward power typically produces better results than coercive power, but intermediaries may come to expect a reward every time the manufacturer wants a certain behavior to occur. • Legitimate power: The manufacturer requests a behavior that is warranted under the contract. As long as the intermediaries view the manufacturer as a legitimate leader, legitimate power works. • Expert power: The manufacturer has special knowledge of the intermediaries' value. Once the intermediaries acquire this expertise, however, expert power weakens. The manufacturer must continue to develop new expertise so intermediaries will want to continue cooperating. • Referent power: The manufacturer is so highly respected that intermediaries are proud to be associated with it. Companies such as IBM, Caterpillar, and Hewlett-Packard have high referent power. Coercive and reward power are objectively observable; legitimate, expert, and referent power are more subjective and depend on the ability and willingness of parties to recognize them.

70) Briefly explain the various service outputs that marketing channels produce.

Channels produce five service outputs: 1. Lot size: The number of units the channel permits a typical customer to purchase on one occasion. 2. Waiting and delivery time: The average time customers wait for receipt of goods. Customers increasingly prefer faster delivery channels. 3. Spatial convenience: The degree to which the marketing channel makes it easy for customers to purchase the product. 4. Product variety: The assortment provided by the marketing channel. Normally, customers prefer a greater assortment because more choices increase the chance of finding what they need, although too many choices can sometimes create a negative effect. 5. Service backup: Add-on services (credit, delivery, installation, repairs) provided by the channel. The greater the service backup, the greater the work provided by the channel.

14) Persuasive advertising aims to create brand awareness and knowledge of new products or new features of existing products.

FALSE

90) Describe the communication-effect research method of evaluating advertising effectiveness.

Communication-effect research, called copy testing, seeks to determine whether an ad is communicating effectively. Marketers should perform this test both before an ad is put into media and after it is printed or broadcast. Pretest critics maintain that agencies can design ads that test well but may not necessarily perform well in the marketplace. Proponents maintain that useful diagnostic information can emerge and that pretests should not be used as the sole decision criterion anyway. Many advertisers use posttests to assess the overall impact of a completed campaign. If a company hoped to increase brand awareness from 20 percent to 50 percent and succeeded in increasing it to only 30 percent, then the company is not spending enough, its ads are poor, or it has overlooked some other factor.

127) What are the different types of promotional pricing?

Companies can use any of the following pricing techniques to stimulate early purchase: • Loss-leader pricing - Supermarkets and department stores often drop the price on well-known brands to stimulate additional store traffic. This pays if the revenue on the additional sales compensates for the lower margins on the loss-leader items. • Special event pricing - Sellers can establish special prices in certain seasons to draw in more customers. • Special customer pricing - Sellers can offer special prices exclusively to certain customers. • Cash rebates - Auto companies and other consumer-goods companies offer cash rebates to encourage purchase of the manufacturers' products within a specified time period. Rebates can help clear inventories without cutting the stated list price. • Low-interest financing - Instead of cutting its price, the company can offer customers low-interest financing. • Longer payment terms - Sellers stretch loans over longer periods and thus lower the monthly payments. Consumers often worry less about the interest rate of a loan, and more about whether they can afford the monthly payment. • Warranties and service contracts - Companies can promote sales by adding a free or low-cost warranty or service contract. • Psychological discounting - This strategy sets an artificially high price and then offers the product at substantial savings. Page Ref: 405 Objective: 3 Difficulty: Moderate

119) How do companies organize new product development?

Companies handle the organizational aspect of new-product development in several ways. Many assign responsibility to product managers. Some employ new-product managers who report to category managers. A few companies have growth leaders, a full-time job for its most creative and successful managers. Some companies have a high-level management committee charged with reviewing and approving proposals. Large companies often establish a new-product department headed by a manager with substantial authority and access to top management whose responsibilities include generating and screening new ideas, working with the R&D department, and carrying out field testing and commercialization. Page Ref: 574 Objective: 2 Difficulty: Easy

119) Explain the product- or brand-management organization and list its advantages and disadvantages.

Companies producing a variety of products and brands often establish a product- (or brand-) management organization. The product- or brand-management organization does not replace the functional organization, but serves as another layer of management. A product-management organization makes sense if the company's products are quite different, or if the sheer number of products is beyond the ability of a functional organization to handle. Product and brand management is sometimes characterized as a hub-and-spoke system. The product-management organization has several advantages. The product manager can concentrate on developing a cost-effective marketing mix for the product and can react more quickly to new products in the marketplace; the company's smaller brands have a product advocate. The disadvantages are that product managers are not given enough authority, they become experts in their product area but rarely achieve functional expertise. The product-management system is costly and brand managers normally manage a brand only for a short time. The fragmentation of markets makes it harder to develop a national strategy. In addition to this, product and brand managers focus on market share and not in building customer relationships. Page Ref: 627-628 Objective: 2 Difficulty: Moderate

43) Explain the product- or brand-management organization and list its advantages and disadvantages.

Companies producing a variety of products and brands often establish a product- (or brand-) management organization. The product- or brand-management organization does not replace the functional organization, but serves as another layer of management. A product-management organization makes sense if the company's products are quite different, or if the sheer number of products is beyond the ability of a functional organization to handle. Product and brand management is sometimes characterized as a hub-and-spoke system. The product-management organization has several advantages. The product manager can concentrate on developing a cost-effective marketing mix for the product and can react more quickly to new products in the marketplace; the company's smaller brands have a product advocate. The disadvantages are that product managers are not given enough authority, they become experts in their product area but rarely achieve functional expertise. The product-management system is costly and brand managers normally manage a brand only for a short time. The fragmentation of markets makes it harder to develop a national strategy. In addition to this, product and brand managers focus on market share and not in building customer relationships.

9) Pricing cues such as sale signs and prices that end in 9 are more influential ________. A) when customers have substantial knowledge about prices B) when customers purchase the particular item regularly C) when product quality is standardized D) when product designs vary over time E) when prices do not vary from time to time

D Page Ref: 388 Objective: 1 Difficulty: Moderate

One of the weaknesses of using surveys to estimate the demand curve is _______.

Consumers exaggerate their willingness to pay for new products and services

145) Most companies today are trying to increase the order-to-payment cycle.

FALSE

140) Troma Inc. is a famous manufacturer of cookware that follows a traditional distributor-retailer system to distribute its products. The company abstains from the use of automated supply chain management (SCM) systems mainly due to the fear of unknown. However, rapidly escalating operational costs and inefficiencies have made it necessary for the company to implement an SCM system. The company goes for a big-bang installation of SCM system to become more competitive and cost effective. Identify the adopter group to which Troma belongs. A) innovator B) early adopter C) early majority D) laggard E) late majority

D Diff: 2 LO: 15.8: What factors affect the rate of diffusion and consumer adoption of newly launched products and services? AACSB: Analytical thinking; Application of knowledge

141) Jason is a technology enthusiast who is happy to conduct alpha and beta testing and report on early weaknesses of consumer electronics products. He is most likely to fall in the ________ adopter category. A) late majority B) early C) early majority D) innovator E) laggard

D Diff: 2 LO: 15.8: What factors affect the rate of diffusion and consumer adoption of newly launched products and services? AACSB: Application of knowledge

138) Which of the following is the mental step in the consumer-adoption process where the customer starts considering whether to try an innovation? A) adoption B) trial C) awareness D) evaluation E) interest

D Diff: 2 LO: 15.8: What factors affect the rate of diffusion and consumer adoption of newly launched products and services? AACSB: Reflective thinking

23) In ________, a salesperson goes to the home of a host who has invited friends, demonstrates the products, and takes orders. A) catalog marketing B) franchising C) direct-response selling D) network marketing E) direct marketing

D Diff: 2 LO: 18.1: What major types of marketing intermediaries occupy this sector? AACSB: Analytical thinking

6) A large staff, along with a higher proportion of specialty goods and slower-moving items and many services, are usually features of ________ retailing. A) self-service B) self-selection C) limited service D) full-service E) limited-selection

D Diff: 2 LO: 18.1: What major types of marketing intermediaries occupy this sector? AACSB: Analytical thinking

11) Which of the following is an example of a limited-service retailer? A) Customers at TAB pick out the products they want and pay at the checkout counter. B) Salespeople at Eli are always on hand to offer shoppers advice and assistance in choosing products. C) Mina's allows customers to checkout their own purchases in order to reduce the staff count. D) As CLO's offers a number of merchandise-return privileges, customers need information and assistance while shopping. E) Alison's stocks a number of specialty products and the store's salespeople generally help shoppers with all their needs.

D Diff: 2 LO: 18.1: What major types of marketing intermediaries occupy this sector? AACSB: Analytical thinking; Application of knowledge

3) Which of the following is an example of retailing? A) Dylan's sends catalogs to retail, industrial, and institutional customers. B) SEZ U Inc. sells a limited line of fast-moving goods to small retailers for cash. C) BEL Inc. sells FMCG goods to merchant wholesalers and distributors. D) Praxis International sells products to consumers directly through the Internet. E) Hub Styles procures its raw materials directly from farmers in the region.

D Diff: 2 LO: 18.1: What major types of marketing intermediaries occupy this sector? AACSB: Analytical thinking; Application of knowledge

30) Which of the following is true for franchisors? A) The franchisor has to pay the franchisee to be part of the franchise system. B) The franchisor licenses the trademark from the franchisee. C) The franchisor must change its operations to suit those of the franchisee. D) The franchisor collects royalty payments from the franchisee. E) The franchisor pays startup costs for the franchisee.

D Diff: 2 LO: 18.1: What major types of marketing intermediaries occupy this sector? AACSB: Reflective thinking

75) Mass merchandisers and discount stores typically fall into the ________ group with respect to margins and volume. A) mixed markup, high-volume B) low-volume, mixed markup C) low-volume, low-markup D) high-volume, low-markup E) high-markup, low-volume

D Diff: 2 LO: 18.3: What marketing decisions do marketing intermediaries make? AACSB: Analytical thinking

81) Which of the following is an ancillary service offered by retailers? A) accepting orders over the telephone B) advertising and window displays C) delivery to the customer's doorstep D) general information E) alterations and tailoring

D Diff: 2 LO: 18.3: What marketing decisions do marketing intermediaries make? AACSB: Reflective thinking

103) Which of the following is true for distributor brands? A) They sell at higher volumes than national brands and are also known as generics. B) They are usually sold at higher prices than national brands because production costs are higher. C) They are always of better quality than national brands as production is strictly monitored. D) Distributor brands can be sold at lower prices yet generate a higher profit margin because of their lower cost structure. E) Advertising and sales promotion costs for distributor brands are much higher than those for national brands.

D Diff: 2 LO: 18.4: What does the future hold for private label brands? AACSB: Reflective thinking

138) E&OE is trying to minimize its inventory costs, which are extremely high. The company has realized that it can achieve this by maintaining a near-zero inventory and producing more products only once it is ordered. Which of the following will be true for E&OE? A) Short production runs will be more expensive than longer ones. B) Setup and order-processing costs will be high. C) The order point will be high. D) Order-processing costs will be lower than the inventory-carrying costs. E) E&OE can reduce the average cost per unit by producing a long run.

D Diff: 2 LO: 18.6: What are some important issues in logistics? AACSB: Analytical thinking; Application of knowledge

5) A company decided to conduct a market survey for its new MP3 player which it had priced at $150. However, in the survey, 95 percent of the participants said that the maximum they would pay for the MP3 player is $100. This is an example of which of the following possible consumer reference prices? A) historical competitor price B) expected future price C) usual discounted price D) upper-bound price E) last price paid

D Page Ref: 387 Objective: 1 AACSB: Analytic skills Difficulty: Moderate

Mass merchandisers and discount stores typically fall into the ________ group with respect to margins and volume. A) mixed markup, high-volume B) low-volume, mixed markup C) low-volume, low-markup D) high-volume, low-markup E) high-markup, low-volume

D) high-volume, low-markup

34) Which of the following is true regarding cause-related marketing? A) The positive impact of cause-related marketing can be increased through sporadic involvement with numerous causes. B) Many companies focus on multiple causes to simplify execution and maximize impact. C) Limiting support to a single cause increases the pool of stakeholders who can transfer positive feelings from the cause to the firm. D) Most firms choose causes that fit their corporate or brand image and matter to their employees and shareholders. E) In order to avoid public backlash, firms are advised to adopt a hard-sell approach to their cause efforts.

D Page Ref: 637-638 Objective: 3 AACSB: Ethical understanding and reasoning abilities Difficulty: Moderate

37) A social marketing program which aims to alter ideas about abortion is an example of a(n) ________. A) cognitive campaign B) active campaign C) behavioral campaign D) value campaign E) normative campaign

D Page Ref: 640 Objective: 3 AACSB: Ethical understanding and reasoning abilities Difficulty: Moderate

44) ________ software provides a set of Web-based applications that automate and integrate project management, campaign management, budget management, asset management, brand management, customer relationship management, and knowledge management. A) Marketing dashboard B) Enterprise resource planning C) Supply chain management D) Marketing resource management E) Enterprise campaign management

D Page Ref: 642 Objective: 5 AACSB: Use of IT Difficulty: Easy

48) Who is primarily responsible for efficiency control? A) BAMT B) marketing auditor C) middle management D) line and staff management E) top management

D Page Ref: 643 Objective: 5 Difficulty: Easy

52) Marketing effectiveness rating instruments and marketing audits are approaches to ________. A) annual-plan control B) profitability control C) efficiency control D) strategic control E) statistical control

D Page Ref: 643 Objective: 5 Difficulty: Easy

51) The purpose of strategic control is to ________. A) examine whether the planned results are being achieved B) examine where the company is making and losing money C) evaluate and improve the spending efficiency and impact of marketing expenditures D) examine whether the company is pursuing its best opportunities with respect to markets, products, and channels E) understand the efficiency of the sales force, advertising, sales promotion, and distribution

D Page Ref: 643 Objective: 5 Difficulty: Moderate

54) Which of the following is an example of a distribution metric used for measuring the performance of marketing plans? A) effective reach B) customer acquisition costs C) market share D) stocks cover in days E) response rate

D Page Ref: 644 Objective: 5 Difficulty: Moderate

67) A(n) ________ of exactly 100 percent means that a company is tied for the market lead. A rise in relative market share means the company is gaining on its leading competitor. A) overall market share B) served market share C) potential market share D) relative market share E) actual market share

D Page Ref: 654 Objective: APP Difficulty: Moderate

Cabot Cars (Scenario) Cabot, a large car manufacturer, has four popular car brands in different segments. It has a manufacturing facility near Detroit, MI, where parts common to all the four brands are manufactured. Other specific parts like the brake system, windshield, bonnet, locks, and so on are manufactured in separate plants. Each make has its own product manager and support staff. All product managers report to the CEO of the company. 70) The CEO's annual compensation is an example of a ________. A) direct cost B) variable cost C) traceable common cost D) non-traceable common cost E) manufacturing cost

D Page Ref: 659 Objective: APP AACSB: Analytic skills Difficulty: Moderate

Which of the following statements is true of the two-step approach to mass communications? A) The influence of mass media on public opinion is more direct, powerful, and automatic than marketers have supposed. B) Communications through mass media bypasses opinion leaders and reaches the individual buyers. C) The two-step flow supports the notion that consumption styles are primarily influenced by a "trickle-down" or "trickle-up" effect from mass media. D) According to the two-step flow, people interact primarily within their own social groups and acquire ideas from opinion leaders in their groups. E) Two-step communication suggests that mass communicators should direct messages to groups of buyers who interpret the message and act accordingly.

D) According to the two-step flow, people interact primarily within their own social groups and acquire ideas from opinion leaders in their groups.

Which of the following is an example of a limited-service retailer? A) Customers at TAB pick out the products they want and pay at the checkout counter. B) Salespeople at Eli are always on hand to offer shoppers advice and assistance in choosing products. C) Mina's allows customers to checkout their own purchases in order to reduce the staff count. D) As CLO's offers a number of merchandise-return privileges, customers need information and assistance while shopping. E) Alison's stocks a number of specialty products and the store's salespeople generally help shoppers with all their needs.

D) As CLO's offers a number of merchandise-return privileges, customers need information and assistance while shopping.

Which of the following steps in the innovation-adoption model of marketing communications corresponds to the cognitive stage that a buyer passes through? A) interest B) evaluation C) trial D) awareness E) adoption

D) Awareness

Which method for establishing the total marketing communications budget sets communication budgets to achieve the same amount of share-of-voice as competitors? A) comparative-parity method B) objective-and-task method C) affordable method D) competitive-parity method E) percentage-of-sales method

D) Competitive-parity method

Which of the following is a disadvantage of using a percentage-of-sales method to determine the marketing communications budget? A) It discourages stability when competing firms spend approximately the same percentage of their sales on communications. B) By using a percentage-of-sales method, communication expenditures tend to be extremely high irrespective of what a company can afford. C) It discourages management from thinking of the relationship among communication cost, selling price, and profit per unit. D) Dependence of the percentage-of-sales method on year-to-year sales fluctuations interferes with long-range planning. E) The percentage-of-sales method views sales as the result in itself rather than the determiner of communications.

D) Dependence of the percentage-of-sales method on year-to-year sales fluctuations interferes with long-range planning.

Which of the following is true for distributor brands? A) They sell at higher volumes than national brands and are also known as generics. B) They are usually sold at higher prices than national brands because production costs are higher. C) They are always of better quality than national brands as production is strictly monitored. D) Distributor brands can be sold at lower prices yet generate a higher profit margin because of their lower cost structure. E) Advertising and sales promotion costs for distributor brands are much higher than those for national brands.

D) Distributor brands can be sold at lower prices yet generate a higher profit margin because of their lower cost structure.

________ serve bulk industries such as coal, lumber, and heavy equipment, assuming title and risk from the time an order is accepted to its delivery. A) Producers' cooperatives B) Cash and carry wholesalers C) Truck wholesalers D) Drop shippers E) Rack jobbers

D) Drop shippers

Along which of the following parameters should marketers evaluate communication options when building brand equity? A) popularity B) innovativeness C) technological sophistication D) efficiency E) novelty

D) Efficiency

When Apple introduced the iPod in October, 2001, it was the first-of-its-kind product that offered sizable storage capacity for songs and a portable device that was not seen before in the market. Which of the following is most likely to have been the marketing communications objective for the iPod at the time of its introduction? A) developing brand awareness B) building customer traffic C) enhancing purchase actions D) establishing product category E) enhancing firm image

D) Establishing Product Category

E&OE is trying to minimize its inventory costs, which are extremely high. The company has realized that it can achieve this by maintaining a near-zero inventory and producing more products only once it is ordered. Which of the following will be true for E&OE? A) Short production runs will be more expensive than longer ones. B) Setup and order-processing costs will be high. C) The order point will be high. D) Order-processing costs will be lower than the inventory-carrying costs. E) E&OE can reduce the average cost per unit by producing a long run.

D) Order-processing costs will be lower than the inventory-carrying costs.

Which of the following marketing communications tools is most effective at influencing customers at the conviction stage of buyer readiness? A) advertising B) publicity C) sales promotion D) personal selling E) events and experiences

D) Personal Selling

Which of the following is an example of retailing? A) Dylan's sends catalogs to retail, industrial, and institutional customers. B) SEZ U Inc. sells a limited line of fast-moving goods to small retailers for cash. C) BEL Inc. sells FMCG goods to merchant wholesalers and distributors. D) Praxis International sells products to consumers directly through the Internet. E) Hub Styles procures its raw materials directly from farmers in the region.

D) Praxis International sells products to consumers directly through the Internet.

Which of the following marketing communications tools is most influential at the reordering stage of buyer readiness? A) events and experiences B) publicity C) direct marketing D) sales promotion E) interactive marketing

D) Sales Promotion

Luke is considering the various options available to him to promote an energy drink, Turbozade, that has decreasing sales volumes after having peaked some time back. Which of the following marketing communications tools should Luke focus marketing efforts on to keep the sales volume up? A) advertising B) direct marketing C) events and experiences D) sales promotions E) publicity

D) Sales Promotions

Which of the following benefits is offered by sales promotion tools? A) Sales promotion tools are more authentic and credible to buyers than others such as advertising, public relations, and personal selling. B) Sales promotion tools can reach prospects who prefer to avoid mass media and targeted promotions. C) Sales promotion tools are typically an indirect form of "soft-sell" and hence, better received by customers. D) Sales promotion tools incorporate some concession, inducement, or contribution that gives value to the consumer. E) Sales promotion tools allow buyers personal choices and encourage them to respond directly.

D) Sales promotion tools incorporate some concession, inducement, or contribution that gives value to the consumer.

Which of the following statements is true of the role of advertising in business markets? A) Advertising is unsuitable for explaining any new features that a product might have. B) Sales calls are more economical than reminder advertisements. C) Sales calls are more effective than advertisements at reminding customers how to use a product and reassure them about their purchase. D) Sales representatives can use copies of the company's ads to legitimize their company and products. E) Advertisements are the least preferred tools when intended to generate leads for sales representatives.

D) Sales representatives can use copies of the company's ads to legitimize their company and products.

Which of the following is an example of an interactive marketing communication platform? A) product demonstrations B) factory tours C) company museums D) TV shopping E) community relations

D) TV Shopping

Which of the following is true for franchisors? A) The franchisor has to pay the franchisee to be part of the franchise system. B) The franchisor licenses the trademark from the franchisee. C) The franchisor must change its operations to suit those of the franchisee. D) The franchisor collects royalty payments from the franchisee. E) The franchisor pays startup costs for the franchisee.

D) The franchisor collects royalty payments from the franchisee.

________ is a source of a spokesperson's credibility that describes how objective and honest the spokesperson is perceived to be. A) Likability B) Expertise C) Experience D) Trustworthiness E) Compassion

D) Trustworthiness

Which of the following is a characteristic of the affordable method of establishing a marketing communications budget? A) fixed annual budget B) suitable for long-range planning C) priority given to role of promotion as an investment D) calculated to reflect what the company can spare for marketing communications E) based on the immediate impact of promotion on sales volume

D) calculated to reflect what the company can spare for marketing communications

A large staff, along with a higher proportion of specialty goods and slower-moving items and many services, are usually features of ________ retailing. A) self-service B) self-selection C) limited service D) full-service E) limited-selection

D) full-service

150) What are direct, traceable common, and nontraceable common costs? Give an example of each.

Direct costs are costs that we can assign directly to the proper marketing entities, such as materials costs and sales-force salaries. Traceable common costs are incurred indirectly but can be attributed on a plausible basis to various marketing entities, such as rent expenses. Nontraceable common costs are costs whose assignments are highly arbitrary, such as "corporate image" expenditures.

146) More stocking locations mean goods can be delivered to customers more quickly, and warehousing and inventory costs are lower.

FALSE

98) A large FMCG company decides to test market Kora, a new brand of face cleanser, to be launched soon. The company initially distributes a few free samples to some prospective consumers. Later it offers the product to the customers at a discounted price and observes that not only more than seventy percent of the customers are purchasing it but the same number are also satisfied using it. The company keeps using this process 3-4 times to obtain a correct count of the number of people purchasing the product repeatedly. Which of the following testing methods is being used here? A) simulated testing B) controlled testing C) full test marketing D) parallel testing E) sales-wave research

E Diff: 2 LO: 15.6: What is the best way to manage concept and strategy development? AACSB: Analytical thinking; Application of knowledge

82) ________ means presenting the product concept to target consumers, symbolically or physically, and getting their reactions. A) Perceptual mapping B) Brand-positioning mapping C) Brand attribute mapping D) Concept development E) Concept testing

E Diff: 2 LO: 15.6: What is the best way to manage concept and strategy development? AACSB: Reflective thinking

90) ________ exhibit replacement cycles dictated by physical wear or obsolescence associated with changing style, features, and performance. A) Frequently purchased products B) High-moving goods C) Inexpensive products D) Commodity products E) Infrequently purchased products

E Diff: 2 LO: 15.6: What is the best way to manage concept and strategy development? AACSB: Reflective thinking

143) Joseph, a student of Columbia University, finds many of his classmates have purchased an iPad tablet from Apple. The iPad, launched a few months before has been identified as a very useful product and many students in the US have rated it highly. Considering all these, Joseph also decides to purchase an iPad. Which of the following is the adopter group to which Joseph belongs? A) early adopter B) innovator C) late majority D) laggard E) early majority

E Diff: 2 LO: 15.8: What factors affect the rate of diffusion and consumer adoption of newly launched products and services? AACSB: Analytical thinking; Application of knowledge

14) An example of a restaurant with a narrow and deep assortment is a ________. A) small lunch counter B) cafeteria C) large restaurant D) casual dining restaurant chain E) delicatessen

E Diff: 2 LO: 18.1: What major types of marketing intermediaries occupy this sector? AACSB: Application of knowledge

59) Electronic shelf labeling allows retailers to ________. A) check inventory levels instantaneously B) order electronically from suppliers C) run continual promotional messages D) advertise sales and special offers E) change price levels instantaneously

E Diff: 2 LO: 18.2: What major changes are occurring in the modern retail marketing environment with respect to competitive market structure and technology? AACSB: Information technology

76) A store selling expensive artwork and luxury goods typically falls into the ________ group with respect to margins and volume. A) mixed markup, high-volume B) low-volume, mixed markup C) low-volume, low-markup D) high-volume, low-markup E) high-markup, low-volume

E Diff: 2 LO: 18.3: What marketing decisions do marketing intermediaries make? AACSB: Analytical thinking; Application of knowledge

73) In supermarkets and other retail outlets, RFID is used to ________. A) change prices instantaneously B) check for spoilage or damage to goods C) advertise special offers and discounts D) run continual promotional messages E) monitor inventory and track goods

E Diff: 2 LO: 18.3: What marketing decisions do marketing intermediaries make? AACSB: Information technology

101) A brand developed by a retailer and/or wholesaler that is available only in selected retail outlets is called a ________ brand. A) generic B) franchisee C) marque D) national E) private-label

E Diff: 2 LO: 18.4: What does the future hold for private label brands? AACSB: Analytical thinking

102) Though it is sold only in Walmart stores, Walmart's Ol'Roy dog food has surpassed Nestlé's Purina brand as the top-selling dog food. Ol'Roy is an example of a ________. A) generic product B) national brand C) franchise D) copy-cat brand E) private label

E Diff: 2 LO: 18.4: What does the future hold for private label brands? AACSB: Analytical thinking; Application of knowledge

117) The owner of supermarket chain Reynold's has realized that customers want a wider variety of goods than is currently available. However, Reynold's cannot afford the costs of storing excess inventory. Additionally, the owner is not willing to take the risk that the new products will remain unsold. Which of the following types of wholesalers can help Reynold's meet customer demand while minimizing costs? A) producers' cooperatives B) cash and carry wholesalers C) truck wholesalers D) drop shippers E) rack jobbers

E Diff: 2 LO: 18.5: What are some of the important issues in wholesaling? AACSB: Analytical thinking; Application of knowledge

131) The elapsed time between an order's receipt, delivery, and payment is called the ________ cycle. A) variable-costs-to-payment B) product-to-payment C) inventory-to-sale D) order-to-inventory E) order-to-payment

E Diff: 2 LO: 18.6: What are some important issues in logistics? AACSB: Analytical thinking

142) Which of the following is an example of a private carrier? A) MET is a transporter that operates only in the Chicago area and charges fixed prices. B) BCL is a family firm that owns only three trucks, but takes small orders for transport. C) VTV owns a fleet of trucks and transports goods for any client for a fee. D) COM is a shipping firm that transports goods by road and rail across the United States. E) BEL Inc. manufactures parts for automobiles and transports its products to customers itself.

E Diff: 2 LO: 18.6: What are some important issues in logistics? AACSB: Analytical thinking; Application of knowledge

70) The price of tickets to the opera vary depending on where the person would like to be seated-in the gallery or in the stalls. This is an example of ________. A) channel pricing B) time pricing C) image pricing D) product-form pricing E) location pricing

E Page Ref: 406 Objective: 3 AACSB: Analytic skills Difficulty: Moderate

75) A low price buys market share but not market loyalty. The same customers will shift to any lower-priced product that may come along. This is called the ________. A) low-price trap B) relative-market-share trap C) shallow-pockets trap D) target-market-share trap E) fragile-market-share trap

E Page Ref: 408 Objective: 4 Difficulty: Easy

156) Exclusive dealings and tying agreements are illegal and prohibited by law.

FALSE

128) The two-step flow supports the notion that consumption styles are primarily influenced by a "trickle-down" or "trickle-up" effect from mass media.

FALSE

154) The coming years will see the rise of the marketing department and the demise of holistic marketing.

FALSE

44) Gap and Pottery Barn are examples of franchise organizations, where two or more retail outlets are owned and controlled by a corporation that employs central buying and merchandising and sells similar lines of merchandise.

FALSE

63) STIHL, which manufactures handheld outdoor power equipment sold to six independent US distributors and six STIHL-owned marketing and distribution centers who sell to a nationwide network of more than 8,000 servicing retail detailers, is a good example of intensive distribution.

FALSE

69) Manufacturers can obtain greater control over the selling process by using a sales agency.

FALSE

84) The macroscheduling decision calls for allocating advertising expenditures within a short period to obtain maximum impact.

FALSE

90) Mass merchandisers are usually high-markup, lower-volume stores.

FALSE

107) Private label or store brands are also known as generics.

FALSE

116) Adding more marketing channels increases the channel cost of selling.

FALSE

117) Fear appeals work best when they are rather strong and reinforce what the audience already believes.

FALSE

119) Loyal brand buyers tend to change their buying patterns as a result of competitive promotions.

FALSE

121) Examples of manufacturer promotions include price cuts and feature advertising.

FALSE

121) Trustworthiness is that source of a spokesperson's credibility that describes his or her attractiveness.

FALSE

122) Wholesalers exclude manufacturers and farmers because they are engaged primarily in production, but include retailers, as they are selling to the end consumer.

FALSE

125) A price-off pack is an example of a consumer franchise building sales promotions.

FALSE

130) Profitability control is the prime responsibility of line and staff management.

FALSE

136) The supply-side method identifies the effect sponsorship has on consumers' brand knowledge.

FALSE

137) The redemption rate for paper coupons (10 percent) far exceeds that of mobile coupons (1 percent).

FALSE

14) A pull strategy is particularly appropriate when there is low brand loyalty in a category, brand choice is made in the store, the product is an impulse item, and product benefits are well understood.

FALSE

49) In what ways does franchising benefit the franchisor? In what ways does it benefit the franchisee?

Franchising benefits both the franchisor and the franchisee. Franchisors gain the motivation and hard work of employees who are entrepreneurs rather than "hired hands," the franchisees' familiarity with local communities and conditions, and the enormous purchasing power of being a franchisor. Franchisees benefit from buying into a business with a well-known and accepted brand name. They find it easier to borrow money for their business from financial institutions, and they receive support in areas ranging from marketing and advertising to site selection and staffing.

152) Full costing allocates nontraceable common costs to marketing entities and has three major weaknesses. What are they?

Full costing has three major weaknesses: 1. the relative profitability of different marketing entities can shift radically when we replace one arbitrary way to allocate nontraceable common costs by another 2. the arbitrariness demoralizes managers, who feel their performance is judged adversely 3. the inclusion of nontraceable common costs could weaken efforts at real cost control

146) Full costing allocates nontraceable common costs to marketing entities and has three major weaknesses. What are they?

Full costing has three major weaknesses: (1) The relative profitability of different marketing entities can shift radically when we replace one arbitrary way to allocate nontraceable common costs by another; (2) the arbitrariness demoralizes managers, who feel their performance is judged adversely; and (3) the inclusion of nontraceable common costs could weaken efforts at real cost control. Page Ref: 659 Objective: APP Difficulty: Moderate

145) GameTech International Inc. engages in the design, development, and marketing of interactive electronic bingo systems primarily in the United States. The company has plans to enter developing markets such as Brazil and India. TechToys, GameTech's major competitor has already started operations in India. GameTech decides to delay its entry to India and to closely observe Techtoy's operations in India. What is the possible rationale behind this delaying decision?

GameTech is using the late entry strategy. The firm will delay its launch until after the competitor has borne the cost of educating the market. It can also avoid the mistakes that TechToys make. The company can also learn the size of the market. Page Ref: 590 Objective: 4 AACSB: Analytic skills Difficulty: Moderate

133) Explain the concept of greenwashing along with an example.

Greenwashing gives products the appearance of being environmentally friendly without living up to that promise. An automobile manufacturer who promotes its cars as being environmentally friendly when the company is in fact a major polluter, would be an example of greenwashing. Page Ref: 635 Objective: 3 AACSB: Analytic skills Difficulty: Moderate

144) Mobile phone company HCF delayed the introduction of a new cellphone model with innovative features so that the launch could coincide with that of its competitor, LCM, which was introducing a phone with similar features. What kind of entry did HCF use? How can this benefit the company?

HCF used a parallel entry strategy. The market may pay more attention when two companies are advertising the new product. Page Ref: 590 Objective: 4 AACSB: Analytic skills Difficulty: Moderate

97) Atronix Solutions is a major manufacturer of electronic measuring devices in the United States. The company often threatens to terminate its relationship with its intermediaries if they do not agree with its terms and conditions. What channel power does Atronix use? What are its effects?

Here, the manufacturer is using coercive power. This power can be effective, but its exercise produces resentment and can lead the intermediaries to organize countervailing power.

124) Why do retailers prefer to deal with wholesalers rather than directly with manufacturers?

In general, retailers prefer to deal with wholesalers rather than directly with manufacturers (and vice versa) as wholesalers are better at performing the following functions: • Selling and promoting: Wholesalers' sales forces help manufacturers reach many small business customers at a relatively low cost. They have more contacts, and buyers often trust them more than they trust a distant manufacturer. • Buying and assortment building: Wholesalers are able to select items and build the assortments their customers need, saving them considerable work. • Bulk breaking: Wholesalers achieve savings for their customers by buying large carload lots and breaking the bulk into smaller units. • Warehousing: Wholesalers hold inventories, thereby reducing inventory costs and risks to suppliers and customers. • Transportation: Wholesalers can often provide quicker delivery to buyers because they are closer to the buyers. • Financing: Wholesalers finance customers by granting credit, and finance suppliers by ordering early and paying bills on time. • Risk bearing: Wholesalers absorb some risk by taking title and bearing the cost of theft, damage, spoilage, and obsolescence. • Market information: Wholesalers supply information to suppliers and customers regarding competitors' activities, new products, price developments, and so on. • Management services and counseling: Wholesalers often help retailers improve their operations by training sales clerks, helping with store layouts and displays, and setting up accounting and inventory-control systems. They may help industrial customers by offering training and technical services.

128) What is third-degree price discrimination?

In third-degree price discrimination, the seller charges different amounts to different classes of buyers, as in the following cases: • Customer-segment pricing - Different customer groups pay different prices for the same product or service. For example, museums often charge a lower admission fee to students and senior citizens. • Product-form pricing - Different versions of the product are priced differently, but not proportionately to their costs. • Image pricing - Some companies price the same product at two different levels based on image differences. A perfume manufacturer can put the perfume in one bottle, give it a name and image, and price it at $10 an ounce. The same perfume in another bottle with a different name and image and price can sell for $30 an ounce. • Channel pricing - Coca-Cola carries a different price depending on whether the consumer purchases it in a fine restaurant, a fast-food restaurant, or a vending machine. • Location pricing - The same product is priced differently at different locations even though the cost of offering it at each location is the same. A theater varies its seat prices according to audience preferences for different locations. • Time pricing - Prices are varied by season, day, or hour. Public utilities vary energy rates to commercial users by time of day and weekend versus weekday. Page Ref: 406 Objective: 3 Difficulty: Moderate

91) Describe the methods used to measure sponsorship activities.

It's a challenge to measure the success of events. The supply-side measurement method focuses on potential exposure to the brand by assessing the extent of media coverage, and the demand-side method focuses on exposure reported by consumers. Supply-side methods approximate the amount of time or space devoted to media coverage of an event, for example, the number of seconds the brand is clearly visible on a television screen or the column inches of press clippings that mention it. These potential "impressions" translate into a value equivalent to the dollar cost of actually advertising in the particular media vehicle. Although supply-side exposure methods provide quantifiable measures, equating media coverage with advertising exposure ignores the content of the respective communications. The advertiser uses media space and time to communicate a strategically designed message. Media coverage and telecasts only expose the brand and don't necessarily embellish its meaning in any direct way. Although some public relations professionals maintain that positive editorial coverage can be worth 5 to 10 times the equivalent advertising value, sponsorship rarely provides such favorable treatment. The demand-side method identifies the effect sponsorship has on consumers' brand knowledge. Marketers can survey event spectators to measure recall of the event as well as resulting attitudes and intentions toward the sponsor.

120) What is a market-management organization?

Many companies sell their products to differing markets. When customers fall into different user groups with distinct buying preferences and practices, a market-management organization is desirable. Market managers supervise several market-development managers, market specialists, or industry specialists and draw on functional services as needed. Market managers of important markets might even have functional specialists reporting to them. Market managers are staff (not line) people, with duties like those of product managers. They develop long-range and annual plans for their markets and are judged by their market's growth and profitability. Because this system organizes marketing activity to meet the needs of distinct customer groups, it shares many advantages and disadvantages of product-management systems. Page Ref: 629 Objective: 2 Difficulty: Moderate

153) Describe the four steps in market logistics planning.

Market logistics includes planning the infrastructure to meet demand, then implementing and controlling the physical flows of materials and final goods from points of origin to points of use, to meet customer requirements at a profit. Market logistics planning has four steps: 1. Deciding on the company's value proposition to its customers. (What on-time delivery standard should we offer? What levels should we attain in ordering and billing accuracy?). 2. Selecting the best channel design and network strategy for reaching the customers. (Should the company serve customers directly or through intermediaries? What products should we source from which manufacturing facilities? How many warehouses should we maintain and where should we locate them?). 3. Developing operational excellence in sales forecasting, warehouse management, transportation management, and materials management. 4. Implementing the solution with the best information systems, equipment, policies, and procedures.

88) What are the adjustments that marketers need to apply to the cost-per thousand measure when selecting specific media vehicles?

Marketers need to apply several adjustments to the cost-per-thousand measure. First, they should adjust for audience quality. For a baby lotion ad, a magazine read by 1 million young mothers has an exposure value of 1 million; if read by 1 million teenagers, it has an exposure value of almost zero. Second, adjust the exposure value for the audience-attention probability. Readers of Vogue may pay more attention to ads than do readers of Newsweek. Third, adjust for the medium's editorial quality (prestige and believability). People are more likely to believe a TV or radio ad and to become more positively disposed toward the brand when the ad is placed within a program they like. Fourth, consider ad placement policies and extra services (such as regional or occupational editions and lead-time requirements for magazines).

138) What is marketing control? List the four types of marketing control.

Marketing control is the process by which firms assess the effects of their marketing activities and programs and make necessary changes and adjustments. The four types of needed marketing control are: annual-plan control, profitability control, efficiency control, and strategic control. Page Ref: 643 Objective: 5 Difficulty: Easy

137) Define marketing implementation.

Marketing implementation is the process that turns marketing plans into action assignments and ensures that such assignments are executed in a manner that accomplishes the plan's stated objectives. Page Ref: 642 Objective: 3 Difficulty: Easy

158) What skills and competencies will marketers need to succeed in the future?

Marketing must become more holistic and less departmental. Marketers must achieve wider influence in the company, continuously create new ideas, and strive for customer insight by treating customers differently but appropriately. They must build their brands more through performance than promotion. They must go electronic and win through building superior information and communication systems. To accomplish these changes and become truly holistic, marketers need a new set of skills and competencies in: • Customer relationship management (CRM) • Partner relationship management (PRM) • Database marketing and data mining • Contact center management and telemarketing • Digital marketing and social media • Public relations marketing (including event and sponsorship marketing) • Brand-building and brand-asset management • Experiential marketing • Integrated marketing communications • Profitability analysis by segment, customer, and channel

151) How is coordination of media achieved through integrated marketing communications?

Media coordination can occur across and within media types, but marketers should combine personal and nonpersonal communications channels through multiple-vehicle, multiple-stage campaigns to achieve maximum impact and increase message reach and impact. Promotions can be more effective when combined with advertising, for example. The awareness and attitudes created by advertising campaigns can increase the success of more direct sales pitches. Advertising can convey the positioning of a brand and benefit from online display advertising or search engine marketing that offers a stronger call to action. Many companies are coordinating their online and offline communications activities. Web addresses in ads (especially print ads) and on packages allow people to more fully explore a company's products, find store locations, and get more product or service information. Even if consumers don't order online, marketers can use Web sites in ways that drive them into stores to buy.

When PepsiCo sold its cola syrup to Russia for rubles and agreed to buy Russian vodka at a certain rate for sale in the United States, it was engaged in the form of counter trade known as a(n) _________.

Offset

Sales rise at the beginning, peak, and then approach zero for _______.

One-Time Product Purchases

27) What is the role of print media in advertising? What are the major advantages and disadvantages associated with print advertising media?

Print media offer a stark contrast to broadcast media. Because readers consume them at their own pace, magazines and newspapers can provide detailed product information and effectively communicate user and usage imagery. At the same time, the static nature of the visual images in print media makes dynamic presentations or demonstrations difficult, and print media can be fairly passive. The two main print media — magazines and newspapers — share many advantages and disadvantages. Although newspapers are timely and pervasive, magazines are typically more effective at building user and usage imagery. Newspapers are popular for local — especially retailer — advertising. Although advertisers have some flexibility in designing and placing newspaper ads, relatively poor reproduction quality and short shelf life can diminish the ads' impact.

47) List and explain the four service levels offered by retailers.

Retailers meet widely different consumer preferences for service levels and specific services. The four levels of service usually offered by retailers are: 1. Self-service: Self-service is the cornerstone of all discount operations. Many customers are willing to carry out their own "locate-compare-select" process to save money. 2. Self-selection: Customers find their own goods, although they can ask for assistance. 3. Limited service: These retailers carry more shopping goods and services such as credit and merchandise-return privileges. Customers need more information and assistance. 4. Full-service: Salespeople are ready to assist in every phase of the "locate-compare-select" process. Customers who like to be waited on prefer this type of store. The high staffing cost, along with the higher proportion of specialty goods and slower-moving items and the many services, result in high-cost retailing.

154) Multichannel conflicts are common when the members of one channel obtain a lower price based on larger-volume purchases.

TRUE

129) How can companies initiate price cuts and what are the traps that companies can fall into because of this?

Several circumstances prompt a firm to cut its prices. One is excess plant capacity: The firm needs additional business and cannot generate it through increased sales effort, product improvement, or other measures. Companies sometimes initiate price cuts in a drive to dominate the market through lower costs. Either the company starts with lower costs than its competitors, or it initiates price cuts in the hope of gaining market share and lower costs. Cutting prices to keep customers or beat competitors often encourages customers to demand price concessions. Other than this, a price-cutting strategy can also lead to the following possible traps: • Low-quality trap - Consumers assume quality is low. • Fragile-market-share trap - A low price buys market share but not market loyalty. The same customers will shift to any lower-priced firm that comes along. • Shallow-pockets trap - Higher-priced competitors match the lower prices but have longer staying power because of deeper cash reserves. • Price-war trap - Competitors respond by lowering their prices even more, triggering a price war. Customers often question the motivation behind price changes. They may assume the item is about to be replaced by a new model; the item is faulty and is not selling well; the firm is in financial trouble; the price will come down even further; or the quality has been reduced. The firm must monitor these attributions carefully while initiating a price cut. Page Ref: 408 Objective: 4 Difficulty: Moderate

129) What is incremental innovation? Give an example of incremental innovation.

Student answers may vary. Incremental innovation refers to entering new markets by tweaking products for new customers, using variations on a core product to stay one step ahead of the market, and creating interim solutions for industry-wide problems. When Scott Paper couldn't compete with Fort Howard Paper Co. on price for the lucrative institutional toilet tissue market, it borrowed a solution from European companies: a dispenser that held bigger rolls. Scott made the larger rolls of paper and provided institutional customers with free dispensers, later doing the same thing with paper towels. Scott not only won over customers in a new market; it became less vulnerable to competitors, such as Fort Howard, which could lower prices but weren't offering the larger rolls or tailor-made dispensers. Page Ref: 571 Objective: 1 AACSB: Analytic skills Difficulty: Moderate

138) Use reverse assumption analysis to analyze a school.

Student answers may vary. Instead of assuming that a school is a place where teachers teach according to a set syllabus and timetable, you could assume that a school is a place where students choose what they want to learn and how they want to learn it. Students could choose people to teach them, depending on what they want to study. Page Ref: 579 Objective: 3 Difficulty: Easy

130) Tosho Electronics, a leading Japanese, electronic manufacturer believes shorter product life cycles of new products reduces the scope for R&D and innovation. Offer a possible reason for shorter lifecycles.

Student answers may vary. Rivals are quick to copy success of an innovation. An innovative product is copied even before the company recovers the cost of R&D. This leads to shorter product life cycles for products. Page Ref: 572 Objective: 1 AACSB: Analytic skills Difficulty: Moderate

56) Mal's father and grandfather ran Reynold's, a general store in the town of Bayswater. When Mal inherited the store, the town was expanding rapidly and a number of multinational franchisors showed interest in entering the town. Mal wants to turn Reynold's into a franchise of Blue Sun, a fast-food chain. What benefits can Mal gain from this move?

Student answers will vary. As the franchisor's brand is usually well-known, Mal obtains the benefit of a brand name that has already been accepted by customers. He could find it easier to get loans and financial help from banks on the basis of the well-known brand name. As the franchisor will provide everything from operational processes to marketing assistance, Mal need not spend time and effort beginning the process from scratch.

96) Skincare company E&OE retails its products through standalone stores or through its own stores within malls. The management uses this method so that customers are sure of finding only E&OE products at the stores and the brand image remains strong. In this scenario, what can E&OE do to generate consumer interest while retaining its exclusivity?

Student answers will vary. E&OE can do any or all of the following: • Feature blockbuster distinctive merchandise events. • Feature surprise or ever-changing merchandise. • Feature the latest or newest merchandise first. • Offer merchandise-customizing services. • Offer a highly targeted assortment.

97) Over the past three years, skincare products retailer E&OE has realized that it is losing sales to competitors who sell products online. E&OE has always cultivated an exclusive upscale image, and the management feels that shifting to the online route will only harm the brand image and sales in the long run. How can E&OE encourage customers to frequent its stores?

Student answers will vary. E&OE can offer more in-store services for its customers. It can offer information about the products, consultation and advice about the kinds of products to choose and allow customers to test or experience the products themselves. The retailer can also revamp the store atmosphere to be more inviting to customers, using music and fragrances and store design to create a pleasurable experience. E&OE can also increase the number of activities and experiences that customers can participate in.

133) Marco is working on promoting his company's Glazer brand of electronic razors. Market research suggests that the target audience possesses an intent to use Glazers, but is dithering over actually making the purchase. How can he modify the communications program to get customers to purchase Glazer razors?

Student answers will vary. Marco's task, once the intent to make a purchase has been fostered in consumers, is to lead them to the actual buying action. This can be achieved by offering the razors at a low introductory purchase to encourage initial use among customers, offering a discount on the price, or holding a promotional event where they can actually try out the product firsthand.

57) Mal's father and grandfather ran Reynold's, a general store in the town of Bayswater. When Mal inherited the store, the town was expanding rapidly and a number of multinational franchisors showed interest in entering the town. Mal wants to turn Reynold's into a franchise of Blue Sun, a fast-food chain. Why shouldn't Mal go the franchise route?

Student answers will vary. Most franchises offer franchisees limits or negligible independence in the questions of staffing, pricing, store decor, sourcing and processes. Mal may have to make changes that he does not agree with. He may not have the freedom to run the business as he thinks best.

138) An ad agency has landed an account for Savola, a brand of healthy cooking oil. Describe how the ad can effectively incorporate negative appeals in its messages.

Student answers will vary. Negative appeals such as fear, guilt, and shame are used by communicators to get people to do things (brush their teeth) or stop doing things (smoking). The cooking oil ad can use fear appeals to get consumers to start using its product. This can be achieved by depicting health risks associated with using cooking oils other than Savola, such as hypertension, heart attacks, etc. Guilt appeal can be used to depict a family grieving the death of someone who did not use Savola. Shame appeal could illustrate the case of a parent who cannot play catch with his kids because his fitness is compromised by not using Savola. However, the ad agency should take care to ensure that the negative appeals do not come across as too strong to the audiences, the credibility of the source in the ads is high, and the ad promises to relieve fears of health risks in a believable and efficient way. The ad is most persuasive when moderately discrepant with audience beliefs. If the ad merely says that using unhealthy oil leads to health risks, it only serves to reinforce the belief, and if the message exaggerates the health risks of not using Savola, audiences will only counterargue and disbelieve the ad.

20) What you say (ad copy) is more important than the number of times you say it (ad frequency).

TRUE

78) Although newspapers are timely and pervasive, magazines are typically more effective at building user and usage imagery.

TRUE

10) List and define some of the important shifts that have taken place in business and marketing practices.

Student answers will vary. Some possible answers include: • Reengineering is the appointment of teams to manage customer-value-building processes and break down walls between departments. • Outsourcing involves buying more goods and services from outside domestic or foreign vendors. • Benchmarking is the study of "best practice companies" to improve performance. • Supplier partnering focuses on partnering with fewer but better value-adding suppliers. • Customer partnering entails working more closely with customers to add value to their operations. • Merging involves acquiring or merging with firms in the same or complementary industries to gain economies of scale and scope. • Globalizing focuses on increasing the effort to "think global" and "act local." • Flattening refers to reducing the number of organizational levels to get closer to the customer. • Focusing involves determining the most profitable businesses and customers and focusing on them. • Accelerating involves designing the organization and setting up processes to respond more quickly to changes in the environment. • Empowering means encouraging and empowering personnel to produce more ideas and take more initiative. • Justifying means becoming more accountable by measuring, analyzing, and documenting the effects of marketing actions. • Broadening involves factoring the interests of customers, employees, shareholders, and other stakeholders into the activities of the enterprise. • Monitoring involves tracking what is said online and elsewhere and studying customers, competitors, and others to improve business practices.

89) Describe the adjustments that an advertiser of high-end laptops has to make to the cost-per-thousand measure.

Student answers will vary. The advertiser needs to apply several adjustments to the cost-per-thousand measure. First, they should adjust for audience quality. The product being high-end laptops, it is better advertised in a magazine read by 1 million college-going kids rather than 2 million housewives, in which case it would have an exposure value of almost zero. Second, the exposure value for the audience-attention probability must be accounted for. Readers of Chip or Digit may pay more attention to gadget ads than do readers of Newsweek. Third, the medium's editorial quality (prestige and believability) should be considered. People are more likely to believe a TV or radio ad and to become more positively disposed toward the brand when the ad is placed within a program they like. To this effect, the ads can be placed within a program like The Big Bang Theory. The fourth adjustment should be made in relation to ad placement policies and extra services (such as regional or occupational editions and lead-time requirements for magazines). Laptop ads would be better received and responded to in magazines that have greater circulation in urban areas with an office-going population or university towns with sizable student populations.

135) When asked about their preferred brand of instant noodles outside a supermarket setting, customers could not remember Nissin's name, but relied on the distinct packaging to help them spot the noodles while shopping. Describe how this problem can be addressed through the objective of Nissin's marketing communications.

Student answers will vary. The objective for Nissin's marketing communications objective should be to build brand awareness, i.e. fostering the consumer's ability to recognize or recall the brand within the category, in sufficient detail to make a purchase. Recognition is easier to achieve than recall — consumers asked to think of a brand of frozen entrées are more likely to recognize Stouffer's distinctive orange packages than to recall the brand. Brand recall is important outside the store; brand recognition is important inside the store. Brand awareness provides a foundation for brand equity.

53) Sandy's Stores is a small chain of grocery stores located in a few neighboring towns. The stores have always been largely self-service, but the company is considering making a switch to full-service stores. What can Sandy's do to justify this move?

Student answers will vary. To justify the increased staff costs of full-service retailing, Sandy's can add higher-value products to its line-up. The company can begin carrying a higher proportion of specialty goods.

128) Jayne runs a small grocery store in a small town. As there are only a few customers, the store does not require to stock goods in large quantities. Explain why sourcing products from a wholesaler will be beneficial for Jayne.

Student answers will vary. Wholesalers are able to select items and build the assortments Jayne needs, saving considerable work. Wholesalers achieve savings for Jayne by buying large carload lots and breaking the bulk into smaller units. Thus, Jayne can benefit from lower bulk prices while buying only as much as the store can sell. Wholesalers also hold inventories, thereby reducing inventory costs and risks for Jayne.

125) JGB manufactures the K-Nine brand of dog food that is carried in supermarkets across the country. The company has always used wholesalers instead of selling directly to the retailers. However, recently, the sales team at JGB has noticed that wholesalers don't aggressively promote JGB's product line. They often don't carry enough inventory and therefore don't fill customers' orders fast enough. However, the marketing team insists that the wholesaling route is the best. What reasons can the marketing team offer to justify this?

Student answers will vary. Wholesalers' sales forces help JGB reach many small business customers at a relatively low cost. They have more contacts, and the buyers often trust them more than they trust JGB. Wholesalers can often provide quicker delivery to buyers because they are closer to the buyers. Wholesalers absorb some risk by taking title and bearing the cost of theft, damage, spoilage, and obsolescence. Wholesalers supply information regarding competitors' activities, new products, price developments, and so on.

124) Identify three key success factors in developing and implementing a social marketing program.

Students may choose three of the following key success factors presented in the text: (1) choose target markets that are most ready to respond; (2) promote a single, doable behavior in clear, simple terms; (3) explain the benefits in compelling terms; (4) make it easy to adopt the behavior; (5) develop attention-grabbing messages and media; and (6) consider an education-entertainment approach. Page Ref: 641 Objective: 3 AACSB: Ethical understanding and reasoning abilities Difficulty: Moderate

122) What is sustainability? How is it related to the concept of greenwashing?

Sustainability is the ability to meet humanity's needs without harming future generations. It now tops many corporate agendas. Major corporations outline in great detail how they are trying to improve the long-term impact of their actions on communities and the environment. Heightened interest in sustainability has also unfortunately resulted in greenwashing, which gives products the appearance of being environmentally friendly without living up to that promise. Because of insincere firms jumping on the green bandwagon, consumers bring a healthy skepticism to environmental claims, but they are also unwilling to sacrifice product performance and quality. Many firms are rising to the challenge and are using the need for sustainability to fuel innovation. Page Ref: 635-636 Objective: 3 AACSB: Ethical understanding and reasoning abilities Difficulty: Moderate

84) What is sustainability? How is it related to the concept of greenwashing?

Sustainability is the ability to meet humanity's needs without harming future generations. It now tops many corporate agendas. Major corporations outline in great detail how they are trying to improve the long-term impact of their actions on communities and the environment. Heightened interest in sustainability has also unfortunately resulted in greenwashing, which gives products the appearance of being environmentally friendly without living up to that promise. Because of insincere firms jumping on the green bandwagon, consumers bring a healthy skepticism to environmental claims, but they are also unwilling to sacrifice product performance and quality. Many firms are rising to the challenge and are using the need for sustainability to fuel innovation.

122) Explain three techniques for stimulating creativity to generate better ideas.

The following are some of the techniques. (1) Attribute listing: List the attributes of an object, such as a screwdriver. Then modify each attribute, such as replacing the wooden handle with plastic, providing torque power, adding different screw heads, and so on. (2) Forced relationships: List several ideas and consider each in relationship to each of the others. (3) Morphological analysis: Start with a problem, such as "getting something from one place to another via a powered vehicle." Now think of dimensions, such as the type of platform (cart, chair, sling, bed), the medium (air, water, oil, rails), and the power source (compressed air, electric motor, magnetic fields). By listing every possible combination, you can generate many new solutions. (4) Reverse assumption analysis: List all the normal assumptions about an entity and then reverse them. Instead of assuming that a restaurant has menus, charges for food, and serves food, reverse each assumption. (5) New contexts: Take familiar processes, such as people-helping services, and put them into a new context. (6) Mind mapping: Start with a thought, such as a car, write it on a piece of paper, then think of the next thought that comes up (say Mercedes), link it to car, then think of the next association (Germany), and do this with all associations that come up with each new word. Perhaps a whole new idea will materialize. Students may explain any three of these techniques. Page Ref: 579-580 Objective: 3 Difficulty: Easy

127) What challenges do marketers face in managing trade promotions?

The growing power of large retailers has increased their ability to demand trade promotion at the expense of consumer promotion and advertising. The company's sales force and its brand managers are often at odds over trade promotion. The sales force says local retailers will not keep the company's products on the shelf unless they receive more trade promotion money, whereas brand managers want to spend their limited funds on consumer promotion and advertising. Manufacturers face several challenges in managing trade promotions. First, they often find it difficult to police retailers to make sure they are doing what they agreed to do. Manufacturers increasingly insist on proof of performance before paying any allowances. Second, some retailers are doing forward buying — that is, buying a greater quantity during the deal period than they can immediately sell. Retailers might respond to a 10 percent-off-case allowance by buying a 12-week or longer supply. The manufacturer must then schedule more production than planned and bear the costs of extra work shifts and overtime. Third, some retailers are diverting, buying more cases than needed in a region where the manufacturer offers a deal and shipping the surplus to their stores in non-deal regions. Manufacturers handle forward buying and diverting by limiting the amount they will sell at a discount, or by producing and delivering less than the full order in an effort to smooth production. Ultimately, many manufacturers feel trade promotion has become a nightmare. It contains layers of deals, is complex to administer, and often leads to lost revenues.

73) Gravity is a company that markets customized T-shirts to its customers. Gravity runs an ad that seeks to attract more customers. Apply the macromodel of the communications process to describe the communication between Gravity and its target customers.

The macromodel of the communications process comprises of nine key factors in effective communication. Two represent the major parties — sender (Gravity) and receiver (audience). Two represent the major tools — message (content of the ad) and media (print, TV, radio, etc). Four represent the major communications functions — encoding (creation and transmission of the ad by Gravity), decoding (the reception and comprehension of the ad by the audience), response (either ignoring the message or buying Gravity shirts), and feedback (customers providing information about the ad to measure its effectiveness). The last element is noise, random and competing messages that may interfere with the intended communication. Examples of noise in this case could be misunderstood messages from the ad, similar ads from competitors, etc.

29) What are the elements of the marketing communications mix?

The marketing communications mix consists of eight major modes of communication: 1. Advertising — Any paid form of nonpersonal presentation and promotion of ideas, goods, or services by an identified sponsor via print, broadcast, network, electronic, and display media. 2. Sales promotion — A variety of short-term incentives to encourage trial or purchase of a product or service including consumer promotions, trade promotions, and business and sales force promotions. 3. Events and experiences — Company-sponsored activities and programs designed to create daily or special brand-related interactions with consumers, including sports, arts, entertainment, and cause events as well as less formal activities. 4. Public relations and publicity — A variety of programs directed internally to employees of the company or externally to consumers, other firms, the government, and media to promote or protect a company's image or its individual product communications. 5. Direct marketing — Use of mail, telephone, fax, e-mail, or Internet to communicate directly with or solicit response or dialogue from specific customers and prospects. 6. Interactive marketing — Online activities and programs designed to engage customers or prospects and directly or indirectly raise awareness, improve image, or elicit sales of products and services. 7. Word-of-mouth marketing — People-to-people oral, written, or electronic communications that relate to the merits or experiences of purchasing or using products or services. 8. Personal selling — Face-to-face interaction with one or more prospective purchasers for the purpose of making presentations, answering questions, and procuring orders.

128) I-ball, a cell phone manufacturer introduces a cell phone targeted at customers aged above 70. It has features such as loud volume, large keys, and so forth. How do you classify this product innovation? What could be a possible disadvantage of this product?

This can be considered a new-to-the-world product. New-to-the-world products are products that create an entirely new market. The product has the greatest cost and risk of all product types. Page Ref: 570 Objective: 1 AACSB: Analytic skills Difficulty: Moderate

122) Crafted Jewels is a jewelry manufacturer that sells its designs to various jewelry retailers. The retailers often complain that the company's online store acts as a competitor and reduces their profitability. What type of conflict is this? Briefly explain.

This is a case of multichannel conflict. It exists when the manufacturer has established two or more channels that sell to the same market.

146) Fred's company has recently sold its resin-producing plant to a local concern in India. As part of the sales price, his company agrees to accept as partial payment the production of the resin at an agreed upon price for six years. This is an example of what type of countertrade?

This is an example of a buyback arrangement. In such arrangements, the seller sells a plant, equipment, or technology to another country and agrees to accept as partial payment products manufactured with the supplied equipment. Page Ref: 404 Objective: 3 AACSB: Analytic skills Difficulty: Moderate

123) UltraMotion Pictures produces and distributes music and television entertainment in the United States. The company distributes music in partnership with a large music retailer, Fromen Tunes. Fromen executives often work for a short time with UltraMotion and some of the UltraMotion executives work at the retail outlets to study Fromen's operations. This strategy minimizes the conflicts between partners. What strategy is being used here? Briefly explain.

This strategy of managing channel conflict is called an employee exchange which involves the exchange of persons between two or more channel levels. Thus participants can grow to appreciate each other's point of view.

121) Many companies are beginning to realize that they are not really market and customer driven, they are product and sales driven. In the attempt to transform themselves into true market-driven companies, firms are required to change. Describe and explain what changes are necessary.

To be truly market-driven, companies need to develop a company-wide passion for customers, organize around customer segments instead of around products, and develop a deep understanding of customers through qualitative and quantitative research. Additionally, the organization must be creative; the firm must build capability in strategic innovation and imagination. This capability comes from assembling tools, processes, skills, and measures that let the firm generate more and better new ideas than its competitors. Page Ref: 630 Objective: 2 Difficulty: Easy

28) What are the legal and social issues associated with advertising?

To break through clutter, some advertisers believe they have to be edgy and push the boundaries of what consumers are used to seeing in advertising. In doing so, marketers must be sure advertising does not overstep social and legal norms or offend the general public, ethnic groups, racial minorities, or special-interest groups. A substantial body of laws and regulations governs advertising. Under US law, advertisers must not make false claims, such as stating that a product cures something when it does not. They must avoid false demonstrations, such as using sand-covered Plexiglas instead of sandpaper to demonstrate that a razor blade can shave sandpaper. It is illegal in the United States to create ads that have the capacity to deceive, even though no one may actually be deceived. The challenge is telling the difference between deception and "puffery" — simple exaggerations that are not meant to be believed and that are permitted by law. Sellers in the United States are legally obligated to avoid bait-and-switch advertising that attracts buyers under false pretenses. Advertising can play a more positive broader social role. The Ad Council is a nonprofit organization that uses top-notch industry talent to produce and distribute public service announcements for nonprofits and government agencies. From its early origins with "Buy War Bonds" posters, the Ad Council has tackled innumerable pressing social issues through the years. One of its recent efforts featured beloved Sesame Street stars Elmo and Gordon exhorting children to wash their hands in the face of the H1N1 flu virus.

76) What are the features of an ideal advertising campaign?

To increase the odds for a successful marketing communications campaign, marketers must attempt to increase the likelihood that each step occurs. For example, the ideal ad campaign would ensure that: • The right consumer is exposed to the right message at the right place and at the right time. • The ad causes the consumer to pay attention but does not distract from the intended message. • The ad properly reflects the consumer's level of understanding of and behaviors with the product and the brand. • The ad correctly positions the brand in terms of desirable and deliverable points-of-difference and points-of-parity. • The ad motivates consumers to consider purchase of the brand. • The ad creates strong brand associations with all these stored communications effects so they can have an impact when consumers are considering making a purchase.

73) A large retail chain in the United States decides to expand its operations by adding an online site for e-commerce. This is called a(n) ________ company. A) B2B B) brick-and-mortar C) m-commerce D) pure-click E) brick-and-click

e


संबंधित स्टडी सेट्स

Second Language Acquisition Theories

View Set

LGS 200 Chapter 21: Employment Discrimination

View Set